You are on page 1of 77

fo/u fopkjr Hkh# tu] ugha vkjEHks dke] foifr ns[k NksM+s rqjra e/;e eu dj ';keA

iq#"k flag ladYi dj] lgrs foifr vusd] ^cuk^ u NksM+s /;s; dks] j?kqcj jk[ks VsdAA
jfpr% ekuo /keZ iz.ksrk
ln~xq# Jh j.kNksM+nklth egkjkt

STUDY PACKAGE
Subject : Mathematics
Topic : VECTORS & 3 DIMENSIONAL GEOMETRY
Available Online : www.MathsBySuhag.com

Index
1. Theory
2. Short Revision
3. Exercise (Ex. 1 + 5 = 6)
4. Assertion & Reason
5. Que. from Compt. Exams
6. 39 Yrs. Que. from IIT-JEE(Advanced)
7. 15 Yrs. Que. from AIEEE (JEE Main)

Student’s Name :______________________


Class :______________________
Roll No. :______________________

Address : Plot No. 27, III- Floor, Near Patidar Studio,


Above Bond Classes, Zone-2, M.P. NAGAR, Bhopal
: 0 903 903 7779, 98930 58881, WhatsApp 9009 260 559
www.TekoClasses.com www.MathsBySuhag.com
THREE DIMENSIONAL GEOMETRY
Coordinate of a point in space
FREE Download Study Package from website: www.TekoClasses.com & www.MathsBySuhag.com

There are infinite number of points in space. We want to identify each and every point of space with the help of
three mutually perpendicular coordinate axes OX, OY and OZ.
Three mutually perpendicular lines OX, OY, OZ are considered as the three axes.

page 2 of 77
The plane formed with the help of x and y axes is called x-y plane, similarly y & z axes form y-z plane and z and
x axes form z - x plane.
Consider any point P in the space, Drop a perpendicular from that point to x -y plane, then the algebraic length
of this perpendicular is considered as z-coordinate and from foot of the perpendicular drop perpendiculars to x
and y axes. These algebraic lengths of perpendiculars are considered as y and x coordinates respectively.

Teko Classes, Maths : Suhag R. Kariya (S. R. K. Sir), Bhopal Phone : 0 903 903 7779, 0 98930 58881.
1. Vector representation of a point in space
If coordinate of a point P in space is (x, y, z) then the position vector of the point P with respect to the
same origin is x î + y ĵ + z k̂ .
2. Distance formula Distance between any two points (x 1, y1, z1) and (x 2, y2, z2) is given as

( x 1 − x 2 ) 2 + ( y 1 − y 2 ) 2 + ( z1 − z 2 ) 2
Vector method We know that if position vector of two points A and B are given as OA and OB then
AB = | OB – OA |
⇒ AB = |(x 2i + y2 j + z2k) – (x 1i + y1j + z1k)| ⇒AB = ( x 2 − x 1 ) 2 + ( y 2 − y 1 ) 2 + ( z 2 − z1 ) 2
3. Distance of a point P from coordinate axes
Let PA, PB and PC are distances of the point P(x, y, z) from the coordinate axes OX, OY and OZ
respectively then
PA = y 2 + z 2 , PB = z 2 + x 2 , PC = x 2 + y 2
Example : Show that the points (0, 7, 10), (– 1, 6, 6) and (– 4, 9, 6) form a right angled isosceles triangle.
Solution: Let A ≡ (0, 7, 10), B ≡ (–1, 6, 6), C ≡ (– 4, 9, 6)
AB2 = (0 + 1)2 + (7 – 6)2 + (10 – 6)2 = 18 ∴ AB = 3 2
Similarly ∴ BC = 3 2 ,
& AC = 6
Clearly AB2 +BC2 = AC2 ∴ ∠ ABC = 90°
Also AB = BC
Hence ∆ ABC is right angled isosceles.
Example : Show by using distance formula that the points (4, 5, –5), (0, –11, 3) and (2, –3, –1) are collinear.
Solution Let A ≡ (4, 5, –5), B ≡ (0, –11, 3), C ≡ (2, –3, –1).
AB = ( 4 − 0)2 + (5 + 11)2 + ( −5 − 3)2 = 336 = 4 × 84 = 2 84

BC = (0 − 2)2 + ( −11 + 3)2 + (3 + 1)2 = 84 AC = ( 4 − 2)2 + (5 + 3 )2 + ( −5 + 1)2 = 84


BC + AC = AB
Hence points A, B, C are collinear and C lies between A and B.
Example : Find the locus of a point which moves such that the sum of its distances from points A(0, 0, – α)
and B(0, 0, α) is constant.
Solution. Let the variable point whose locus is required be P(x, y, z)
Given PA + PB = constant = 2a (say)
∴ ( x − 0 ) 2 + ( y − 0 ) 2 + ( z + α )2 + ( x − 0)2 + ( y − 0)2 + ( z − α )2 = 2a
1
⇒ = 2a – x 2 + y 2 + ( z − α )2
2

⇒ x 2 + y2 + z2 + α2 + 2zα = 4a2 + x 2 + y2 + z2 + α2 – 2zα – 4a x 2 + y 2 + ( z − α )2

⇒ 4zα – 4a2 = – 4a x 2 + y 2 + ( z − α )2
z 2α 2
⇒ + a2 – 2zα = x 2 + y2 + z2 + α2 – 2zα
a2
 α2  x 2 + y 2 z2
 
or, x 2 + y2 + z2 1 − 2  = a2 – α2 or, 2 + = 1 This is the required locus.
 a  a − α 2 a2
FREE Download Study Package from website: www.TekoClasses.com & www.MathsBySuhag.com

Self practice problems :


1. One of the vertices of a cuboid is (1, 2, 3) and the edges from this vertex are along the +ve x-axis,
+ve y-axis and +z axis respectively and are of length 2, 3, 2 respectively find out the vertices.
Ans. (1, 2, 5), (3, 2, 5), (3, 2, 3), (1, – 1, 3), (1, – 1, 5), (3, – 1, 5), (3, – 1, 3).
2. Show that the points (0, 4, 1), (2, 3, –1), (4, 5, 0) and (2, 6, 2) are the vertices of a square.

page 3 of 77
3. Find the locus of point P if AP2 – BP2 = 18, where A ≡ (1, 2, – 3) and B ≡ (3, – 2, 1)
Ans. 2x – 4y + 4z – 9 = 0
4. Section Formula
If point P divides the distance between the points A (x 1, y1, z1) and B (x 2, y2, z2) in the ratio of m : n, then
coordinates of P are given as
 mx 2 + nx1 my 2 + ny1 mz 2 + nz1 
 , , 

Teko Classes, Maths : Suhag R. Kariya (S. R. K. Sir), Bhopal Phone : 0 903 903 7779, 0 98930 58881.
 m+n m+n m+n 
Note :- Mid point
 x 1 + x 2 y 1 + y 2 z1 + z 2 
 , , 
 2 2 2 
5. Centroid of a triangle

 x 1 + x 2 + x 3 y 1 + y 2 + y 3 z1 + z 2 + z 3 
G ≡ , , 
 3 3 3 

 ax1 + bx 2 + cx 3 ay 1 + by 2 + cy 3 az1 + bz 2 + cz 3 
6. Incentre of triangle ABC:  , , 
 a+b+c a+b+c a+b+c 
Where AB = c, BC = a, CA = b
7. Centroid of a tetrahedron A (x 1, y1, z1) B (x 2, y2, z2) C (x 3, y3, z3) and D (x 4, y4, z4) are the
vertices of a tetrahedron then coordinate of its centroid (G) is given as

 ∑ ∑ ∑
xi
,
yi
,
z i 
 4 4 4 
 
Example : Show that the points A(2, 3, 4), B(–1, 2, –3) and C(–4, 1, –10) are collinear. Also find the ratio in
which C divides AB.
Solution: Given A ≡ (2, 3, 4), B ≡ (–1, 2, –3), C ≡ (– 4, 1, –10).

A (2, 3, 4) B (–1, 2, –3)


Let C div ide AB internally in the ratio k : 1, then
 − k + 2 2k + 3 − 3k + 4 
C≡  , , 
 k +1 k +1 k +1 
−k + 2
∴ =–4 ⇒ 3k = – 6 ⇒ k = –2
k +1
2k + 3 −3k + 4
For this value of k, = 1, and = –10
k +1 k +1
Since k < 0, therefore k divides AB externally in the ratio 2 : 1 and pointsA(5,
A, B,
4, C
6)are collinear.
Example : The vertices of a triangle are A(5, 4, 6), B(1, –1, 3) and C(4, 3, 2). The internal bisector of ∠ BAC
meets BC in D. Find AD.
Solution AB = 42 + 52 + 32 = 5 2
AC = 12 + 12 + 4 2 = 3 2
Since AD is the internal bisector of ∠ BAC
B D C
BD AB 5 (1, –1, 3) (4, 3, 2)
∴ = =
DC AC 3
∴ D divides BC internally in the ratio 5 : 3
 5 × 4 + 3 × 1 5 × 3 + 3( −1) 5 × 2 + 3 × 3   23 12 19 
∴ D≡  , ,  or, D=  , , 
 5+3 5+3 5+3   8 8 8 
2 2 2
 23   12   19 
∴ AD = 5 −  + 4 −  + 6 − 
 8   8   8 
1530
= unit
8
FREE Download Study Package from website: www.TekoClasses.com & www.MathsBySuhag.com

Example :
If the points P, Q, R, S are (4, 7, 8), (– 1, – 2, 1), (2, 3, 4) and (1, 2, 5) respectively, show that PQ and
RS intersect. Also find the point of intersection.
Solution

page 4 of 77
Let the lines PQ and RS intersect at point A.
Let A divide PQ in the ratio λ : 1, then P(4, 7, 8) S(1, 2, 5)
 − λ + 4 − 2λ + 7 λ + 8  λ l
A≡  , , . .... (1)
 λ +1 λ +1 λ +1
1
Let A divide RS in the ratio k : 1, then A
 k + 2 2k + 3 5k + 4  K
A≡  , , 

Teko Classes, Maths : Suhag R. Kariya (S. R. K. Sir), Bhopal Phone : 0 903 903 7779, 0 98930 58881.
..... (2)
 k +1 k +1 k +1  R(2, 3, 4) Q(–1, –2, 1)
From (1) and (2), we have,
−λ + 4 k + 2
= ..... (3)
λ +1 k +1
−2λ + 7 2k + 3
= ..... (4)
λ +1 k +1
λ + 8 5k + 4
= ..... (5)
λ +1 k +1
From (3), – λk – λ + 4k + 4 = λk + 2λ + k + 2
or 2λk + 3λ – 3k – 2 = 0 ..... (6)
From (4), –2λk – 2λ + 7k + 7 = 2λk + 3λ + 2k + 3
or 4λk + 5λ – 5k – 4 = 0 ..... (7)
Multiplying equation (6) by 2, and subtracting from equation (7), we get
–λ+k=0 or , λ=k
Putting λ = k in equation (6), we get
2λ2 + 3λ – 3λ – 2 = 0
or, λ = ± 1.
But λ ≠ –1, as the co-ordinates of P would then be underfined and in this case
PQ || RS, which is not true.
∴ λ = 1 = k.
Clearly λ k = 1 satisfies eqn. (5).
Hence our assumption is correct
 − 1+ 4 − 2 + 7 1+ 8  3 5 9
∴ A≡  , ,  or, A≡  , , .
 2 2 2  2 2 2
Self practice problems:
1. Find the ratio in which xy plane divides the line joining the points A (1, 2, 3) and B (2, 3, 6).
Ans. – 1 : 2
2. Find the co-ordinates of the foot of perpendicular drawn from the point A(1, 2, 1) to the line joining the
point B(1, 4, 6) and C(5, 4, 4).
Ans. (3, 4, 5)
8 
3. Two vertices of a triangle are (4, –6, 3) and (2, –2, 1) and its centroid is  , − 1, 2  . Find the third vertex.
 3 
Ans. (2, 5, 2)
4. If centroid of the tetrahedron OABC, where co-ordinates of A, B, C are (a, 2, 3), (1, b, 2) and (2, 1, c)
respectively be (1, 2, 3), then find the distance of point (a, b, c) from the origin.
Ans. 107
 1 
5. Show that  − , 2, 0  is the circumcentre of the triangle whose vertices are A (1, 1, 0), B (1, 2, 1) and
 2 
C (– 2, 2, –1) and hence find its orthocentre. Ans. (1, 11, 0)
8. Direction Cosines And Direction Ratios
(i) Direction cosines: Let α, β, γ be the angles which a directed
line makes with the positive directions of the axes of x, y and
z respectively, then cos α, cosβ, cos γ are called the direction
cosines of the line. The direction cosines are usually denoted
by (, m, n).
Thus  = cos α, m = cos β , n = cos γ .
(ii) If , m, n be the direction cosines of a line, then 2 + m 2 + n2 = 1
(iii) Direction ratios: Let a, b, c be proportional to the direction cosines , m, n then a, b, c are
called the direction ratios.
If a, b, c, are the direction ratios of any line L then a î + b ĵ + ck̂ will be a vector parallel to the line L.
If , m, n are direction cosines of line L then  î + m ĵ + n k̂ is a unit vector parallel to the line L.
(iv) If , m, n be the direction cosines and a, b, c be the direction ratios of a vector, then
FREE Download Study Package from website: www.TekoClasses.com & www.MathsBySuhag.com

 a b c 
 = ,m = ,n = 
 2 2 2 2 2 2 2 2 2 
 a + b + c a + b + c a + b + c 
−a −b −c

page 5 of 77
or = ,m= ,n=
a2 + b2 + c 2 a2 + b2 + c 2 a2 + b2 + c 2
(v) If OP = r, when O is the origin and the direction cosines of OP are , m, n then the coordinates
of P are (r, mr, nr).
I f d i r e c t i o n c o s i n e s , m,n, |AB| = r, and the coordinates of A is (x 1, y1, z1)
o f t h e l i n e A B a r e

then the coordinates of B is given as (x 1 + r , y1 + rm, z1 + rn)


(vi) If the coordinates P and Q are (x 1, y1, z1) and (x 2, y2, z2) then the direction ratios of line PQ are,

Teko Classes, Maths : Suhag R. Kariya (S. R. K. Sir), Bhopal Phone : 0 903 903 7779, 0 98930 58881.
x 2 − x1
a = x 2 − x 1, b = y2 − y1 & c = z2 − z1 and the direction cosines of line PQ are  = ,
| PQ |
y 2 − y1 z 2 − z1
m= and n =
| PQ | | PQ |
(vii) Direction cosines of axes: Since the positive x −axis makes angles 0º, 90º, 90º with axes of x,
y and z respectively. Therefore
Direction cosines of x −axis are (1, 0, 0)
Direction cosines of y−axis are (0, 1, 0)
Direction cosines of z−axis are (0, 0, 1)
Example : If a line makes angles α, β, γ with the co-ordinate axes, prove that sin2α + sin2β + sinγ2 = 2.
Solution Since a line makes angles α, β, γ with the co-ordinate axes,
hence cosα, cosβ, cosγ are its direction cosines
∴ cos2α + cos2β + cos2γ = 1
⇒ (1 – sin2α) + (1 – sin2β) + (1 – sin2γ) = 1
⇒ sin2α + sin2β + sin2γ = 2.
Example : Find the direction cosines , m, n of a line which are connected by the relations  + m + n = 0,
2mn + 2m  – n = 0
Solution Given,  + m + n = 0 ..... (1)
2mn + 2m  – n = 0 ..... (2)
From (1), n = – ( + m).
Putting n = – ( + m) in equation (2), we get,
– 2m( + m) + 2m  + ( + m)  = 0
or, – 2m  – 2m 2 + 2m  + 2 + m  = 0
or, 2 + m  – 2m 2 = 0
2
 
or,   +  – 2 = 0 [dividing by m 2]
m m
 − 1± 1+ 8 − 1± 3
or = = = 1, –2
m 2 2

Case I. when = 1 : In this case m = 
m
From (1), 2 + n = 0 ⇒ n = – 2
∴ :m:n=1:1:–2
∴ Direction ratios of the line are 1, 1, – 2
∴ Direction cosines are
1 1 −2
± ,± ,±
2 2
1 + 1 + ( −2 ) 2
12 + 12 + ( −2)2 1 + 12 + ( −2)2
2

1 1 −2 1 1 2
or, , , or – ,– ,
6 6 6 6 6 6

Case II. When = – 2 : In this case  = – 2m
m
From (1), – 2m + m + n = 0 ⇒ n=m
∴  : m : n = – 2m : m : m
=–2:1:1
∴ Direction ratios of the line are – 2, 1, 1.
∴ Direction cosines are
−2 −1 −1 −2 1 , 1 .
, , or, ,
2 2 2 2 2 2 6
( −2) + 1 + 1 ( −2 ) + 1 + 1 ( −2) + 12 + 12
2 6 6

Self practice problems:


1. Find the direction cosine of a line lying in the xy plane and making angle 30° with x-axis.
1 3
Ans. m=± ,= ,n=0
2 2
2. A line makes an angle of 60° with each of x and y axes, find the angle which this line makes with
FREE Download Study Package from website: www.TekoClasses.com & www.MathsBySuhag.com

z-axis. Ans. 45°


3. A plane intersects the co-ordinates axes at point A(a, 0, 0), B(0, b, 0), C(0, 0, c) O is origin. Find the
direction ratio of the line joining the vertex B to the centroid of face AOC.
a c

page 6 of 77
Ans. , – b,
3 3
4. A line makes angles α, β, γ, δ with the four diagonals of a cube, prove that
4
cos2α + cos2β + cos2γ + cos2δ = .
3
9. Angle Between Two Line Segments:
If two lines have direction ratios a1, b1, c1 and a 2, b2, c2 respectively then we can consider two vectors

Teko Classes, Maths : Suhag R. Kariya (S. R. K. Sir), Bhopal Phone : 0 903 903 7779, 0 98930 58881.
parallel to the lines as a1i + b1j + c1k and a2i + b2j + c2k and angle between them can be given as.
a 1a 2 + b1b 2 + c1c 2
cos θ = .
a1 + b12 + c12 a 22 + b 22 + c 22
2

a1 b c1
(i)The line will be perpendicular if a1a2 + b1b2 + c1c2 = 0 (ii)The lines will be parallel if = 1 =
a2 b2 c2
(iii) Two parallel lines have same direction cosines i.e.  1 =  2, m 1 = m 2, n1 = n2
3 1 3 3 1 3
Example : What is the angle between the lines whose direction cosines are − , ,− ;− , , ?
4 4 2 4 4 2
Solution Let θ be the required angle, then
cosθ =  1 2 + m 1m 2 + n 1n 2
 3   3   1   1   3   3 

= − 4  − 4  +  4   4  + − 2  .  2 
         
3 1 3 1
= + − =− ⇒ θ = 120°,
16 16 4 2
Example : Find the angle between any two diagonals of a cube.
Solution The cube has four diagonals
Y

G(0, a, 0) F(a, a, 0)

E (a, a, a)
D
(0, a, a)

O A X
(0, 0, 0) (a, 0, 0)

C(0, 0, a) B(a, 0, a)
Z’
OE, AD, CF and GB
The direction ratios of OE are
a, a, a or, 1, 1, 1
1 1 1
∴ its direction cosines are , ,
.
3 3 3
Direction ratios of AD are – a, a, a. or, – 1, 1, 1.
−1 1 1
∴ its direction cosines are , , .
3 3 3
Similarly, direction cosines of CF and GB respectively are
1 1 −1 1 −1 1
, , and , , .
3 3 3 3 3 3
We take any two diagonals, say OE and AD
Let θ be the acute angle between them, then
 1   −1  1   1   1   1  1  1
cosθ =  

+
 
. 
 
+
 
.
 
 =
 3 or, θ = cos–1   .
 3 3  3  3  3  3 3
Example : If two pairs of opposite edges of a tetrahedron are mutually perpendicular, show that the third pair
will also be mutually perpendicular.
Solution: Let OABC be the tetrahedron where O is the origin and co-ordinates of A, B, C be (x 1, y1, z1),
(x 2, y2, z2), (x 3, y3, x 3) respectively.
FREE Download Study Package from website: www.TekoClasses.com & www.MathsBySuhag.com

A (x1, y1, z1)

page 7 of 77
O (0, 0, 0)

B C
(x2, y2, z2) (x3, y3, z3)
Let OA ⊥ BC and OB ⊥ CA .
We have to prove that

Teko Classes, Maths : Suhag R. Kariya (S. R. K. Sir), Bhopal Phone : 0 903 903 7779, 0 98930 58881.
OC ⊥ BA .
Now, direction ratios of OA are x 1 – 0, y1 – 0, z 1 – 0 or, x 1, y1, z1
direction ratios of BC are (x 3 – x 2), (y3 – y2), (z3 – z2).
∵ OA ⊥ BC .
∴ x 1(x 3 – x 2) + y1(y3 – y2) + z1(z3 – z2) = 0 ..... (1)
Similarly,
∵ OB ⊥ CA
∴ x 2(x 1 – x 3) + y2(y1 – y3) + z2(z1 – z3) = 0 ..... (2)
Adding equations (1) and (2), we get
x 3(x 1 – x 2) + y3(y1 – y2) + z3(z1 – z2) = 0
∴ OC ⊥ BA [ ∵ direction ratios of OC are x 3, y3, z3 and that of BA are (x 1 – x 2), (y1 – y2), (z1 – z2)]
Self practice problems:
1. Find the angle between the lines whose direction cosines are given by  + m + n = 0 and
2 + m 2 – n2 = 0 Ans. 60°
2. P (6, 3, 2)
Q (5, 1, 4)
R (3, 3, 5)
are vertices of a ∆ find ∠Q. Ans. 90°
3. Show that the direction cosines of a line which is perpendicular to the lines having directions cosines
1 m 1 n1 and 2 m 2 n2 respectively are proportional to
m 1n2 – m2n1 , n1 2 – n2 1,  1m 2 –  2m 1
1 0 . Projection of a line segment on a line
(i) If the coordinates P and Q are (x 1, y1, z1) and (x2, y2, z2) then the projection of the line segments
PQ on a line having direction cosines , m, n is (x 2 − x1 ) + m(y 2 − y1 ) + n(z2 − z1 )
 
   a.b
(ii) Vector form: projection of a vector a on another vector b is a . b̂ = 
|b|
→ 
In the above case we can consider PQ as (x 2 – x 1) î + (y2 – y1) ĵ + (z2 – z1) k̂ in place of a and
 
 î + m ĵ + n k̂ in place of b . (iii)  | r |, m | r | & n | r | are the projection of r in OX, OY &
 
OZ axes. (iv) r = | r | ( î + m ĵ + n k̂ )
Solved Example : Find the projection of the line joining (1, 2, 3) and (–1, 4, 2) on the line having direction
ratios 2, 3, – 6.
Solution Let A ≡ (1, 2, 3), B ≡ (–1, 4, 2)
B
A

90° 90°
P L M Q
Direction ratios of the given line PQ are 2, 3, – 6
22 + 3 2 + ( −6)2 = 7 ∴ direction cosines of PQ are
2 3 6
, ,–
7 7 7
Projection of AB on PQ
=  (x 2 – x 1) + m(y2 – y1) + n(z2 – z1)
2 3 6 −4 + 6 + 6 8
= (–1 – 1) + (4 – 2) – (2 – 3) = =
7 7 7 7 7
Self practice problems:
1. A (6, 3, 2), B (5, 1, 1,), C(3, –1, 3) D (0, 2, 5)
Find the projection of line segment AB on CD line. Ans. 5/7
2. The projections of a directed line segment on co-ordinate axes are – 2, 3, – 6. Find its length and
12 4 3
direction cosines. Ans. 13 ; , ,
13 13 13
FREE Download Study Package from website: www.TekoClasses.com & www.MathsBySuhag.com

3. Find the projection of the line segment joining (2, – 1, 3) and (4, 2, 5) on a line which makes equal
7
acute angles with co-ordinate axes. Ans.
3

page 8 of 77
A PLANE
If line joining any two points on a surface lies completely on it then the surface is a plane.
OR
If line joining any two points on a surface is perpendicular to some fixed straight line. Then this surface
is called a plane. This fixed line is called the normal to the plane.
1 1 . Equation Of A Plane
(i) Normal form of the equation of a plane is x + my + nz = p, where, ,m n are the direction

Teko Classes, Maths : Suhag R. Kariya (S. R. K. Sir), Bhopal Phone : 0 903 903 7779, 0 98930 58881.
cosines of the normal to the plane and p is the distance of the plane from the origin.
(ii) General form: ax + by + cz + d = 0 is the equation of a plane, where a, b, c are the
direction ratios of the normal to the plane.
(iii) The equation of a plane passing through the point (x 1, y1, z1) is given by
a (x − x 1) + b( y − y1) + c (z − z1) = 0 where a, b, c are the direction ratios of the normal
to the plane.
(iv) Plane through three points: The equation of the plane through three non−collinear points
x y z 1
x 1 y1 z 1 1
(x 1, y1, z1), (x 2, y2, z2), (x 3, y3, z3) is =0
x 2 y2 z2 1
x 3 y3 z3 1
x y z
(v) + + =1
Intercept Form: The equation of a plane cutting intercept a, b, c on the axes is
a b c
(vi) Vector form: The equation of a plane passing through a point having position vector a &
       
normal to vector n is ( r − a ). n = 0 or r . n = a . n
Note: (a) Vector equation of a plane normal to unitvector n̂ and at a distance d from the origin is
. =d
r n
(b) Coordinate planes (i) Equation of yz−plane is x = 0 (ii) Equation of xz−plane is y = 0
(iii) Equation of xy−plane is z = 0
(c) Planes parallel to the axes:
If a = 0, the plane is parallel to x −axis i.e. equation of the plane parallel to the x −axis is
by + cz + d = 0.
Similarly, equation of planes parallel to y−axis and parallel to z−axis are ax + cz +d = 0
and ax + by + d = 0 respectively.
(d) Plane through origin: Equation of plane passing through origin is ax + by + cz = 0.
(e) Transformation of the equation of a plane to the normal form: To reduce any equation
ax + by + cz − d = 0 to the normal form, first write the constant term on the right hand
side and make it positive, then divide each term by a 2 + b2 + c2 , where a, b, c are
coefficients of x, y and z respectively e.g.
ax by cz d
+ + =
± a 2 + b 2 + c2 ± a 2 + b2 + c2 ± a 2 + b2 + c2 ± a 2 + b2 + c2
Where (+) sign is to be taken if d > 0 and (−) sign is to be taken if d < 0.
(f) Any plane parallel to the given plane ax + by + cz + d = 0 is ax + by + cz + λ = 0.
Distance between two parallel planes ax + by + cz + d1 = 0 and ax + by + cz + d2 = 0 is
| d1 − d2 |
given as
a2 + b2 + c 2
(g) Equation of a plane passing through a given point & parallel to the given vectors:

The equation of a plane passing through a point having position vector a and parallel to
     
b & c is r = a + λ b + µ c (parametric form) where λ & µ are scalars.
     
or r . (b × c) = a . (b × c) (non parametric form)
(h) A plane ax + by + cz + d = 0 divides the line segment joining (x 1, y1, z1) and (x 2, y2, z2). in the
 ax1 + by1 + cz1 + d 
ratio − 
 ax 2 + by2 + cz2 + d 
(i) The xy−plane divides the line segment joining the points (x 1, y1, z1) and (x 2, y2, z2) in the ratio
z1 x1 y1
− z . Similarly yz−plane in − x and zx −plane in − y
FREE Download Study Package from website: www.TekoClasses.com & www.MathsBySuhag.com

2 2 2
(j) Coplanarity of four points
The points A(x 1 y1 z1), B(x 2 y2 z2) C(x 3 y3 z3) and D(x 4 y4 z4) are coplaner then
x 2 − x 1 y 2 − y 1 z 2 − z1

page 9 of 77
x 3 − x 1 y 3 − y 1 z 3 − z1
=0
x 4 − x 1 y 4 − y 1 z 4 − z1
   
very similar in vector method the points A ( r1 ), B( r2 ), C( r3 ) and D( r4 ) are coplanar if
     
[ r4 – r1 , r4 – r2 , r4 – r3 ] = 0
Example : Find the equation of the plane upon which the length of normal from origin is 10 and direction ratios

Teko Classes, Maths : Suhag R. Kariya (S. R. K. Sir), Bhopal Phone : 0 903 903 7779, 0 98930 58881.
of this normal are 3, 2, 6.
Solution If p be the length of perpendicular from origin to the plane and , m, n be the direction cosines
of this normal, then its equation is
x + my + nz = p ..... (1)
Here p = 10; Direction ratios of normal to the plane are 3, 2, 6
3 2 + 22 + 6 2 = 7 ∴ Direction cosines of normal to the required plane are
3 2 6
= ,m= ,n=
7 7 7
Putting the values of , m, n, p in (1), equation of required plane is
3 2 6
x + y + z = 10 or, 3x + 2y + 6z = 70
7 7 7
Example : Show that the points (0, – 1, 0), (2, 1, – 1), (1, 1, 1), (3, 3, 0) are coplanar.
Solution Let A ≡ (0, – 1, 0), B ≡ (2, 1, – 1), C ≡ (1, 1, 1) and D ≡ (3, 3, 0)
Equation of a plane through A (0, – 1, 0) is
a (x – 0) + b (y + 1) + c (z – 0) = 0
or, ax + by + cz + b = 0 ..... (1)
If plane (1) passes through B (2, 1, – 1) and C (1, 1, 1)
Then 2a + 2b – c = 0 ..... (2)
and a + 2b + c = 0 ..... (3)
From (2) and (3), we have
a b c
= =
2 + 2 − 1− 2 4 − 2
a b c
or, = = = k (say)
4 −3 2
Putting the value of a, b, c, in (1), equation of required plane is
4kx – 3k(y + 1) + 2kz = 0
or, 4x – 3y + 2z – 3 = 0 ..... (2)
Clearly point D (3, 3, 0) lies on plane (2)
Thus points D lies on the plane passing through A, B, C and hence points A, B, C and D are coplanar.
Example : If P be any point on the plane x + my + nz = p and Q be a point on the line OP such that
OP . OQ = p2, show that the locus of the point Q is p(x + my + nz) = x 2 + y2 + z2.
Solution Let P ≡ (α, β, γ), Q ≡ (x 1, y1, z1)
Direction ratios of OP are α, β, γ and direction ratios of OQ are x 1, y1, z1.
Since O, Q, P are collinear, we have
α β γ
= =
x1 y1 z1 = k (say) ..... (1)
As P (α, β, γ) lies on the plane x + my + nz = p,
α + mβ + nγ = p or k(x 1 + my1 + nz1) = p ..... (2)
Given, OP . OQ = p2
∴ α 2 + β2 + γ 2 x12 + y12 + z12 = p2
or, k 2 ( x12 + y12 + z12 ) x12 + y12 + z12 = p2
or, k ( x12 + y12 + z12 ) = p2 ..... (3)
On dividing (2) by (3), we get,
x1 + my1 + nz1 1
=
x12 + y12 + z12 p
or, p (x 1 + my1 + nz1) = x12 + y 12 + z12
Hence the locus of point Q is
p (x + my + nz) = x 2 + y2 + z2.
x y z
Example : A point P moves on a plane + + = 1. A plane through P and perpendicular to OP meets the
a b c
co-ordinate axes in A, B and C. If the planes through A, B and C parallel to the planes x = 0, y = 0,
z = 0 intersect in Q, find the locus of Q.
Solution Given plane is
FREE Download Study Package from website: www.TekoClasses.com & www.MathsBySuhag.com

x y z
+ + =1 ..... (1)
a b c
Let P ≡ (h, k, )
h k 

page 10 of 77
Then + + = 1 ..... (2)
a b c
OP = h2 + k 2 +  2
h k 
Direction cosines of OP are , ,
2 2 2 2 2 2
h +k + h +k + h + k 2 + 2 2

∴ Equation of the plane through P and normal to OP is

Teko Classes, Maths : Suhag R. Kariya (S. R. K. Sir), Bhopal Phone : 0 903 903 7779, 0 98930 58881.
h k 
x+ y+ = h2 + k 2 +  2
2 2 2 2 2 2 2 2 2
h +k + h +k + h +k +
or, hx + ky + z = (h2 + k2 + 2)
 h2 + k 2 +  2   h2 + k 2 +  2 
∴ A ≡  , 0, 0  , B ≡  0, , 0 ,

h  k
   
 h2 + k 2 +  2 
C ≡  0, 0, 

  
Let Q ≡ (α, β, γ), then
h2 + k 2 +  2 h2 + k 2 +  2 h2 + k 2 +  2
α= ,β= ,γ= ..... (3)
h k 
1 1 1 h2 + k 2 +  2 1
Now 2
+ 2
+ 2
= 2 2 2 2
= ..... (4)
α β γ (h + k +  ) (h + k 2 +  2 )
2

h2 + k 2 +  2
From (3), h =
α
h h2 + k 2 +  2
∴ =
a aα
k h2 + k 2 +  2  h2 + k 2 +  2
Similarly = and =
b bβ c cγ
h2 + k 2 +  2 h 2 + k 2 +  2 h 2 + k 2 +  2 h k 
∴ + + = + + = 1 [from (2)]
aα bβ cγ a b c
1 1 1 1 1 1 1
or, + + = = + + [from (4)]
a α b β cγ h 2 + k 2 +  2 α 2 β 2 γ 2
∴ Required locus of Q (α, β, γ) is
1 1 1 1 1 1
+ + = + + .
ax by cz x 2 y 2 z 2
Self practice problems :
1. Check wether this point are coplanar if yes find the equation of plane containing them
A ≡ (1, 1, 1)
B ≡ (0, – 1, 0)
C ≡ (2, 1, –1)
D ≡ (3, 3, 0) Ans. yes, 4x – 3y + 2z = 3
2. Find the plane passing through point (– 3, – 3, 1) and perpendicular to the line joining the points
(2, 6, 1) and (1, 3, 0). Ans. x + 3y + z + 11 = 0
3. Find the equation of plane parallel to x + 5y – 4z + 5 = 0 and cutting intercepts on the axes whose rum
3000
is 150. Ans. x + 5y – 4z =
19
4. Find the equation of plane passing through (2, 2, 1) and (9, 3, 6) and perpendicular to the plane
x + 3y + 3z = 8. Ans. 3x + 4y – 5z = 9
5. Find the equation of the plane | | to î + ĵ + k̂ and î − ĵ and passing through (1, 1, 2).
Ans. x + y – 2z + 2 = 0
6. Find the equation of the plane passing through the point (1, 1, – 1) and perpendicular to the planes
x + 2y + 3z – 7 = 0 and 2x – 3y + 4z = 0. Ans. 17x + 2y – 7z = 26
1 2 . Sides of a plane:
A plane div ides the three dim ensional space in two equal parts. Two points A (x 1 y 1 z 1 )
and B (x 2 y2 z2) are on the same side of the plane ax + by + cz + d = 0 if ax 1 + by1 + cz1 + d and
ax 2 + by2 + cz2 + d are both positive or both negative and are opposite side of plane if both of these
values are in opposite sign.
Example : Show that the points (1, 2, 3) and (2, – 1, 4) lie on opposite sides of the plane x + 4y + z – 3 = 0.
Solution Since the numbers 1+ 4 × 2 + 3 – 3 = 9 and 2 – 4 + 4 – 3 = – 1 are of opposite sign., the points are
FREE Download Study Package from website: www.TekoClasses.com & www.MathsBySuhag.com

on opposite sides of the plane.


1 3 . A Plane & A Point
ax'+ by'+ cz'+ d

page 11 of 77
(i) Distance of the point (x ′ , y′ , z′ ) from the plane ax + by + cz+ d = 0 is given by .
a 2 + b2 + c2
  
(ii) The length of the perpendicular from a point having position vector a to plane r . n = d is
 
| a .n − d |
given by p =  .
|n|
(iii) The coordinates of the foot of perpendicular from the point (x, y, z) to the plane

Teko Classes, Maths : Suhag R. Kariya (S. R. K. Sir), Bhopal Phone : 0 903 903 7779, 0 98930 58881.
x'− x1 y'− y1 z'−z1 (ax1 + by1 + cz1 + d)
ax + by + cz + d = 0 aregain by = = =–
a b c a2 + b2 + c 2
(iv) To find image of a point w.r.t. a plane.
Let P (x 1, y1, z1) is a given point and ax + by + cz + d = 0 is given plane Let (x′, y′, z′) is the
image point. then
(a) x′ – x 1 = λa, y′ – y1 = λb, z′ – z1 = λc
⇒ x′ = λa + x 1, y′ = λb + y1, z′ = λc + z1
 x′ + x1   y′ + y1   z′ + z1 
(b) a  + b  + c  =0
 2   2   2 
from (i) put the values of x′, y′, z′ in (ii) and get the values of λ and resubtitute in (i) to get
(x′ y′ z′).
The coordinate of the image of point (x 1 , y1 , z1) w.r.t the plane ax + by + cz + d = 0 are given
x'− x1 y'− y1 z'−z1 (ax1 + by1 + cz1 + d)
by = = =–2
a b c a2 + b2 + c 2
(v) The distance between two parallel planes ax + by + cx + d = 0 and ax + by + cx + d’ = 0 is
| d − d' |
given by
a + b2 + c 2
2

Example : Find the image of the point P (3, 5, 7) in the plane 2x + y + z = 16.
Solution Given plane is 2x + y + z = 16 ..... (1)
P ≡ (3, 5, 7)
Direction ratios of normal to plane (1) are 2, 1, 1
Let Q be the image of point P in plane (1). Let PQ meet plane (1) in R
then PQ ⊥ plane (1)
Let R ≡ (2r + 3, r + 5, r + 7)
Since R lies on plane (1)
∴ 2(2r + 3) + r + 5 + r + 7 = 0 or, 6r + 18 = 0 ∴ r=–3
∴ R ≡ (– 3, 2, 4)
Let Q ≡ (α, β, γ)
Since R is the middle point of PQ
α+3
∴ –3= ⇒ α=–9
2
β+5
2= ⇒ β=–1
2
γ+7
4= ⇒ γ=1 ∴ Q = (– 9, – 1, 1).
2
Example : Find the distance between the planes 2x – y + 2z = 4 and 6x – 3y + 6z = 2.
Solution Given planes are
2x – y + 2z – 4 = 0 ..... (1)
and 6x – 3y + 6z – 2 = 0 ..... (2)
a1 b1 c1
We find that = = Hence planes (1) and (2) are parallel.
a2 b2 c 2
2
Plane (2) may be written as 2x – y + 2z – =0 ..... (3)
3
∴ Required distance between the planes
2
4−
3 10 10
= = =
2 2
2 + ( −1) + 2 2 3.3 9
Example : A plane passes through a fixed point (a, b, c). Show that the locus of the foot of perpendicular
to it from the origin is the sphere x 2 + y2 + z2 – ax – by – cz = 0
Solution Let the equation of the variable plane be
O(0, 0, 0)
FREE Download Study Package from website: www.TekoClasses.com & www.MathsBySuhag.com

P(α, β, γ)

page 12 of 77
x + my + nz + d = 0 ..... (1)
Plane passes through the fixed point (a, b, c)
∴ a + mb + nc + d = 0 ..... (2)
Let P (α, β , γ) be the foot of perpendicular from origin to plane (1).
Direction ratios of OP are
α – 0, β – 0, γ – 0 i.e. α, β , γ
From equation (1), it is clear that the direction ratios of normal to the plane i.e. OP are , m, n ;
α, β , γ and , m, n are the direction ratios of the same line OP

Teko Classes, Maths : Suhag R. Kariya (S. R. K. Sir), Bhopal Phone : 0 903 903 7779, 0 98930 58881.
α β γ 1
∴ = = = (say)
 m n k
∴  = k α, m = k β , n = k γ ..... (3)
Putting the values of
, m, n in equation (2), we get
kaα + kbβ + kcγ + d = 0 ..... (4)
Since α, β , γ lies in plane (1)
∴ α + m β + nγ + d = 0 ..... (5)
Putting the values of , m, n from (3) in (5), we get
k α2 + k β 2 + k γ2 + d = 0 ..... (6)
or kα2 + kβ 2 + kγ2 – kaα – kbβ – kcγ = 0
[putting the value of d from (4) in (6)]
or α2 + β 2 + γ2 – a α – b β – c γ = 0
Therefore, locus of foot of perpendicular P (α, β , γ) is
x 2 + y2 + z2 – ax – by – cz = 0 ..... (7)
Self practice problems:
1. Find the intercepts of the plane 3x + 4y – 7z = 84 on the axes. Also find the length of perpendicular
from origin to this line and direction cosines of this normal.
1 3 4 −7
Ans. a = 28, b = 21, c = – 12, p = ; , ,
74 74 74 74
2. Find : (i) perpendicular distance
(ii) foot of perpendicular
(iii) image of (1, 0, 2) in the plane 2x + y + z = 5
1  4 1 13  5 1 7
Ans. (i) (ii)  , ,  (iii)  , , 
6 3 6 6  3 3 3
1 4 . Angle Between Two Planes:
(i) Consider two planes ax + by + cz + d = 0 and a′ x + b′ y + c′ z + d′ = 0. Angle between these
planes is the angle between their normals. Since direction ratios of their normals are (a, b, c)
and (a′ , b′ , c′ ) respectively, hence θ, the angle between them, is given by
aa'+ bb'+ cc'
cos θ =
a 2 + b2 + c2 a '2 + b'2 + c'2
a b c
Planes are perpendicular if aa′ + bb′ + cc′ = 0 and planes are parallel if = =
a' b' c'
 
    n1 . n 2
(ii) The angle θ between the planes r . n = d1 and r . n 2 = d2 is given by, cos θ =  
| n1 | . | n 2 |
   
Planes are perpendicular if n1 . n 2 = 0 & planes are parallel if n1 = λ n 2 .
1 5 . Angle Bisectors
(i) The equations of the planes bisecting the angle between two given planes
a1x + b1y + c1z + d1 = 0 and a2x + b2y + c2z + d2 = 0 are
a1x + b1y + c1z + d1 a 2 x + b 2 y + c2 z + d 2

a12 + b12 + c12 a 22 + b22 + c22
(ii) Equation of bisector of the angle containing origin: First make both the constant terms positive.
a1x + b1y + c1z + d1 a x + b2 y + c2 z + d 2 gives the bisector of
Then the positive sign in =± 2
a12 + b12 + c12 a 22 + b22 + c22
the angle which contains the origin.
(iii) Bisector of acute/obtuse angle: First make both the constant terms positive. Then
a 1a 2 + b 1b 2 + c 1c 2 > 0 ⇒ origin lies on obtuse angle
a 1a 2 + b 1b 2 + c 1c 2 < 0 ⇒ origin lies in acute angle
1 6 . Family of Planes
(i) Any plane passing through the line of intersection of non−parallel planes or equation of
FREE Download Study Package from website: www.TekoClasses.com & www.MathsBySuhag.com

the plane through the given line in serval form.


a 1x + b 1y + c 1z + d 1 = 0 & a2x + b2y + c2z + d2 = 0 is
a1x + b1y + c1z + d1 + λ (a2x + b2y + c2z + d2) = 0
 

page 13 of 77
(ii) The equation of plane passing through the intersection of the planes r . n1 = d 1 &
   
r . n 2 = d2 is r . (n1 + λ n 2 ) = d1 + λ d2 where λ is arbitrary scalar
Example : The plane x – y – z = 4 is rotated through 90° about its line of intersection with the plane
x + y + 2z = 4. Find its equation in the new position.
Solution Given planes are
x–y–z=4 ..... (1)
and x + y + 2z = 4 ..... (2)

Teko Classes, Maths : Suhag R. Kariya (S. R. K. Sir), Bhopal Phone : 0 903 903 7779, 0 98930 58881.
Since the required plane passes through the line of intersection of planes (1) and (2)
∴ its equation may be taken as
x + y + 2z – 4 + k (x – y – z – 4) = 0
or (1 + k)x + (1 – k)y + (2 – k)z – 4 – 4k = 0 ..... (3)
Since planes (1) and (3) are mutually perpendicular,
∴ (1 + k) – (1 – k) – (2 – k) = 0
2
or, 1+k–1+k–2+k=0 or, k=
3
2
Putting k = in equation (3), we get,
3
5x + y + 4z = 20
This is the equation of the required plane.
Example : Find the equation of the plane through the point (1, 1, 1) which passes through the line of
intersection of the planes x + y + z = 6 and 2x + 3y + 4z + 5 = 0.
Solution Given planes are
x+y+z–6=0 ..... (1)
and 2x + 3y + 4z + 5 = 0 ..... (2)
Given point is P (1, 1, 1).
Equation of any plane through the line of intersection of planes (1) and (2) is
x + y + z – 6 + k (2x + 3y + 4z + 5) = 0 ..... (3)
If plane (3) passes through point P, then
3
1 + 1 + 1 – 6 + k (2 + 3 + 4 + 5) = 0 or, k=
14
From (1) required plane is
20x + 23y + 26z – 69 = 0
Example : Find the planes bisecting the angles between planes
2x + y + 2z = 9 and 3x – 4y + 12z + 13 = 0.
Which of these bisector planes bisects the acute angle between the given planes. Does origin lie in the
acute angle or obtuse angle between the given planes ?
Solution Given planes are
– 2x – y – 2z + 9 = 0 ..... (1)
and 3x – 4y + 12z + 13 = 0 ..... (2)
Equations of bisecting planes are
−2 x − y − 2 z + 9 3 x − 4 y + 12z + 13

2 2 2
( −2) + ( −1) + ( −2) 3 2 + ( −4)2 + (12)2
or, 13 [– 2x – y – 2z + 9] = ± 3 (3x – 4y + 12z + 13)
or, 35x + y + 62z = 78, ..... (3) [Taking +ve sign]
and 17x + 25y – 10z = 156 ..... (4) [Taking – ve sign]
Now a1a2 + b1b2 + c1c2 = (– 2) (3) + (– 1) (– 4) + (– 2) (12)
= – 6 + 4 – 24 = – 26 < 0
∴ Bisector of acute angle is given by 35x + y + 62z = 78
∵ a1a2 + b1b2 + c1c2 < 0, origin lies in the acute angle between the planes.
Example : If the planes x – cy – bz = 0, cx – y + az = 0 and bx + ay – z = 0 pass through a straight line,
then find the value of a2 + b2 + c2 + 2abc.
Solution Given planes are
x – cy – bz = 0 ..... (1)
cx – y + az = 0 ..... (2)
bx + ay – z = 0 ..... (3)
Equation of any plane passing through the line of intersection of planes (1) and (2) may be taken as
x – cy – bz + λ (cx – y + az) = 0
or, x (1 + λc) – y (c + λ) + z (– b + aλ) = 0 ..... (4)
If planes (3) and (4) are the same, then equations (3) and (4) will be identical.
1 + cλ −(c + λ ) −b + aλ
∴ = =
b a −1
(i) (ii) (iii)
From (i) and (ii), a + acλ = – bc – bλ
(a + bc )
or, λ=– ..... (5)
(ac + b)
From (ii) and (iii),
FREE Download Study Package from website: www.TekoClasses.com & www.MathsBySuhag.com

(ab + c )
c + λ = – ab + a2λ or λ= ..... (6)
1− a2
From (5) and (6), we have,

page 14 of 77
−(a + bc ) −(ab + c )
= .
ac + b (1 − a 2 )
or, a – a + bc – a bc = a2bc + ac2 + ab2 + bc
3 2

or, a2bc + ac2 + ab2 + a3 + a2bc – a = 0


or, a2 + b2 + c2 + 2abc = 1.
Self practice problems:
1. A tetrahedron has vertices at O(0, 0, 0), A(1, 2, 1), B(2, 1, 3) and C(–1, 1, 2). Prove that the angle

Teko Classes, Maths : Suhag R. Kariya (S. R. K. Sir), Bhopal Phone : 0 903 903 7779, 0 98930 58881.
 19 
between the faces OAB and ABC will be cos–1   .
 35 
2. Find the equation of plane passing through the line of intersection of the planes 4x – 5y – 4z = 1 and
10
2x + y + 2z = 8 and the point (2, 1, 3). Ans. 32x – 5y + 8z – 83 = 0, λ =
3
3. Find the equations of the planes bisecting the angles between the planes
x + 2y + 2z – 3 = 0, 3x + 4y + 12z + 1 = 0 and sepecify the plane which bisects the acute angle
between them. Ans. 2x + 7y – 5z = 21, 11x + 19y + 31z = 18; 2x + 7y – 5z = 21
4. Show that the origin lies in the acute angle between the planes
x + 2y + 2z – 9 = 0 and 4x – 3y + 12z + 13 = 0
5. Prove that the planes 12x – 15y + 16z – 28 = 0, 6x + 6y – 7z – 8 = 0 and 2x + 35y – 39z + 12 = 0 have
a common line of intersection.
17. Area of a triangle:
Let A (x 1, y1, z1), B (x 2, y2, z2), C (x 3, y3, z3) be the vertices of a triangle, then ∆ = (∆2x + ∆2y + ∆2z )

y1 z1 1 z1 x1 1 x1 y1 1
1 1
where ∆ x = y2 z2 1 , ∆ y = z x 2 1 and ∆z = x 2 y2 1
2 2 2
y3 z3 1 z3 x3 1 x3 y3 1
→ →
Vector Method − From two vector AB and AC . Then area is given by
i j k
1 → → 1
| AB x AC | = x 2 − x1 y 2 − y1 z2 − z1
2 2
x 3 − x1 y 3 − y1 z3 − z1
Example : Through a point P (h, k, ) a plane is drawn at right angles to OP to meet the co-ordinate axes in A,
p5
B and C. If OP = p, show that the area of ∆ ABC is .
2hk 
Solution OP = h 2 + k 2 +  2 = p
Direction cosines of OP are
h k 
, ,
h2 + k 2 +  2 h2 + k 2 +  2 h2 + k 2 +  2
Since OP is normal to the plane, therefore, equation of the plane will be,
h k 
x+ y+ z = h2 + k 2 +  2
2 2 2 2 2 2 2 2 2
h +k + h +k + h +k +
or, hx + ky + z = h2 + k2 + 2 = p2 ..... (1)
 p2   p2   2 
 , 0, 0   0, , 0   0, 0, p 
∴ A≡  h , B ≡  k  ,C≡   
    
Now area of ∆ ABC, ∆ = A2xy + A2yz + A2zx
Now Axy = area of projection of ∆ ABC on xy-plane = area of ∆ AOB
p2
0 1
h
1 p2 1 p4
= Mod of 0 1 =
2 k 2 | hk |
0 0 1
1 p4 1 p4
Similarly, Ayz = and Azx =
2 | k | 2 | h |
FREE Download Study Package from website: www.TekoClasses.com & www.MathsBySuhag.com

1 p8 1 p8 1 p8
∴ ∆2 = + +
4 h 2k 2 4 k 2  2 4  2 h 2
p8 p10 p5

page 15 of 77
= (  2 + k 2 + h2) = or, ∆= .
4h 2k 2  2 4h 2k 2  2 2hk
1 8 . Volume Of A Tetrahedron:
Tetrahedron
Volume of a tetrahedron with vertices A (x 1, y1, z1), B( x 2, y2, z2), C (x 3, y3, z3) and
x1 y1 z1 1
x2 y2 z2 1
1
D (x 4, y4, z4) is given by V =

Teko Classes, Maths : Suhag R. Kariya (S. R. K. Sir), Bhopal Phone : 0 903 903 7779, 0 98930 58881.
6 x3 y3 z3 1
x4 y4 z4 1
A LINE
1 9 . Equation Of A Line
(i) A straight line in space is characterised by the intersection of two planes which are not parallel and
therefore, the equation of a straight line is a solution of the system constituted by the equations of the
two planes, a1x + b1y + c1z + d1 = 0 and a2x + b2y + +c2z + d2 =0. This form is also known as non−
symmetrical form.
x − x1
(ii) The equation of a line passing through the point (x 1, y1, z1) and having direction ratios a, b, c is
a
y − y1 z − z1
= = = r. This form is called symmetric form. A general point on the line is given by (x 1 +
b c
ar, y1 + br, z1 + cr).
(iii) Vector equation: Vector equation of a straight line passing through a fixed point with position
    
vector a and parallel to a given vector b is r = a + λ b where λ is a scalar..
(iv) The equation of the line passing through the points (x 1, y1, z1) and (x 2, y2, z2) is
x − x1 y − y1 z − z1
= =
x 2 − x1 y 2 − y1 z 2 − z1
 
(v) Vector equation of a straight line passing through two points with position vectors a & b
   
is r = a + λ ( b − a ).
(vi) Reduction of cartesion form of equation of a line to vector form & vice versa
x − x1 y − y1 z − z1  = (x + y + z ) + λ (a + b + c ).
= = ⇔ r 1 î 1 ĵ 1 k̂ î ĵ k̂
a b c
Note: Straight lines parallel to co-ordinate axes:
Straight lines Equation Straight lines Equation
(i) Through origin y = mx, z = nx (v) Parallel to x −axis y = p, z = q
(ii) x −axis y = 0, z = 0 (vi) Parallel to y−axis x = h, z = q
(iii) y−axis x = 0, z = 0 (vii) Parallel to z−axis x = h, y = p
(iv) z−axis x = 0, y = 0
Example : Find the equation of the line through the points (3, 4, –7) and (1, – 1, 6) in vector form as well as in
cartesian form.
Solution Let A ≡ (3, 4, – 7), B ≡ (1, – 1, 6)
→ → → →

Now a = OA = 3 i + 4 j – 7 k ,
→ → → → →
= b = OB = i – j + 6 k
→ → → → → →
Equation of the line through A( a ) and B( b ) is r = a +t (b – a )
→ → → → → → →
or r = 3 i + 4 j – 7 k + t (–2 i – 5 j + 13 k ) ..... (1)
Equation in cartesian form :
x−3 y−4 z+7 x−3 y−4 z+7
Equation of AB is = = or, = =
3 −1 4 +1 − 7 − 6 2 5 − 13
x −1 y + 2 z − 3
Example : Find the co-ordinates of those points on the line = = which is at a distance of
2 3 6
3 units from point (1, –2, 3).
x −1 y + 2 z − 3
Solution Given line is = = ..... (1)
2 3 6
Let P ≡ (1, –2, 3) Direction ratios of line (1) are 2, 3, 6
FREE Download Study Package from website: www.TekoClasses.com & www.MathsBySuhag.com

2 3 6
∴ Direction cosines of line (1) are , ,
7 7 7
Equation of line (1) may be written as

page 16 of 77
x −1 y + 2 z − 3
= = ..... (2)
2 3 6
7 7 7
Co-ordinates of any point on line (2) may be taken as
2 3 6 
 r + 1, r − 2, r + 3 
7 7 7 

Teko Classes, Maths : Suhag R. Kariya (S. R. K. Sir), Bhopal Phone : 0 903 903 7779, 0 98930 58881.
2 3 6 
Let Q ≡  r − 1, r − 2, r + 3 
7 7 7 
Distance of Q from P = | r |
According to question | r | = 3 ∴ r=±3 Putting the value of r, we have
 1 5 39   13 23 3 
Q ≡ − , − ,  or Q ≡ − , − , 
 7 7 7   7 7 7
Example : Find the equation of the line drawn through point (1, 0, 2) to meet at right angles the line
x +1 y − 2 z +1
= =
3 −2 −1
Solution Given line is
x +1 y − 2 z +1
= = ..... (1)
3 −2 −1
Let P ≡ (1, 0, 2)
Co-ordinates of any point on line (1) may be taken as
Q ≡ (3r – 1, – 2r + 2, – r – 1)
Direction ratios of PQ are 3r – 2, – 2r + 2, – r – 3
Direction ratios of line AB are 3, – 2, – 1
Since PQ ⊥ AB
∴ 3 (3r – 2) – 2 (– 2r + 2) – 1 (– r – 3) = 0
1
⇒ 9r – 6 + 4r – 4 + r + 3 = 0 ⇒ 14r = 7 ⇒ r=
2
Therefore, direction ratios of PQ are
1 7
– , 1, – or, – 1, 2, – 7
2 2
Equation of line PQ is
x −1 y = 0 z − 2 x −1 y z−2
= = or, = =
−1 2 −7 1 −2 7
x −1 y − 2 z − 3 x − 4 y −1
Example : Show that the two lines = = and = = z intersect. Find also the
2 3 4 5 2
point of intersection of these lines.
x −1 y − 2 z − 3
Solution Given lines are = = ..... (1)
2 3 4
x − 4 y −1 z − 0
and = = ..... (2)
5 2 1
Any point on line (1) is P (2r + 1, 3r + 2, 4r +3)
and any point on line (2) is Q (5λ + 4, 2λ + 1, λ)
Lines (1) and (2) will intersect if P and Q coincide for some value of λ and r.
∴ 2r + 1 = 5λ + 4 ⇒ 2r – 5λ = 3 ..... (1)
3r + 2 + 2λ + 1 ⇒ 3r – 2λ = – 1 ..... (2)
4r + 3 = λ ⇒ 4r – λ = – 3 ..... (3)
Solving (1) and (2), we get r = – 1, λ = – 1
Clearly these values of r and λ satisfy eqn. (3)
Now P ≡ (– 1, – 1, – 1) Hence lines (1) and (2) intersect at (– 1, – 1, – 1).
Self practice problems:
1. Find the equation of the line passing through point (1, 0, 2) having direction ratio 3, – 1, 5. Prove that
x −1 y z−2
this line passes through (4, – 1, 7). Ans. − =
3 −1 5
x − 2 y +1 z − 7
2. Find the equation of the line parallel to line = = and passing through the point (3, 0, 5).
3 1 9
x−3 y z−5
Ans. = =
3 1 3
3. Find the coordinates of the point when the line through (3, 4, 1) and (5, 1, 6) crosses the xy plane.
FREE Download Study Package from website: www.TekoClasses.com & www.MathsBySuhag.com

 13 23 
Ans.  , , 0
 5 5 
2 0 . Reduction Of Non-Symmetrical Form To Symmetrical Form:

page 17 of 77
Let equation of the line in non−symmetrical form be a1x + b1y + c1z + d1 = 0, a2x + b2y + c2z + d2 = 0.
To find the equation of the line in symmetrical form, we must know (i) its direction ratios (ii) coordinate
of any point on it.
(i) Direction ratios: Let , m, n be the direction ratios of the line. Since the line lies in both the
planes, it must be perpendicular to normals of both planes. So a1  + b1m + c1n = 0,
a2 + b2m + c2n = 0. From these equations, proportional values of , m, n can be found by
 m n

Teko Classes, Maths : Suhag R. Kariya (S. R. K. Sir), Bhopal Phone : 0 903 903 7779, 0 98930 58881.
cross−multiplication as = =
b1c2 − b2 c1 c1a 2 − c2a1 a1b2 − a 2 b1
i j k
Alternative method The vector a1 b1 c1 = i (b1c2 – b2c1) + j (c1a2 – c2a1) + k (a1b2 – a2b1) will be parallel
a2 b2 c 2
to the line of intersection of the two given planes. hence  : m: n = (b1c2 – b2c1): (c1a2 – c2a1):
(a1b2 – a2b1)
(ii) Point on the line − Note that as , m, n cannot be zero simultaneously, so at least one must
be non−zero. Let a1b2 − a2b1 ≠ 0, then the line cannot be parallel to xy plane, so it intersect it.
Let it intersect xy−plane in (x 1, y1, 0). Then a1x 1 + b1y1 + d1 = 0 and a2x 1 + b2y1 + d2 = 0.Solving
these,we get a point on the line. Then its equation becomes.
b1d 2 − b 2d1 d a − d 2a1
x− y− 1 2
x − x1 y − y1 z−0 a1b 2 − a 2 b1 a1b2 − a 2 b1 z−0
= = or = =
b1c2 − b2 c1 c1a 2 − c2a1 a1b2 − a 2 b1 b1c2 − b 2 c1 c1a 2 − c2a1 a1b2 − a 2 b1
Note: If  ≠ 0, take a point on yz−plane as (0, y1, z1) and if m ≠ 0, take a point on xz−plane as (x 1, 0, z1).
Alternative method
a1 b1
If a ≠ b Put z = 0 in both the equations and solve the equations a1x + b1y + d1 = 0, a2x + b2y + d2 =0
2 2
otherwise Put y = 0 and solve the equations a 1x + c1z + d1 = 0 and a2x + c2z + d2 = 0
Example : Find the equation of the line of intersection of planes 4x + 4y – 5z = 12, 8x + 12y – 13z = 32 in
the symmetric form.
Solution Given planes are 4x + 4y – 5z – 12 = 0 ..... (1)
and 8x + 12y – 13z – 32 = 0 ..... (2)
Let , m, n be the direction ratios of the line of intersection :
then 4 + 4m – 5n = 0 ..... (3)
and 8 + 12m – 13n = 0
 m n  m n  m n
∴ = = or, = = or, = =
− 52 + 60 − 40 + 52 48 − 32 8 12 16 2 3 4
Hence direction ratios of line of intersection are 2, 3, 4.
Here 4 ≠ 0, therefore line of intersection is not parallel to xy-plane.
Let the line of intersection meet the xy-plane at P (α, β , 0).
Then P lies on planes (1) and (2)
∴ 4α + 4β + 12 = 0
or, α+β–3=0 ..... (5)
and 8α + 12β – 32 = 0
or, 2α + 3β – 8 = 0 ..... (6)
Solving (5) and (6), we get
α β 1 α β 1
= = or, = =
−8+9 −6+8 3−2 1 2 1
∴ α = 1, β = 2
x −1 y − 2 z − 0
Hence equation of line of intersection in symmetrical form is = = .
2 3 4
Example : Find the angle between the lines x – 3y – 4 = 0, 4y – z + 5 = 0 and x + 3y – 11 =0, 2y – z + 6 = 0.
Solution Given lines are
x − 3 y − 4 = 0
 ..... (1)
4 y − z + 5 = 0
x + 3 y − 11 = 0
and  ..... (2)
2y − z + 6 = 0 
Let 1, m 1, n1 and 1, m 2, n2 be the direction cosines of lines (1) and (2) respectively
∵ line (1) is perpendicular to the normals of each of the planes
x – 3y – 4 = 0 and 4y – z + 5 = 0
∴ 1 – 3m 1 + 0.n1 = 0 ..... (3)
and 01 + 4m 1 – n1 = 0 ..... (4)
FREE Download Study Package from website: www.TekoClasses.com & www.MathsBySuhag.com

Solving equations (3) and (4), we get,


1 m1 n  1 m1 n1
= = 1 or, = = = k (let).
3−0 0 − ( −1) 4 − 0 3 1 4

page 18 of 77
Since line (2) is perpendicular to the normals of each of the planes
x + 3y – 11 = 0 and 2y – z + 6 = 0,
∴ 2 + 3m 2 = 0 ..... (5)
and 2m 2 – n2 = 0 ..... (6)
2
∴ 2 = – 3m 2 or, = m2
−3
n2

Teko Classes, Maths : Suhag R. Kariya (S. R. K. Sir), Bhopal Phone : 0 903 903 7779, 0 98930 58881.
and n2 = 2m 2 or, = m 2.
2
2 m n
∴ = 3 = 2 = t (let).
−3 1 2
If θ be the angle between lines (1) and (2), then
cosθ = 12 + m 1m 2 + n1n2
= (3k) (– 3t) + (k) (t) + (4k) (2t)
= – 9kt + kt + 8kt = 0 ∴ θ = 90°.
Self practice problems:
1. Find the equation of the line of intersection of the plane
4x + 4y – 5z = 12
x −1 y−2 z−0
8x + 12y – 13z = 32 Ans. = =
2 3 4
2. Show that the angle between the two lines defined by the equations x = y and xy + yz + zx = 0 is
 1
cos–1  
3
3. Prove that the three planes 2x + y – 4z – 17 = 0, 3x + 2y – 2z – 25 = 0, 2x – 4y + 3z + 25 = 0 intersect
at a point and find its co-ordinates. Ans. (3, 7, – 1)
2 1 . Foot, Length And Equation Of Perpendicular From A Point To A Line:
x−a y − b z−c
(i)Cartesian form: Let equation of the line be = = = r (say) ..........(i)
 m n
and A (α, β, γ ) be the point.
Any point on line (i) is P (r + a, mr + b, nr + c) ......... (ii)
If it is the foot of the perpendicular from A on the line, then AP is perpendicular to the line. So  (r + a − α) +
m (mr + b − β ) + n (nr + c − γ ) = 0 i.e. r = (α − a)  + (β − b) m + (γ − c)n since 2 + m 2 + n2 = 1. Putting this
value of r in (ii), we get the foot of perpendicular from point A on the given line. Since foot of perpendicular P is
2 2 2
known, then the length of perpendicular is given by AP = (r + a − α ) + ( mr + b − β) + ( nr + c − γ ) the
x−α y −β z−γ
equation of perpendicular is given by = = (ii) Vector Form: Equation of a line
r + a − α mr + b − β nr + c − γ
      
passing through a point having position vector α and perpendicular to the lines r = a1 + λ b1 and r = a 2 + λ b2
      
is parallel to b1 x b2 . So the vector equation of such a line is r = α + λ ( b1 x b 2 ). Position vector β of the
  
      2 (a − α) . b   
  2
image of a point α in a straight line r = a + λ b is given by β = 2 a −   b − α . Position vector of
 |b| 
  
   (a −α) . b  
the foot of the perpendicular on line is f = a −  2  b . The equation of the perpendicular is r = α + µ
 | b | 
    (a − α   
) .b 
(a − α) −   2 

b .
  |b|  
22. To find image of a point w. r. t a line
x − x2 y − y2 z − z2
Let L ≡ = = is a given line
a b c
Let (x′, y′, z′) is the image of the point P (x 1, y1, z1) with respect to the line L. Then
(i) a (x 1 – x′) + b (y1 – y′) + c (z1 – z′) = 0
x1 + x′ y 1 + y′ z1 + z′
− x2 − y2 − z2
(ii) 2 = 2 = 2 =λ
a b c
FREE Download Study Package from website: www.TekoClasses.com & www.MathsBySuhag.com

from (ii) get the value of x′, y′, z′ in terms of λ as


x′ = 2aλ + 2x 2 – x 1, y′ = 2bα – 2y2 – y1,
z′ = 2cλ + 2z2 – z1
now put the values of x′, y′, z′ in (i) get λ and resubtitute the value of λ to to get (x′ y′ z′).

page 19 of 77
x +1 y − 3 z + 2
Example : Find the length of the perpendicular from P (2, – 3, 1) to the line = = .
2 3 −1
x +1 y − 3 z + 2
Solution Given line is = = ..... (1)
2 3 −1
P ≡ (2, – 3, 1)
Co-ordinates of any point on line (1) may be taken as

Teko Classes, Maths : Suhag R. Kariya (S. R. K. Sir), Bhopal Phone : 0 903 903 7779, 0 98930 58881.
Q ≡ (2r – 1, 3r + 3, – r – 2)
Direction ratios of PQ are 2r – 3, 3r + 6, – r – 3
Direction ratios of AB are 2, 3, – 1
Since PQ ⊥ AB
∴ 2 (2r – 3) + 3 (3r + 6) – 1 (– r – 3) = 0
−15
or, 14r + 15 = 0 ∴ r=
14
 − 22 − 3 − 13  531
∴ Q≡  , ,  ∴ PQ = units.
 7 14 14  14
Second method : Given line is
x +1 y − 3 z + 2
= =
2 3 −1 P (2, –3, 1)
P ≡ (2, – 3, 1)
2 3 1
Direction ratios of line (1) are , ,–
14 14 14
RQ = length of projection of RP on AB A R Q B
2 3 1 15 (–1, 3, –2)
= (2 + 1) + ( −3 − 3) − (1 + 2) =
14 14 4 14
PR2 = 32 + 62 + 32 = 54
225 531
∴ PQ =PR 2 − RQ 2 = 54 − =
14 14
Self practice problems:
x − 11 y + 2 z + 8
1. Find the length and foot of perpendicular drawn from point (2, –1, 5) to the line = = .
10 −4 − 11
Also find the image of the point in the line. Ans. 14 , N ≡ (1, 2, 3), Ι ≡ (0, 5, 1)
x y −1 z − 2
2. Find the image of the point (1, 6, 3) in the line = = . Ans. (1, 0 , 7)
1 2 3
x − 15 y − 29 z − 5
3. Find the foot and hence the length of perpendicular from (5, 7, 3) to the line = = .
3 8 −5
x−5 y−7 z−3
Find also the equation of the perpendicular. Ans. (9, 13, 15) ; 14 ; = =
2 3 6
2 3 . Angle Between A Plane And A Line:
x − x1 y − y1 z − z1
(i) If θ is the angle between line = = and the plane ax + by + cz + d = 0, then
 m n
 
a  + bm + cn
sin θ =  .
 (a 2 + b2 + c2 ) 2 2 2 
 +m + n 

 
   b. n 
(ii) Vector form: If θ is the angle between a line r = ( a + λ b ) and r . n = d then sin θ =     .
| b | | n |
 m n  
(iii) Condition for perpendicularity = = b xn = 0
a b c
 
(iv) Condition for parallel a + bm + cn = 0 b.n = 0
24. Condition For A Line To Lie In A Plane
x − x1 y − y1 z − z1
(i) Cartesian form: Line = = would lie in a plane
FREE Download Study Package from website: www.TekoClasses.com & www.MathsBySuhag.com

 m n
ax + by + cz + d = 0, if ax 1 + by1 + cz1 + d = 0 & a + bm + cn = 0.
        
(ii) Vector form: Line r = a + λ b would lie in the plane r . n = d if b . n = 0 & a . n = d

page 20 of 77
2 5 . Coplanar Lines:
x −α y −β z − γ x − α' y − β' z − γ '
(i) If the given lines are = = and = = , then condition
 m n ' m' n'
α − α ' β − β' γ − γ '
for intersection/coplanarity is  m n = 0 & plane containing the above

Teko Classes, Maths : Suhag R. Kariya (S. R. K. Sir), Bhopal Phone : 0 903 903 7779, 0 98930 58881.
' m' n'
x −α y −β z− γ
two lines is  m n =0
' m' n'
(ii) Condition of coplanarity if both the lines are in general form Let the lines be
ax + by + cz + d = 0 = a′x + b′ y + c′ z + d′ &
αx + β y + γ z + δ = 0 = α′x + β′ y + γ′ z + δ′
a b c d
a' b' c' d'
They are coplanar if =0
α β γ δ
α' β' γ' δ'
i j k
 m n
Alternative method: get vector along the line of shortest distance as
′ m′ n′
Now get unit vector along this vector
û = i + mj + nk

Let v = (α – α′ ) î + (B – B′) ĵ + (y – y′)
S. D. = u. v
Example : Find the distance of the point (1, 0, – 3) from the plane x – y – z = 9 measured parallel to the
x−2 y+2 z−6
line = = .
2 3 −6
Solution Given plane is x – y – z = 9 ..... (1)
x−2 y+2 z−6
Given line AB is = = ..... (2)
2 3 −6
Equation of a line passing through the point Q(1, 0, – 3) and parallel to line (2) is
x −1 y z + 3
= = = r.. ..... (3)
2 3 −6
Co-ordinates of point on line (3) may be taken as
P (2r + 1, 3r, – 6r – 3)
If P is the point of intersection of line (3) and plane (1), then P lies on plane (1),
∴ (2r + 1) – (3r) – (– 6r – 3) = 9
r=1
or, P ≡ (3, 3, – 9)
Distance between points Q (1, 0, – 3) and P (3, 3, – 9)
PQ = (3 − 1)2 + (3 − 0)2 + ( −9 − (3))2 = 4 + 9 + 36 = 7.
B

Q (1, 0, – 3)

P
x + 3 y −1 z − 2
Example: Find the equation of the plane passing through (1, 2, 0) which contains the line = = .
3 4 −2
Solution Equation of any plane passing through (1, 2, 0) may be taken as
a (x – 1) + b (y – 2) + c (z – 0) = 0 ..... (1)
where a, b, c are the direction ratios of the normal to the plane. Given line is
FREE Download Study Package from website: www.TekoClasses.com & www.MathsBySuhag.com

x + 3 y −1 z − 2
= = ..... (2)
3 4 −2
If plane (1) contains the given line, then
3a + 4b – 2c = 0 ..... (3)

page 21 of 77
Also point (– 3, 1, 2) on line (2) lies in plane (1)
∴ a (– 3 – 1) + b (1 – 2) + c (2 – 0) = 0
or, – 4a – b + 2c = 0 ..... (4)
Solving equations (3) and (4), we get,
a b c
= =
8 − 2 8 − 6 − 3 + 16
a b c

Teko Classes, Maths : Suhag R. Kariya (S. R. K. Sir), Bhopal Phone : 0 903 903 7779, 0 98930 58881.
or, = = = k (say). ..... (5)
6 2 13
Substituting the values of a, b and c in equation (1), we get,
6 (x – 1) + 2 (y – 2) + 13 (z – 0) = 0.
or, 6x + 2y + 13z – 10 = 0. This is the required equation.
x −1 y +1 z − 3
Example : Find the equation of the projection of the line = = on the plane x + 2y + z = 9.
2 −1 4 B
Solution A
Let the given line AB be
x −1 y +1 z − 3
= = ..... (1)
2 −1 4
Given plane is
x + 2y + z = 9 ..... (2) D C
Let DC be the projection of AB on plane (2)
Clearly plane ABCD is perpendicular to plane (2).
Equation of any plane through AB may be taken as (this plane passes through the point (1, – 1, 3) on
line AB)
a (x – 1) + b (y + 1) + c (z – 3) = 0 ..... (3)
where 2a – b + 4c = 0 ..... (4)
[∵ normal to plane (3) is perpendicular to line (1)]
Since plane (3) is perpendicular to plane (2),
∴ a + 2b + c = 0 ..... (5)
Solving equations (4) & (5), we get,
a b c
= = .
−9 2 5
Substituting these values of a, b and c in equation (3), we get
9 (x – 1) – 2 (y + 1) – 5 (z – 3) = 0
or, 9x – 2y – 5z + 4 = 0 ...... (6)
Since projection DC of AB on plane (2) is the line of intersection of plane ABCD and plane (2), therefore
equation of DC will be
9 x − 2y − 5z + 4 = 0 .....(i) 
and  ..... (7)
x + 2y + z − 9 = 0 .....(ii)
Let , m, n be the direction ratios of the line of intersection of planes (i) and (ii)
∴ 9 – 2m – 5n = 0 ..... (8)
and  + 2m + n = 0 ..... (9)
 m n
∴ = =
− 2 + 10 − 5 − 9 18 + 2
x − 3 y +1 z + 2 x−7 y z+2
Example : Show that the lines = = and = = are coplanar. Also find the
2 −3 1 −3 1 2
equation of the plane containing them.
Solution Given lines are
x − 3 y +1 z + 2
= = = r (say) ..... (1)
2 −3 1
x−7 y z+7
and = = = R (say) ..... (2)
−3 1 2
If possible, let lines (1) and (2) intersect at P.
Any point on line (1) may be taken as
(2r + 3, – 3r – 1, r – 2) = P (let).
Any point on line (2) may be taken as
(– 3R + 7, R, 2R – 7) = P (let).
∴ 2r + 3 = – 3R + 7
or, 2r + 3R = 4 ..... (3)
Also – 3r – 1 = R
or, – 3r – R = 1 ..... (4)
and r – 2 = 2R – 7
or, r – 2R = – 5. ..... (5)
Solving equations (3) and (4), we get,
r = – 1, R = 2
FREE Download Study Package from website: www.TekoClasses.com & www.MathsBySuhag.com

Clearly r = – 1, R = 2 satisfies equation (5).


Hence lines (1) and (2) intersect. ∴ lines (1) and (2) are coplanar.
Equation of the plane containing lines (1) and (2) is

page 22 of 77
x − 3 y +1 z + 2
2 −3 1
=0
−3 1 2
or, (x – 3) (– 6 – 1) – (y + 1) (4 + 3) + (z + 2) (2 – 9) = 0
or, – 7 (x – 3) – 7 (y + 1) – 7 (z + 2) = 0
or, x–3+y+1+z+2=0 or, x + y + z = 0.
Self practice problems:

Teko Classes, Maths : Suhag R. Kariya (S. R. K. Sir), Bhopal Phone : 0 903 903 7779, 0 98930 58881.
x−2 y+3 z−6
1. Find the values of a and b for which the line = = is perpendicular to the plane
a 4 −2
3x – 2y + bz + 10 = 0. Ans. a = 3, b = – 2
x −1 y − 2 z − 3 x−2 y−3 z−4
2. Prove that the lines = = and = = are coplanar. Also find the equation
2 3 3 3 4 5
of the plane in which they lie. Ans. x – 2y + z = 0
x−2 y−3 z−4 x +1 y −1 −z + 1
3. Find the plane containing the line = = and parallel to the line = =
2 3 5 1 −2 1
Ans. 13x + 3y – 72 – 7 = 0

x −1 y−2 z−3 x−4 y −1


4. Show that the line = = & = = z are intersecting each other. Find thire
2 3 4 5 2
intersection and the plane containing the line. Ans. (– 1, – 1, – 1) & 5x – 8y + 11z – 2 = 0
 
5. Show that the lines r = (– î – 3 ĵ – 5 k̂ ) + λ (–3 î – 5 ĵ – 7 k̂ ) & r (2 î + 4 ĵ + 6 k̂ ) + µ ( î +4 ĵ + 7 k̂ ) are
coplanar and find the plane containing the line. Ans. ( )
r . î − 2 ĵ + k̂ = 0
2 6 . Skew Lines:
Lines
(i) The straight lines which are not parallel and non−coplanar i.e. non−intersecting are called
α'−α β'−β γ '− γ
skew lines. If ∆ =  ≠ 0, then lines are skew..
m n
' m' n'
(ii) Shortest distance: Suppose the equation of the lines are
x −α y −β z− γ x − α' y − β' z − γ '
= = and = =
 m n ' m' n'
(α − α' ) (mn'− m' n) + (β − β' ) (n − n' ) + ( γ − γ' ) (m'−' m)
S.D. =
∑ (mn'−m' n) 2

α'−α β'−β γ '− γ


=
 m n ÷ ∑ ( mn′ − m′n) 2
' m' n'
   
(iii) Vector Form: For lines a 1 + λ b1 & a 2 + λ b 2 to be skew
       
( b1 x b 2 ). ( a 2 − a1 ) ≠ 0 or [ b1 b 2 ( a 2 − a1 )] ≠ 0.
 
(iv) Shortest distance between the two parallel lines r = a 1 + λ b &
  
   (a 2 − a1 ) x b
r = a 2 + µ b is d =  .
|b|
Example : Find the shortest distance and the vector equation of the line of shortest distance between the
lines given by
→ → → →  → → → → → → →  → → →
r = 3 r + 8 j + 3 k + λ  3 r − j + k  and r = −3 r − 7 j + 6 k + µ  − 3 i + 2 j + 4 k 
   
Solution Given lines are
→ → → →  → → →
r = 3 r + 8 j + 3 k + λ  3 i − j + k  ..... (1)
 
A L B
→ → → →  → → →
and r = −3 r − 7 j + 6 k + µ  − 3 i + 2 j + 4 k  ..... (2) 90°
 
FREE Download Study Package from website: www.TekoClasses.com & www.MathsBySuhag.com
Equation of lines (1) and (2) in cartesian form is
x −3 y −8 z−3 90°
AB : = = =λ
3 −1 1 C M D

page 23 of 77
 1 
and CD :   =µ
 3 
Let L ≡ (3λ + 3, – λ + 8, λ + 3)
and M ≡ (– 3µ – 3, 2µ – 7, 4µ + 6)
Direction ratios of LM are
3λ + 3µ + 6, – λ – 2µ + 15, λ – 4µ – 3.
Since LM ⊥ AB
∴ 3 (3λ + 3µ + 6) – 1 (– λ – 2µ + 15) + 1 (λ – 4µ – 3) = 0

Teko Classes, Maths : Suhag R. Kariya (S. R. K. Sir), Bhopal Phone : 0 903 903 7779, 0 98930 58881.
or, 11λ + 7µ = 0 ..... (5)
Again LM ⊥ CD
∴ – 3 (3λ + 3µ + 6) + 2 (– λ – 2µ + 15) + 4 (λ – 4µ – 3) = 0
or, – 7λ – 29µ – 0 ..... (6)
Solving (5) and (6), we get λ = 0, µ = 0
∴ L ≡ (3, 8, 3), M ≡ (– 3, – 7, 6)
Hence shortest distance LM = (3 + 3 ) 2 + ( 8 + 7 ) 2 + ( 3 − 6 ) 2
= 270 = 3 30 units
Vector equation of LM is
→ → → →  → → →
r = 3 i + 8 j + 3 k + t  6 i + 15 j − 3 k 
 
x−3 y−8 z−3
Note : Cartesian equation of LM is = = .
6 15 −3
Example : Prove that the shortest distance between any two opposite edges of a tetrahedron formed by
the planes y + z = 0, x + z = 0, x + y = 0, x + y + z = 3 a is 2 a.
Solution Given planes are y+z=0 ..... (i) x+z=0 ..... (ii)
x+y=0 ..... (iii) x+y+z= 3a ..... (iv)
Clearly planes (i), (ii) and (iii) meet at O(0, 0, 0)
Let the tetrahedron be OABC
Let the equation to one of the pair of opposite edges OA and BC be A
y + z = 0, x + z = 0 ..... (1) , 0) P
O (0, 0
x + y = 0, x + y + z = 3 a ..... (2)
equation (1) and (2) can be expressed in symmetrical form as
x −0 y −0 z−0
= = ..... (3)
1 1 −1 C
(0, 0, Q D
x−0 y−0 z− 3a 3 a
and, = = ..... (4) )
1 −1 0
d. r. of OA and BC are (1, – 1) and (1, – 1, 0).
Let PQ be the shortest distance between OA and BC having direction cosine (, m, n)
∴ PQ is perpendicular to both OA and BC.
∴ +m–n=0
and –m=0
Solving (5) and (6), we get,
 m n
= = = k (say)
1 1 2
2 2
also,  + m + n = 1 2 O
A
1
∴ k2 + k2 + 4k2 = 1 ⇒ k =
6
1 1 2
∴ = ,m= ,n= B C
6 6 6
Shortest distance between OA and BC
i.e. PQ = The length of projection of OC on PQ P A
= | (x 2 – x 1)  + (y2 – y1) m + (z2 – z1) n | O
90°
1 1 2
= 0 . + 0 . + 3 a . = 2 a.
6 6 6 90°
Self practice problems: C Q B
x −1 y − 2 z − 3 x−2 y−4 z−5
1. Find the shortest distance between the lines = = and = = . Find also
2 3 4 3 4 5
FREE Download Study Package from website: www.TekoClasses.com & www.MathsBySuhag.com
1
its equation. Ans., 6x – y = 10 – 3y = 6z – 25
6
2. Prove that the shortest distance between the diagonals of a rectangular parallelopiped whose sides are
bc ca ab

page 24 of 77
a, b, c and the edges not meeting it are , ,
2 2 2 2
b +c c +a a + b2
2

27. Sphere: General equation of a sphere is given by x 2


+ y2 + z2 + 2ux + 2vy + 2wz + d = 0 (−u,
−v, −w) is the centre and u 2 + v 2 + w 2 − d is the radius of the sphere.
Example : Find the equation of the sphere having centre at (1, 2, 3) and touching the plane x + 2y + 3z = 0.
Solution : Given plane is x + 2y + 3z = 0 ..... (1)

Teko Classes, Maths : Suhag R. Kariya (S. R. K. Sir), Bhopal Phone : 0 903 903 7779, 0 98930 58881.
Let H be the centre of the required sphere.
Given H ≡ (1, 2, 3) H
Radius of the sphere,
HP = length of perpendicular from H to plane (1)
P
| 1+ 2 × 2 + 3 × 3 |
= = 14
14
Equation of the required sphere is (x – 1)2 + (y – 2)2 + (z – 3)2 = 14
or x 2 + y2 + z2 – 2x – 4y – 6z = 0
→ → → →
Example : Find the equation of the sphere if it touches the plane r .( 2 i − 2 j − k ) = 0 and the position
→ → →
vector of its centre is 3 i + 6 j − 4 k
→ → → →
Solution Given plane is r .( 2 i − 2 j − k ) = 0 ..... (1)
Let H be the centre of the sphere, then
→ → → → →
OH = 3 i + 6 j − 4 k = c (say)
Radius of the sphere = length of perpendicular from H ot plane (1)
→ → → →
| c .(2 i − 2 j − k ) |
= → → →
|2 i −2 j− k |
→ → → → → →
| (3 i + 6 j − 4 k ).(2 i − 2 j − k ) |
= → → →
|2 i −2 j− k |
| 6 − 12 + 4 | 2
= = = a (say)
3 3
Equation of the required sphere is
→ →
| r−c| = a
→ → → → → → 2 → → 4

or | x i + y j + z k − (3 i + 6 j − 4 k ) | = or | (x – 3) i + (y – 6) j + (z + 4) k |2 =
3 9
4
or (x – 3)2 + (y – 6)2 + (z + 4)2 = or 9 (x 2 + y2 + z2 – 6x – 12y + 8z + 61) = 4
9
or 9x 2 + 9y2 + 9z2 – 54x – 108y + 72z + 545 = 0
Example : Find the equation of the sphere passing through the points (3, 0, 0), (0, – 1, 0), (0, 0, – 2) and
whose centre lies on the plane 3x + 2y + 4z = 1
Solution Let the equation of the sphere be
x 2 + y2 + z2 + 2ux + 2vy + 2wz + d = 0 ..... (1)
Let A ≡ (3, 0, 0), B ≡ (0, – 1, 0), C ≡ (0, 0, – 2)
Since sphere (1) passes through A, B and C,
∴ 9 + 6u + d = 0 ..... (2)
1 – 2v + d = 0 ..... (3)
4 – 4w + d = 0 ..... (4)
Since centre (– u, – v, – w) of the sphere lies on plane
3x + 2y + 4z = 1
∴ – 3u – 2v – 4w = 1 ..... (5)
(2) – (3) ⇒ 6u + 2v = – 8 ..... (6)
(3) – (4) ⇒ – 2v + 4w = 3 ..... (7)
− 2v − 8
From (6), u = ..... (8)
6
From (7), 4w = 3 + 2v ..... (9)
Putting the values of u, v and w in (5), we get
2v + 8
− 2v – 3 – 2v = 1
2
FREE Download Study Package from website: www.TekoClasses.com & www.MathsBySuhag.com

⇒ 2v + 8 – 4v – 6 – 4v = 2 ⇒ v = 0
0−8 4
From (8), u= =−
6 3

page 25 of 77
3
From (9), 4w = 3 ∴ w=
4
From (3), d = 2v – 1 = 0 – 1 = – 1
From (1), equation of required sphere is
0−8 8 3
x 2 + y2 + z2 – − x+ z–1=0
6 3 2
or 6x 2 + 6y2 + 6z2 – 16x + 9z – 6 = 0

Teko Classes, Maths : Suhag R. Kariya (S. R. K. Sir), Bhopal Phone : 0 903 903 7779, 0 98930 58881.
Example :
Find the equation of the sphere with the points (1, 2, 2) and (2, 3, 4) as the extremities of a diameter.
Find the co-ordinates of its centre.
Solution
Let A ≡ (1, 2, 2), B ≡ (2, 3, 4)
Equation of the sphere having (x 1, y1, z1) and (x 2, y2, z2) as the extremities of a diameter is
(x – x 1) (x – x 2) + (y – y1) (y – y2) + (z – z1) (z – z2) = 0
Here x 1 = 1, x 2 = 2, y1 = 2, y2 = 3, z1 = 2, z2 = 4
∴ required equation of the sphere is
(x – 1) (x – 2) + (y – 2) (y – 3) + (z – 2) (z – 4) = 0
or x 2 + y2 + z2 – 3x – 5y – 6z + 16 = 0
Centre of the sphere is middle point of AB
3 5 
∴ Centre is  , , 3 
2 2 

Self practice problems:

1. Find the value of k for which the plane x + y + z = 3 k touches the sphere
x 2 + y2 + z2 – 2x – 2y – 2z – 6 = 0.
Ans. 3 ±3

2. Find the equation to the sphere passing through (1, – 3, 4), (1, – 5, 2) and (1, – 3, 0) which has its
centre in the plane x + y + z = 0
Ans. x 2 + y2 + z2 – 2x + 6y – 4z + 10 = 0

3. Find the equation of the sphere having centre on the line 2x – 3y = 0, 5y + 2z = 0 and passing through
the points (0, – 2, – 4) and (2, – 1, – 1).
Ans. x 2 + y2 + z2 – 6x – 4y + 10 z + 12 = 0

4. Find the centre and radius of the circle in which the plane 3x + 2y – z – 7 14 = 0 intersects the sphere
x 2 + y2 + z2 = 81.
Ans. 4 2 units
5. A plane passes through a fixed point (a, b, c) and cuts the axes in A, B, C. Show that the locus of the
centre of the sphere OABC is
a b c
+ +
x y z = 2.
Vector
FREE Download Study Package from website: www.TekoClasses.com & www.MathsBySuhag.com

1. Vectors & Their Representation:


Vector quantities are specified by definite magnitude and definite directions. A vector is generally

page 26 of 77
represented by a directed line segment, say AB . A is called the initial point & B is called the
terminal point. The magnitude of vector AB is expressed by  AB .
Zero Vector:
Vector A vector of zero magnitude is a zero vector. i.e. which has the same initial & terminal
point, is called a Zero Vector. It is denoted by O. The direction of zero vector is indeterminate.
 
Unit Vector:
Vector A vector of unit magnitude in the direction of a vector a is called unit vector along a

a
and is denoted by â symbolically, â =  .
|a|

Teko Classes, Maths : Suhag R. Kariya (S. R. K. Sir), Bhopal Phone : 0 903 903 7779, 0 98930 58881.
Equal Vectors:
Vectors Two vectors are said to be equal if they have the same magnitude, direction &
represent the same physical quantity.
Collinear Vectors: Two vectors are said to be collinear if their directed line segments are parallel
irrespective of their directions. Collinear vectors are also called parallel vectors. If they have the
same direction they are named as like vectors otherwise unlike vectors.
   
Symbolically, two non zero vectors a and b are collinear if and only if, a = Kb , where K ∈ R
  a1 a2 a3
Vectors a = a1 î + a 2 ĵ + a 3k̂ and b = b1 î + b 2 ĵ + b 3k̂ are collinear if = =
b1 b2 b3
Coplanar Vectors:
Vectors A given number of vectors are called coplanar if their line segments are all
parallel to the same plane. Note that “T WO VECTORS ARE ALWAYS C OPLANAR”.
Solved Example Find unit vector of î − 2 ĵ + 3k̂
 
Solution a = î − 2 ĵ + 3k̂ if a = a x î + a y ĵ + a zk̂
 2 2 2 
then | a | = ax + ay + az ∴ | a | = 14

a 1 2 3
â = | a | = 14 î – ĵ +
14 k̂
14

Solved Example Find values of x & y for which the vectors a = (x + 2) î – (x – y) ĵ + k̂

b = (x – 1) î + (2x + y) ĵ + 2 k̂ are parallel.
  x+2 y−x 1
Solution a and b are parallel if x − 1 = 2x + y = 2
x = – 5, y = – 20
2. Angle Between two Vectors
It is the smaller angle formed when the initial points or the terminal points of the two vectors are
brought together. It should be noted that 0º ≤ θ ≤ 180º .

3. Addition Of Vectors:
  → →  
 If two vectors a & b are represented by OA & OB , then their sum a + b is a vector represented

by OC , where OC is the diagonal of the parallelogram OACB.
         
 a + b = b + a (commutative)  (a + b) + c = a + ( b + c) (associativity)
             
 a+0 = a = 0+a  a + (−a ) = 0 = (−a ) + a  |a+b|≤|a|+|b|
       
 | a − b | ≥|| a| − |b||  a±b = | a |2 + | b |2 ±2 | a || b | cos θ where θ is the angle between the vectors
 
  a b
 A vector in the direction of the bisector of the angle between the two vectors a & b is  +  . Hence
a b

( )
 
bisector of the angle between the two vectors a and b is λ a + b , where λ ∈ R+. Bisector of the exterior
 
( )
angle between a & b is λ a − b , λ ∈ R+.
4. Multiplication Of A Vector By A Scalar:
  
If a is a vector & m is a scalar, then m a is a vector parallel to a whose modulus is m times that of
  
a . This multiplication is called SCALAR MULTIPLICATION. If a and b are vectors & m, n are scalars, then:
     
m (a ) = (a ) m = m a m (na) = n(ma) = (mn )a
     
(m + n ) a = m a + n a m (a + b ) = m a + m b
 
FREE Download Study Package from website: www.TekoClasses.com & www.MathsBySuhag.com

Solved Example: If a = î + 2 ĵ + 3k̂ and b = 2 î + 4 ĵ − 5k̂ represent two adjacent sides of a parallelogram, find
unit vectors parallel to the diagonals of the parallelogram.
 

page 27 of 77
Solution. Let ABCD be a parallelogram such that AB = a and BC = b .
 
Then, AB + BC = AC ⇒ AC = a + b = 3 î + 6 ĵ − 2k̂
and AB + BD = AD ⇒ AD + AD = AB
 
⇒ BD = AD − AB = b − a
Now, AC = 3 î + 6 ĵ − 2k̂ ⇒ | AC | = 9 + 36 + 4 = 7

Teko Classes, Maths : Suhag R. Kariya (S. R. K. Sir), Bhopal Phone : 0 903 903 7779, 0 98930 58881.
and, BD = î + 2 ĵ − 8k̂ ⇒ | BD | = 1 + 4 + 64 = 69

( )
AC 1
∴ Unit vector along AC = = 3 î + 6 ĵ − 2k̂
| AC | 7

(î + 2 ĵ − 8k̂ )
BD 1
and, Unit vector along BD = =
| BD | 69
Solved Example ABCDE is a pentagon. Prove that the resultant of the forces AB , AE , BC , DC , ED
and AC is 3 AC .
Solution. Let R be the resultant force
∴ R = AB + AE + BC + DC + ED + AC
∴ R = ( AB + BC ) + ( AE + ED + DC ) + AC
= AC + AC + AC
= 3 AC . Hence proved.
Self Practice Problems :
1. Express : (i) The vectors BC CA and AB in terms of the vectors OA , OB and OC
(ii) The vectors OA , OB and in terms of the vectors OC , OB and OC .
Ans. (i) BC = OC − OB , CA = OA − OC , AB = OB − OA
2. Given a regular hexagon ABCDEF with centre O, show that
(i) OB – OA = OC – OD (ii) OD + OA = 2 OB + OF (iii) AD + EB + PC = 4 AB

3. The vector − î + ĵ − k̂ bisects the angle between the vectors c and 3 î + 4 ĵ . Determine the unit vector
 1 2 14
along c . Ans. − î + ĵ − k̂
3 15 15
4. The sum of the two unit vectors is a unit vector. Show that the magnitude of the their difference is 3 .
5. Position Vector Of A Point:
Point
 
let O be a fixed origin, then the position vector of a point P is the vector OP . If a and b are position
vectors of two points A and B, then,
 
AB = b − a = pv of B − pv of A.
DISTANCE
DISTANCE FORMULA
F ORMULA
   
Distance between the two points A (a) and B (b) is AB = a − b
SECTION FORMULA
 
If a and b are the position vectors of two points A & B then the p.v. of
 
 na + m b
a point which divides AB in the ratio m: n is given by: r = .
 m+n

a+b
Note p.v. of mid point of AB = .
2
Solved Example: ABCD is a parallelogram. If L, M be the middle point of BC and CD, express AL and
3
AM in terms of AB and AD , also show that AL + AM = AC .
2
 
Solution.Let the position vectors of points B and D be respectively b and a referred to A as origin of reference.
Then AC = AD + DC = AD + AB [∵ DC = AB ]
   
= d + b ∵ AB = b , AD = d
 
i.e. position vector of C referred to A is d + b
FREE Download Study Package from website: www.TekoClasses.com & www.MathsBySuhag.com

∴ AL = p.v. of L, the mid point of BC .

=
1
[p.v. of D + p.v. of C] =
1   
(
b + d + b = AB +
1
)
AD

page 28 of 77
2 2 2
1   
[ ]
AM = 2 d + d + b = AD + 2 AB
1

 1   1 
∴ AL + AM = b + d + d+ b
2 2
3  3  3   3
= b + d = (b + d ) = AC .

Teko Classes, Maths : Suhag R. Kariya (S. R. K. Sir), Bhopal Phone : 0 903 903 7779, 0 98930 58881.
2 2 2 2
Solved Example If ABCD is a parallelogram and E is the mid point of AB, show by vector method that
DE trisects and is trisected by AC.
 
Solution. Let AB = a and AD = b
  
Then BC = AD = b and AC = AB + AD = a + b
Also let K be a point on AC, such that AK : AC = 1 : 3

1 1  
or, AK = AC ⇒ AK = (a + b ) .........(i)
3 3
Again E being the mid point of AB, we have
1 
AE = a
2
Let M be the point on DE such that DM : ME = 2 : 1
 
AD + 2AE b+a
∴ AM = = ..........(ii)
1+ 2 3
1  
From (i) and (ii) we find that : AK = ( a + b ) = AM , and so we conclude that K and M coincide. i.e. DE
3
trisect AC and is trisected by AC. Hence proved.
Self Practice Problems
   
1. If a, b are position vectors of the points (1, –1), (–2, m), find the value of m for which a and b are
collinear. Ans. m = 2
2. The position vectors of the points A, B, C, D are î + ĵ + k̂ , 2 î + 5 ĵ , 3 î + 2 ĵ − 3k̂ , î − 6 ĵ − k̂ respectively..
Show that the lines AB and CD are parallel and find the ratio of their lengths. Ans. 1 : 2
  
3. The vertices P, Q and S of a triangle PQS have position vectors p, q and s respectively..
  
(i) Find m , the position vector of M, the mid-point of PQ, in terms of p and q .
   
(ii) Find t , the position vector of T on SM such that ST : TM = 2 : 1, in terms of p, q and s .

(iii) If the parallelogram PQRS is now completed. Express r , the position vector of the point R in
  
terms of p, q and s
Prove that P, T and R are collinear.
 1    1     1   
Ans. m = 2 (p + q) , t =
2
(p + q + s) , r = 2 q+p−s

4. D, E, F are the mid-points of the sides BC, CA, AB respectively of a triangle. Show FE = 1/2 BC and
that the sum of the vectors AD , BE , CF is zero.
5. The median AD of a triangle ABC is bisected at E and BE is produced to meet the side AC in F; show
that AF = 1/3 AC and EF = 1/4 BF.
6. Point L, M, N divide the sides BC, CA, AB of ∆ABC in the ratios 1 : 4, 3 : 2, 3 : 7 respectively. Prove
that AL + BM + CN is a vector parallel to CK , when K divides AB in the ratio 1 : 3.

6. Scalar Product Of Two Vectors:


Geometrical interpretation of Scalar Product
 
Let a and b be vectors represented by OA and OB respectively. Let θ be the angle between OA and
OB . Draw BL ⊥ OA and AM ⊥ OB.
From ∆s OBL and OAM, we have OL = OB cos θ and OM = OA cos θ. Here OL and OM are known as
   
projections of b on a and a on b respectively..
   
Now, a.b = | a | | b | cos θ

= | a | (OB cos θ )
FREE Download Study Package from website: www.TekoClasses.com & www.MathsBySuhag.com


= | a | (OL)
  
= (Magnitude of a ) (Projection of b on a ) ........(i)

page 29 of 77
     
Again a . b = | a | | b | cos θ = | b | (| a | cos θ )

= | b | (OA cos θ)

= | b | (OM)
  
= (magnitude of b ) (Projection of a on b ) ........(ii)
Thus geometrically interpreted, the scalar product of two vectors is the product of modulus of either
vector and the projection of the other in its direction.

Teko Classes, Maths : Suhag R. Kariya (S. R. K. Sir), Bhopal Phone : 0 903 903 7779, 0 98930 58881.
 
 a.b
1. i.i = j.j = k.k = 1; i.j = j.k = k.i = 0  projection of a on b = 
|b|
   
2. if a = a1i + a2j + a3k & b = b1i + b2j + b3k then a . b = a1b1 + a2b2 + a3b3
 
a = a 12 + a 2 2 + a 3 2 , b = b12 + b 2 2 + b 32

  a.b
3. the angle φ between a & b is given by cos φ =   0≤φ≤π
|a| |b|
       
4. a . b = a b cos θ (0 ≤ θ ≤ π) , note that if θ is acute then a . b > 0 & if θ is obtuse then a . b < 0
  2          
5. a . a = a = a 2 , a.b = b.a (commutative)  a . (b + c ) = a . b + a . c (distributive)
     
6. a.b = 0 ⇔ a ⊥ b (a ≠ 0 b ≠ 0 )
   
7. (m a ) . b = a . (m b) = m (a . b) (associative) where m is scalar..
       
Note : (i) Maximum value of a . b is  a   b  (ii) Minimum value of a . b is –  a   b 
(iii)

( ) ( ) ( )
         
Any vector a can be written as, a = a . i i + a . j j + a . k k .


Solved Example Find the value of p for which the vectors a = 3 î + 2 ĵ + 9k̂ and b = î + p ĵ + 3k̂ are
(i) perpendicular (ii) parallel
Solution. (i)
 
a⊥b ⇒
 
a.b = 0 ⇒ ( ) ( )
3 î + 2 ĵ + 9k̂ . î + p ĵ + 3k̂ = 0
⇒ 3 + 2p + 27 = 0 ⇒ p = – 15
 
(ii) We know that the vectors a = a1î + a 2 ĵ + a 3k̂ and b = b1î + b 2 ĵ + b 3k̂ are parallel iff
 
( ) ( )
a = λb ⇔ a1î + a 2 ĵ + a 3k̂ = λ b1î + b 2 ĵ + b 3k̂ ⇔ a1 = λb1, a2 = λb2, a3 = λb3
a1 a2 a3
⇔ b = b = b ( =λ)
1 2 3
 
So, vectors a = 3 î + 2 ĵ + 9k̂ and b = î + p ĵ + 3k̂ are parallel iff
3 2 9 2
= = ⇒ 3= ⇒ p=2/3
1 p 3 p
        
Solved Example: If a + b + c = 0 , | a | = 3, | b | = 5 and | c | = 7, find the angle between a and b .
   
Solution. We have, a + b + c = 0

 
a+b = –c


 
(  
) ( )
a + b . a + b = (− c ) . (− c )
 

  2   2  2    2
⇒ a + b = | c |2 ⇒ a + b + 2a . b = c
 2  2   
⇒ a + b +2 a b cos θ = c 2
1 π
⇒ 9 + 25 + 2 (3) (5) cos θ = 49 ⇒ cos θ = . ⇒ θ=
2 3

Solved Example Find the values of x for which the angle between the vectors a = 2x 2 î + 4x ĵ + k̂ and

b = 7 î – 2 ĵ + x k̂ is obtuse.
 
  a.b
Solution. The angle q between vectors a and b is given by cos θ =  
|a||b|
 
a.b
Now, θ is obtuse ⇒ cos θ < 0 ⇒   <0
|a||b|
   
FREE Download Study Package from website: www.TekoClasses.com & www.MathsBySuhag.com

⇒ a.b < 0 [∵, | a |, | b | > 0 ]


⇒ 14x 2 – 8x + x < 0
1

page 30 of 77
⇒ 17x (2x – 1) < 0 ⇒ x(2x – 1) < 0 ⇒ 0 < x <
2
Hence, the angle between the given vectors is obtuse if x ∈ (0, 1/2)
Solved Example:D is the mid point of the side BC of a triangle ABC, show that AB2 + AC2 = 2 (AD2 + BD2)
Solution. We have
AB = AD + DB
⇒ AB2 = ( AD + DB )2

Teko Classes, Maths : Suhag R. Kariya (S. R. K. Sir), Bhopal Phone : 0 903 903 7779, 0 98930 58881.
= AD2 + DB + 2AD . DB .............(i)
Also we have
AC2 = ( AD + DC)
2
AC = AD + DC ⇒
= AD2 + DC2 + 2AD . DC .............(ii)
Adding (i) and (ii), we get
AB2 + AC2 = 2AD2 + 2BD2 + 2 AD . (DB + DC)
= 2(DA2 + DB2), for DB + DC = 0
Solved Example
 
If a = î + ĵ + k̂ and b = 2 î – ĵ + 3 k̂ , then find
   
(i) Component of b along a . (ii) Component of b perpendicular to along a .
 
Solution. (i) Component of b along a is
 
 a.b  
   a
 | a |2 
 
 
Here a . b = 2 – 1 + 3 = 4

| a |2 = 3
 
 a.b   4  4
  
Hence  2  a = a = ( î + ĵ + k̂ )
 | a |  3 3
 
 a.b  
( )
     1
(ii) Component of b perpendicular to along a is b –   2  a . = 2 î − 7 ĵ + 5k̂
| a |  3
  θ
Self Practice Problems :1. If a and b are unit vectors and θ is angle between them, prove that tan =
 2

|a−b| 
  . 2. Find the values of x and y is the vectors a = 3 î + xĵ − k̂ and b = 2 î + ĵ + yk̂ are mutually
|a+b|
31 41
perpendicular vectors of equal magnitude. Ans. x = – , y=
12 12
 
3. Let a = x 2 î + 2 ĵ − 2k̂ , b = î − ĵ + k̂ and c = x 2 î + 5 ĵ − 4k̂ be three vectors. Find the values of x for which the
   
angle between a and b is acute and the angle between b and c is obtuse. Ans. (–3, –2) ∪ (2, 3)
     
4. The points O, A, B, C, D, are such that OA = a , OB = b , OC = 2a + 3b , OD = a + 2b . Give that the
length of OA is three times the length of OB show that BD and AC are perpendicular..
5. ABCD is a tetrahedron and G is the centroid of the base BCD. Prove that
AB 2 + AC2 + AD2 = GB 2 + GC2 + GD2 + 3GA 2
7. Vector Product Of Two Vectors:
       
1. If a & b are two v ectors & θ is the angle between them then a x b = a b sin θ n , where n is the unit
    
vector perpendicular to both a & b such that a , b & n forms a right handed screw system.
   
2. Geometrically a x b = area of the parallelogram whose two adjacent sides are represented by a & b .

3. î × î = ĵ × ĵ = k̂ × k̂ = 0 ; î × ĵ = k̂, ĵ × k̂ = î, k̂ × î = ĵ

 î ĵ k̂
  
FREE Download Study Package from website: www.TekoClasses.com & www.MathsBySuhag.com

4. If a = a1 î +a2 ĵ + a3 k̂ & = b1 î + b2 ĵ + b3 k̂ then a × b = a1 a 2 a 3


b
b1 b 2 b3

page 31 of 77
   
5. a x b ≠ b x a (not commutative)
   
6. (m a ) × b = a × m b = m a × b (associative) where m is a scalar..
( ) ( )
      
7. a x ( b + c ) = (a x b) + (a x c ) (distributive)
       
8. a x b = 0 ⇔ a & b are parallel (collinear) (a ≠ 0 , b ≠ 0) i.e. a = K b , where K is a scalar..

Teko Classes, Maths : Suhag R. Kariya (S. R. K. Sir), Bhopal Phone : 0 903 903 7779, 0 98930 58881.
 
  axb
9. Unit vector perpendicular to the plane of a & b is n = ±  
axb


 
 
r axb
A vector of magnitude ‘r’ & perpendicular to the palne of a & b is ±  
( )
axb
 
  axb
 If θ is the angle between a & b then sin θ =  
a b
  
 If a , b & c are the pv’s of 3 points A, B & C then the vector area of triangle ABC =

[
1  
]
          
a x b + b x c + cxa . The points A, B & C are collinear if a x b + b x c + c x a = 0
2
1  
 
 Area of any quadrilateral whose diagonal vectors are d1 & d 2 is given by d1 x d 2
2
   
    2  2  2   2 a .a a . b
 Lagrange's Identity: for any two vectors a & b ;(a x b) = a b − (a . b) =    
a .b b.b
Solved Example
Find a vector of magnitude 9, which is perpendicular to both the vectors 4 î + ĵ + 3k̂ and − 2 î + ĵ − 2k̂ .
 
Solution. Let a = 4 î − ĵ + 3k̂ and b = − 2 î + ĵ − 2k̂ . Then,

î ĵ k̂
  4 − 1 3 = (2 – 3) – (–8 + 6) + (4 – 2)
a×b = î ĵ k̂ = − î + 2 ĵ + 2k̂
−2 1 −2
 
⇒ | a × b | = ( −1)2 + 2 2 + 2 2 = 3
 
 a×b  9
   
∴ Required vector = 9   = ( − î + 2 ĵ + 2k̂ ) = − 3 î + 6 ĵ + 6k̂
| a×b | 3
Solved Example
            
For any three vectors a, b, c . Show that a × (b + c ) + b × (c + a) + c × (a + b) = 0 .
        
Solution. We have, a × (b + c ) + b × (c + a) + c × (a + b)
           
= a × b + a × c + b × c + b × a + c × a + c × b [Using distributive law]
               
= a×b + a×c + b×c − a×b − a×c −b×c [∵ a × b = −b × a etc]
    
Solved Example: For any vector a , prove that | a × î |2 + | a × ĵ |2 + | a × k̂ |2 = 2 | a |2

Solution. Let a = a1î + a 2 ĵ + a 3k̂ . Then

a × î = (a1î + a 2 ĵ + a 3k̂ ) × î = a1 ( î × î ) + a2 ( ĵ × î ) + a3 (k̂ × î ) = –a2 k̂ + a 3 î

⇒ | a × î |2 = a22 + a32

a × ĵ = (a1î + a 2 ĵ + a 3k̂ ) × ĵ = a1k̂ − a 3 î
 
⇒ | a × ĵ |2 = a21 + a32 ⇒ | a × k̂ |2 = a12 + a22
  
∴ | a × î |2 + | a × ĵ |2 + | a × k̂ |2 = a22 + a33 + a12 + a32 + a12 + a22

2 (d12 + a22 + a32) = 2 | a |2
   
Solved Example: Let OA = a , OB = 10 a + 2b and OC = b where O is origin. Let p denote the area of the
quadrilateral OABC and q denote the area of the parallelogram with OA and OC as adjacent sides.
Prove that p = 6q.
FREE Download Study Package from website: www.TekoClasses.com & www.MathsBySuhag.com

Solution. We have,
p = Area of the quadrilateral OABC
1

page 32 of 77
= | OB × AC |
2
1
= | OB × (OC − OA ) |
2
1    
= | (10a + 2b ) × (b − a) |
2
1        
= | 10(a × b − 10(a × a) + 2(b × b ) − 2(b × a) |

Teko Classes, Maths : Suhag R. Kariya (S. R. K. Sir), Bhopal Phone : 0 903 903 7779, 0 98930 58881.
2
1    
= | 10(a × b) − 0 + 0 + 2(a × b) |
2
and, q = Area of the parallelogram with OA and OC as adjacent sides
 
= | OA × OC | = | a × b | ........(ii)
From (i) and (ii), we get p = 6q
Self Practice Problems :
    
1. If p and q are unit vectors forming an angle of 30º; find the area of the parallelogram having a = p + 2q
  
and b = 2p + q as its diagonals. Ans. 3/4 sq. units
        
2. Show that {( a + b + c ) × ( c – b )} . a = 2 [ a b c ].
  
3. Prove that the normal to the plane containing the three points whose position vectors are a, b, c lies in
     
the direction b × c + c × a + a × b
4. ABC is a triangle and EF is any straight line parallel to BC meeting AC, AB in E, F respectively. If BR
and CQ be drawn parallel to AC, AB respectively to meet EF in R and Q respectively, prove that
∆ ARB = ∆ACQ.
8. Scalar Triple Product:
        
 a x b . c = a b c sin θ cos φ where
The scalar triple product of three vectors a , b & c is defined as:
       
θ is the angle between a & b & φ is the angle between a x b & c . It is also written as [ a b c ] and
spelled as box product.
 Scalar triple product geometrically represents the volume
of the parallelopiped whose three coterminous edges are
     
represented by a , b & c i. e. V = [ a b c ]

 In a scalar triple product the position of dot & cross can be interchanged i.e.
            
a . ( b x c ) = (a x b). c OR [ a b c ] = [ b c a ] = [ c a b ]
       
 a . (b x c) = − a .( cx b) i. e. [ a b c ] = − [ a c b ]
1 2 a a a
3
   
 If a = a1i+a2j+a3k; b = b1i+b2j+b3k & c = c1i+c2j+c3k then [a b c] = b1 b 2 b 3 .
c1 c2 c3
           
In general, if a = a 1 l + a 2 m + a 3 n ; b = b1 l + b 2 m + b 3 n & c = c1 l + c2 m + c3 n
a1 a 2 a 3
[ ]

[ ]
   
then a b c = b1 b 2 b 3 l m n ; where  , m & n are non coplanar vectors.
c1 c2 c3
   
 If a , b , c are coplanar ⇔ [ a b c ] = 0 .

 Scalar product of three vectors, two of which are equal or parallel is 0 i.e. [ a b c ] = 0 ,
    
 If a , b , c are non − coplanar then [ a b c ] > 0 for right handed system & [ a b c ] < 0 for left handed
system.
      
 [i j k] = 1  [ K a b c ] = K[ a b c ][(a + b) c d ] = [ a c d ] + [ b c d ]

  
 The volume of the tetrahedron OABC with O as origin & the pv’s of A, B and C being a , b & c respectively
1 
is given by V = [ a b c ]
6
   
 The positon vector of the centroid of a tetrahedron if the pv’s of its vertices are a , b , c & d are given by
1    
[a + b + c + d ] .
FREE Download Study Package from website: www.TekoClasses.com & www.MathsBySuhag.com

4
Note that this is also the point of concurrency of the lines joining the vertices to the centroids of the
opposite faces and is also called the centre of the tetrahedron. In case the tetrahedron is regular it is

page 33 of 77
equidistant from the vertices and the four faces of the tetrahedron.
[ ] [ ] [  ]
          
Remember that: a − b b − c c − a = 0 & a + b b + c c + a = 2 a b c .
Solved Example
Find the volume of a parallelopiped whose sides are given by − 3 î + 7 ĵ + 5k̂ , − 5 î + 7 ĵ − 3k̂ and 7 î − 5 ĵ − 3k̂
  
Solution. Let a = −3 î + 7 ĵ + 5k̂ , b = −5 î + 7 ĵ + 3k̂ and c = 7 î − 5 ĵ − 3k̂ .
     

Teko Classes, Maths : Suhag R. Kariya (S. R. K. Sir), Bhopal Phone : 0 903 903 7779, 0 98930 58881.
We know that the volume of a parallelopiped whose three adjacent edges are a, b, c is [a, b, c ] .
−3 7 5
 −5 7 −3
Now, [a b c ] = = –3 (–21 – 15) – 7 (15 + 21) + 5 (25 – 49)
7 −5 −3
= 108 – 252 – 120 = –264
  
So, required volume of the parallelopiped = [a, b, c ] = | – 264 | = 264 cubic units
   
Solved Example: Simplify [a − b b − c c − a]
Solution. We have :
         
[a − b b − c c − a] = {(a − b) × (b − c )} . (c − a) [by def.]
         
= ( a × b − a × c − b × b + b × c ) . (c − a ) [by dist. law]
         
= ( a × b + c × a + b × c ) . (c − a ) [ ∵ b×b = 0 ]
                 
= (a × b) . c – (a × b) . a + (c × a) . c – (c × a) . a + (b × c ) . c – (b × c ) . a
[by dist. law]
     
= [a b c ] – [a b a] + [c a c ] – [c a a] + [b c c ] – [b c a]
 
= [a b c ] – [b c a] [∵ scalar triple product when any two vectors are equal is zero ]
   
= [a b c ] – [a b c ] =0 [∵ [b c a] = [a b c ] ]
   
Solved Example: Find the volume of the tetrahedron whose four vertices have position vectors a b c and d .
   
Solution. Let four vertices be A, B, C, D with p. v. a b c and d . respectively..
 
∴ DA = (a – d)
 
DB = (b – d )
 
DC = (c – d)
1      
Hence volume = [a – d b – d c – d]
6
1      
= ( a – d ) . [( b – d ) × ( c – d )]
6
1        
= (a – d) . [b × c – b × d + c × d]
6
1            
= {[ a b c ] – [ a b d ] + [ a c d ] – [ d b c ]}
6
1            
= {[ a b c ] – [ a b d ] + [ a c d ] – [ b c d ]}
6
           
Solved Example: Show that the vectors a = −2 i + 4 j − 2k , b = 4 i − 2 j − 2k and c = −2 i − 2 j + 4k are coplanar..
−2 4 −2

Solution: The vectors are coplanar since [a b c ] = −2 −2 = 0
4
−2 −2 4
     
Self Practice Problems : 1. Show that a . (b + c ) × (a + b + c ) = 0
2. One vertex of a parallelopiped is at the point A (1, –1, –2) in the rectangular cartesian co-ordinate. If three
adjacent vertices are at B(–1, 0, 2), C(2, –2, 3) and D(4, 2, 1), then find the volume of the parallelopiped.
Ans. 72
3. Find the value of m such that the vectors 2 î − ĵ + k̂ , î + 2 ĵ − 3k̂ and 3 î + m ĵ + 5k̂ are coplanar..
Ans. – 4
        
4. Show that the vector a, b, c , are coplanar if and only if b + c , c + a , a + b are coplanar..
9. Vector Triple Product:
   
FREE Download Study Package from website: www.TekoClasses.com & www.MathsBySuhag.com

 
Let a , b , c be any three vectors, then the expression a x ( b x c ) is a vector & is called a vector
triple product.
  

page 34 of 77
Geometrical Interpretation of a x ( b x c )
  
Consider the expression a x ( b x c ) which itself is a vector, since it is a cross product of two
        
vectors a & ( bx c ) . Now a x ( b x c ) is a vector perpendicular to the plane containing a & ( b x c )
      
but b x c is a vector perpendicular to the plane containing b & c , therefore a x ( b x c ) is a vector
     
which lies in the plane of b & c and perpendicular to a . Hence we can express a x ( b x c ) in terms

Teko Classes, Maths : Suhag R. Kariya (S. R. K. Sir), Bhopal Phone : 0 903 903 7779, 0 98930 58881.
      
of b & c i.e. a x ( b x c ) = xb + yc where x & y are scalars.
                 
 a x ( b x c ) = (a . c) b − (a . b) c  (a x b) x c = (a . c) b − (b . c) a
     
 (a x b) x c ≠ a x (b x c) , in general
     
Solved Example For any vector a , prove that î × (a × î ) + ĵ × (a × ĵ ) + k̂ × (a × k ) = 2a
  
Solution. Let a = a1î + a 2 ĵ + a 3k̂ . Then, î × (a × î ) + ĵ × (â × ĵ) + k̂ × (a × k̂ )
     
= {( î . î )a − ( î . a) î } + {( ĵ . ĵ )a − ( ĵ . a) ĵ} + {(k̂ . k̂ )a − (k̂ . a)k̂ }
     
= {(a − ( î . a) î } + {a − ( ĵ . a) ĵ} + {a − (k̂ . a)k̂ }
   
= 3a − {( î . a) î + ( ĵ . a) ĵ + (k̂ . a )k̂

= 3a − (a1î + a 2 ĵ + a 3k̂ )
  
= 3a − a = 2a
           
Solved Example Prove that a × {b × (c × a)} = (b . d)(a × c ) – (b. c ) (a × d)
Solution. We have,
          
a × {b × (c × a)} = a × {(b . d) c − (b . c ) d}
       
= a × {(b . d) c } − a × {(b . c ) d} [by dist. law]
         
= (b . d) (a × c ) − (b . c ) − (b . c ) (a × d)
  
Solved Example: Let a = α î + 2 ĵ – 3k̂ , b = î + 2α ĵ – 2k̂ and c = 2 î – α ĵ + k̂ . Find the value(s) of α, if
any, such that
 
{(
a×b × b×c )(
 
)} × ( c × a ) = 0. Find the vector product when α = 0.
 

Solution.: {(
 
a×b × b×c )(  
)}
× ( c ×a )
 

[
   
]
= a b c b × ( c×a )
 
= a b c[
] {( ) ( ) }
        
a.b c − b.c a
  
( ) ( )
  
which vanishes if (i) a . b c = b . c a (ii) a b c = 0 [ ]   

( )
     
( )
(i) a . b c = b . c a leads to the equation 2 α3 + 10 α + 12 = 0, α2 + 6α = 0 and 6α – 6 = 0, which do
not have a common solution. (ii) a b c = 0 [
  
]
α 2 −3
1 2α −2 2
⇒ =0 ⇒ 3α = 2 ⇒ α=
2 −α 1 3

[ ] ( )
      
when α = 0, a b c = – 10, a . b = 6, b . c = 0 and the vector product is – 60 2 î + k̂ .
        
  
a×b + a 
   
(
b × a + a | a |2 −1 )
Sol ExaIf A + B = a , A . a = 1 and A × B = b , then prove that A =  and B =  .
| a |2 | a |2
  
Solution.: Given A + B = a .....(i)
( )  
            2
⇒ a. A +B = a.a ⇒ a. A +a . B = a.a ⇒ 1 + a.B = |a|
( )
   2        
⇒ a.B = |a| – 1 Given A × B = b ⇒ a× A ×B = a × b

  
( ) (     
a . B A – a . A B = a×b )


( )    
| a |2 −1 A − B = a × b [using equation (2)]
solving equation (1) and (5), simultaneously, we get

  
a×b + a


  
b × a + a | a |2 −1

( )
A = and B =
| a |2 | a |2
        
Sol. Ex. Solve for r , the simultaneous equations r × b = c × b , r . a = 0 provided a is not perpnedicular to b.
     
Solution (r − c) × b = 0 ⇒ r − c and b are collinear
    

FREE Download Study Package from website: www.TekoClasses.com & www.MathsBySuhag.com

r − c = kb ⇒ r = c + kb ........(i)
    
r .a = 0 ⇒ (c + kb ) . a = 0
   

page 35 of 77
a.c   a.c 
⇒ k=–   putting in (i) we get r =c−   b
a.b a.b
      
Solved Example :If x × a + kx = b , where k is a scalar and a, b are any two vectors, then determine x in
 
terms of a, b and k.
   
Solution: x × a + kx = b ..........(i)


Teko Classes, Maths : Suhag R. Kariya (S. R. K. Sir), Bhopal Phone : 0 903 903 7779, 0 98930 58881.
Premultiple the given equation vectorially by a
      
a × ( x × a ) + k (a × x ) = a × b
         
⇒ (a . a) x − (a . x ) a + k(a × x ) = a × b ..........(ii)

Premultiply (i) scalarly by a
      
[a x a ] + k ( a . x ) = a . b
   
k(a . x ) = a . b .......(iii)
   
Substituting x × a from (i) and a . x from (iii) in (ii) we get
 
 1     (a . b )  
x = a 2 + k 2 kb + (a × b) + k a
 
        
Self Practice Problems : 1. Prove that a × (b × c ) + b × (c × a) + c × (a × b ) = 0 .
2. Find the unit vector coplanar with î + ĵ + 2 k̂ and î + 2 ĵ + k̂ and perpendicular to î + ĵ + k̂ .
1 1
Ans. ( – ĵ + k̂ ) or, ( ĵ – k̂ )
2 2
       
3. Prove that a × {a × (a × b)} = (a . a) (b × a) .
 1       1   
4. Given that x +  2 (p . x ) p = q , show that p . x = p . q and find x in terms of p and q .
p 2
      
5. If x . a = 0, x . b = 0 and x . c = 0 for some non-zero vector x , then show that [a b c ] = 0
           
 ( r . a) (b × c ) ( r . b ) (c × a ) ( r . c ) (a × b )   
6. Prove that r = + + where a, b, c are three non-coplanar vectors
[abc ] [abc ] [abc ]
     
1 0 . Reciprocal System Of Vectors: If a, b, c & a' , b' , c' are two sets of non coplanar
    
vectors such that a.a' = b. b' = c. c' = 1 then the two systems are called Reciprocal System of vectors.
     
 bxc  cxa  axb
Note:
[ ] [
a=    b=    c =   
abc abc ]
abc [ ]
   
Solved Example If a b c and a′, b′, c ′ be the reciprocal system of vectors, prove that
            
(i) a . a′ + b . b′ + c . c ′ = 3 (ii) a × a′ + b × b′ + c × c ′ = 0
     
Solution. (i) We have : a . a′ = b . b′ = c . c ′ = 1
     
a . a′ + b . b′ + c . c ′ = 1 + 1 + 1 = 3
         1

(ii) We have : a′ = λ (b × c ) , b′ = λ (c × a) and c ′ = λ ( (a × b) , where λ = [a b c ]
             
a × a′ = a × λ(b × c ) = λ {a × (b × c )} = λ {(a . c ) b − (a . b) c }
             
b × b′ = b × λ(c × a) = λ {b × (c × a )} = λ {(b . a) c − (b . c ) a}
             
and c × c ′ = c × λ(a × b) = λ {c × (a × b )} = λ {(c . b ) a − (c . a) b}
     
∴ a × a′ + b × b′ + c × c ′
                 
= λ {(a . c ) b − (a . b) c } + λ {(b . a) c − (b . c ) a} + λ {(c . b) a − (c . a ) b}
                 
= λ [(a . c ) b − (a . b) c + (b . a ) c − (b . c ) a + (c . b) a − (c . a ) b]
                 
= λ [(a . c ) b − (a . b) c + (a . b ) c − (b . c ) a + (b . c ) a − (a . c ) b]
 
= λ0 = 0
1 1 . Linear Combinations:
      
Given a finite set of vectors a , b , c ,...... then the vector r = xa + yb + zc + ........ is called a linear
FREE Download Study Package from website: www.TekoClasses.com & www.MathsBySuhag.com

  
combination of a , b , c ,...... for any x, y, z..... ∈ R. We have the following results:
     
(a) If a , b are non zero, non−collinear vectors then xa + yb = x' a + y' b ⇒ x = x' ; y = y'
  

page 36 of 77
(b) FundamentalTheorem: Let a , b be non zero, non collinear vectors. Then any vector r coplanar
   
with a , b can be expressed uniquely as a linear combination of a , b
  
i.e. There exist some uniquly x, y ∈ R such that xa + yb = r .
  
(c) If a , b , c are non−zero, non−coplanar vectors then:
     
xa + yb + zc = x' a + y' b + z' c ⇒ x = x' , y = y' , z = z'

Teko Classes, Maths : Suhag R. Kariya (S. R. K. Sir), Bhopal Phone : 0 903 903 7779, 0 98930 58881.
  
(d) Fundamental Theorem In Space: Let a , b , c be non−zero, non−coplanar vectors in space. Then any
   
vector r , can be uniquly expressed as a linear combination of a , b , c i.e. There exist some unique x,y
   
∈ R such that xa + yb + zc = r .
  
(e) I f x1 , x 2 ,...... x n are n non zero v ect ors, & k 1 , k 2 , . . . . . k n are n scal ars & i f t h e l i near
  
com bi n at i on k 1x1 + k 2 x 2 +........ k n x n = 0 ⇒ k1 = 0, k 2 = 0..... k n = 0 t hen we say t hat
  
vectors x1 , x 2 ,...... x n are L INEARLY I NDEPENDENT VECTORS.
  
(f) If x1 , x 2 ,...... x n are not L INEARLY I NDEPENDENT then they are said to be L INEARLY DEPENDENT vectors. i.e.
     
if k 1x1 + k 2 x 2 + ........ + k n x n = 0 & if there exists at least one kr ≠ 0 then x1 , x 2 ,...... x n are said to
be
L I N E AR LY
LY D E PE N D E N T .
    
Note 1: If kr ≠ 0; k1x1 + k 2 x 2 + k 3 x 3 + ....... + k r x r + ...... + k n x n = 0
     
− k r x r = k1x1 + k 2 x 2 + ....... + k r −1 . x r −1 + k r +1 . x r +1 +......+ k n x n
1  1  1  1  1 
−k r x r = k1 x1 + k 2 x 2 + .....+ k r −1 . x r −1 +.....+ k n xn
kr kr kr kr kr
     
x r = c1x1 + c 2 x 2 +......+ c r −1x r −1 + c r x r −1 +......+ c n x n

i.e. x r is expressed as a linear combination of vectors.
    
x1 , x 2 ,.......... x r −1 , x r +1 ,........... x n
     
Hence x r with x1 , x 2 ,.... x r −1 , x r +1 .... x n forms a linearly dependent set of vectors.
  
Note 2:  If a = 3 î + 2 ĵ + 5 k̂ then a is expressed as a LINEAR COMBINATION of vectors î , ĵ , k̂ Also, a , î , ĵ ,
k̂ form a linearly dependent set of vectors. In general, every set of four vectors is a linearly dependent
system.
 î , ĵ , k̂ are Linearly Independent set of vectors. For K î + K ĵ + K k̂ = 0 ⇒ K = K = K = 0
    1 2  3 1 2 3

 Two vectors a & b are linearly dependent ⇒ a is parallel to b i.e. a x b = 0 ⇒ linear dependence
     
of a & b . Conversely if a x b ≠ 0 then a & b are linearly independent.
     
 If three vectors a, b, c are linearly dependent, then they are coplanar i.e. [ a , b, c ] = 0 , conversely,,
  
if [ a , b, c ] ≠ 0 , then the vectors are linearly independent.
       
Solved Example: Given A that the points a – 2 b + 3 c , 2 a + 3 b – 4 c , – 7 b + 10 c , A, B, C have
position vector prove that vectors AB and AC are linearly dependent.
Solution. Let A, B, C be the given points and O be the point of reference then
       
OA = a – 2 b + 3 c , OB = 2 a + 3 b – 4 c and OC = – 7 b + 10 c
Now AB = p.v. of B – p.v. of A
  
= OB – OA = ( a + 5 b – 7 c ) = – AB
∴ AC = λ AB where λ = – 1. Hence AB and AC are linearly dependent
        
Solved Example: Prove that the vectors 5 a + 6 b + 7 c , 7 a – 8 b + 9 c and 3 a + 20 b + 5 c are linearly
  
dependent a , b , c being linearly independent vectors.
Solution. We know that if these vectors are linearly dependent , then we can express one of
them as a linear combination of the other two.
Now let us assume that the given vector are coplanar, then we can write
        
5 a + 6 b + 7 c = ( 7 a – 8 b + 9 c ) + m (3 a + 20 b + 5 c )
where , m are scalars
 
FREE Download Study Package from website: www.TekoClasses.com & www.MathsBySuhag.com


Comparing the coefficients of a , b and c on both sides of the equation
5 = 7 + 3 ..........(i) 6 = – 8 + 20 m ..........(ii)
7 = 9 + 5m ..........(iii)
From (i) and (iii) we get

page 37 of 77
1
4 = 8 ⇒ = = m which ev idently satisfies (ii) equation too.
2
Hence the given vectors are linearly dependent .
Self Practice Problems :
           
1. Does there exist scalars u, v, w such that ue1 + ve 2 + we 3 = i where e1 = k , e 2 = j + k , e 3 = − j + 2k ?
Ans. No
     
2. Consider a base a, b, c and a vector − 2a + 3b − c . Compute the co-ordinates of this vector relatively to

Teko Classes, Maths : Suhag R. Kariya (S. R. K. Sir), Bhopal Phone : 0 903 903 7779, 0 98930 58881.
          
the base p, q, r where p = 2a − 3b , q = a − 2b + c , r = −3a + b + 2c . Ans. (0, –7/5, 1/5)
      
3. If a and b are non-collinear vectors and A =(x + 4y) a + (2x + y + 1) b and B = (y – 2x + 2) a +
  
(2x – 3y – 1) b , find x and y such that 3 A = 2B . Ans. x = 2, y = –1
         
4. If vectors a, b,c be linearly independent, then show that :(i) a − 2b + 3c , − 2a + 3b − 4c , − b + 2c are
       
linearly dependent (ii) a − 3b + 2c , − 2a − 4b − c , 3a + 2b − c are linearly independent.
5. Given that î − ĵ , î − 2 ĵ are two vectors. Find a unit vector coplanar with these vectors and perpendicular
to the first vector î − ĵ . Find also the unit vector which is perpendicular to the plane of the two given
1
vectors. Do you thus obtain an orthonormal triad? Ans. ( î + ĵ ) ; k; Yes
es
2
  
6. If with reference to a right handed system of mutually perpendicular unit vectors î, ĵ, k̂ α = 3 i − j ,
           
β = 2 i + j − 3k express β in the form β = β1 + β 2 where β1 is parallel to α & β 2 is perpendicular to α .
 3 1  1 3 
Ans. β1 = i − j , β 2 = i + j − 3k
2 2 2 2

7. Prove that a vector r in space can be expressed linearly in terms of three non-coplanar, non-null
     
    [ r b c ] a + [ r c a] b + [ r a b] c
vectors a, b, c in the form r = 
[a b c ]
  
Note: Test Of Collinearity: Three points A,B,C with position vectors a, b, c respectively are collinear, if &
  
only if there exist scalars x , y, z not all zero simultaneously such that; xa + yb + zc = 0 , where x + y
+ z = 0.
   
Note: Test Of Coplanarity: Four points A, B, C, D with position vectors a, b, c, d respectively are coplanar
   
if and only if there exist scalars x, y, z, w not all zero simultaneously such that xa + yb + zc + wd = 0
where, x + y + z + w = 0.
        
Solved Example Show that the vectors 2a − b + 3c , a + b − 2c and a + b − 3c are non-coplanar vectors.
Solution. Let, the given vectors be coplanar.
Then one of the given vectors is expressible in terms of the other two.
       
Let

( ) ( )
2a − b + 3c = x a + b − 2c + y a + b − 3c , for some scalars x and y..
   
 
⇒ 2a − b + 3c = (x + y) a (x + y) b + (–2x – 3y) c
⇒ 2 = x + y, –1 = x + y and 3 = 2x – 3y.
Solving, first and third of these equations, we get x = 9 and y = –7.
Clearly, these values do not satisfy the thrid equation.
Hence, the given vectors are not coplanar.
           
Solved Example: Prove that four points 2a + 3b − c , a − 2b + 3c , 3a + 4b − 2c and a − 6b + 6c are coplanar..
Solution. Let the given four points be P, Q, R and S respectively. These points are coplanar if the vectors PQ ,
PR and PS are coplanar. These vectors are coplanar iff one of them can be expressed as a linear
combination of other two. So, let
PQ = x PR + y PS
          
 
( ) ( 
) 
⇒ − a − 5b + 4c = x a + b − c + y − a − 9b − 7c ⇒ − a − 5b + 4c = (x – y) a + (x – 9y) b + (–x + 7y) c
  
⇒x – y = –1, x – 9y = –5, –x + 7y = 4 [Equating coeff. of a, b, c on both sides]
1 1
Solving the first of these three equations, we get x = – ,y= .
2 2
These values also satisfy the third equation. Hence, the given four points are coplanar.
Self Practice Problems :
   
1. If, a, b, c, d are any four vectors in 3-dimensional space with the same initial point and such that
FREE Download Study Package from website: www.TekoClasses.com & www.MathsBySuhag.com

    
3a − 2b + c − 2d = 0 , show that the terminal A, B, C, D of these vectors are coplanar. Find the point at
which AC and BD meet. Find the ratio in which P divides AC and BD.
          
2. Show that the vector a − b + c , b − c − a and 2a − 3b − 4c are non-coplanar, where a, b, c , are any non-

page 38 of 77
coplanar vectors.
3. Find the value of λ for which the four points with position vectors − ĵ − k̂ , 4 î + 5 ĵ + λk̂ . 3 î + 9 ĵ + 4k̂ and
− 4 î + 4 ĵ + 4k̂ are coplanar.. Ans. λ=1
 
1 2 . Application Of Vectors: (a) Work done against a constant force F over a displacement s
  
is defined as W = F . s (b) The tangential velocity V of a body moving in a circle is giv en
   

Teko Classes, Maths : Suhag R. Kariya (S. R. K. Sir), Bhopal Phone : 0 903 903 7779, 0 98930 58881.
by V = w x r where r is the pv of the point P..

     
(c) The moment of F about ’O’ is defined as M = r x F where r is the pv of P wrt ’O’. The direction of M
  
is along the normal to the plane OPN such that r , F & M form a right handed system.
    
(d) Moment of the couple = ( r1 − r2 ) x F where r1 & r2 are pv’s of the point of the application of the forces
 
F & −F .

Solved Example: Forces of magnitudes 5 and 3 units acting in the directions 6 î + 2 ĵ + 3k̂ and 3 î + 2 ĵ + 6k̂
respectively act on a particle which is displaced from the point (2, 2, –1) to (4, 3, 1). Find the work
done by the forces.
 
Solution. Let F be the resultant force and d be the displacement vector. Then,
 (6 î + 2 ĵ + 3k̂ ) (3 î + 2 ĵ + 6k̂ ) 1
5
F = 36 + 4 + 9
+3
9 + 4 + 36
=
7
(39 î + 4 ĵ + 33k̂ )

and, d = ( 4 î + 3 ĵ + k̂ ) – (2 î + 2 ĵ − k̂ ) = 2 î + ĵ + 2k̂
  1
∴ Total work done = F . d = (39 î + 4 ĵ + 33k̂ ) . (2 î + ĵ + 2k̂ )
7
1 148
= (78 + 4 + 66) = units.
7 7
Self Practice Problems :1. A point describes a circle uniformly in the î , ĵ plane taking 12 seconds to
complete one revolution. If its initial position vector relative to the centre is î , and the rotation is from
î to ĵ , find the position vector at the end of 7 seconds. Also find the velocity vector.. Ans. 1 / 2
( ) (
ĵ − 3 î , p/12 î − 3 ĵ )
2. The force represented by 3 î + 2k̂ is acting through the point 5 î + 4 ĵ − 3k̂ . Find its moment about the
point î + 3 ĵ + k̂ . Ans. 2 î − 20 ĵ − 3k̂
3. Find the moment of the comple formed by the forces 5 î + k̂ and − 5 î − k̂ acting at the points
(9, –1, 2) and (3, –2, 1) respectively Ans. î − ĵ − 5k̂
Miscellaneous Solved Examples
Solved Example: Show that the points A, B, C with position vectors 2 î − ĵ + k̂ , î − 3 ĵ − 5k̂ and 3 î − 4 ĵ − 4k̂
respectively, are the vertices of a right angled triangle. Also find the remaining angles of the triangle.
Solution. We have, AB = Position vector of B – Position vector of A
= ( î − 3 ĵ − 5k̂ ) – (2 î − ĵ + k̂ ) = − î − 2 ĵ − 6k̂
BC = Position vector of C – Position vector of B
= (3 î − 4 ĵ − 4k̂ ) – ( î − 3 ĵ − 5k̂ ) = 2 î − ĵ + k̂
and, CA = Position vector of A – Position vector of C
FREE Download Study Package from website: www.TekoClasses.com & www.MathsBySuhag.com

= (2 î − ĵ + k̂ ) – (3 î − 4 ĵ − 4k̂ ) = − î + 3 ĵ + 5k̂

Since AB + BC + CA = ( − î − 2 ĵ − 6k̂ ) + (2 î − ĵ + k̂ ) + ( − î + 3 ĵ + 5k̂ ) = 0
So, A, B and C are the vertices of a triangle.

page 39 of 77
Now, BC . CA = (2 î − ĵ + k̂ ) . ( − î + 3 ĵ + 5k̂ ) = –2 – 3 + 5 = 0
π
⇒ BC ⊥ CA ⇒ ∠BCA = Hence, ABC is a right angled triangle.
2
Since a is the angle between the vectors AB and AC . Therefore
AB . AC ( − î − 2 ĵ − 6k̂ ) . ( î − 3 ĵ − 5k̂ )
cos A = =

Teko Classes, Maths : Suhag R. Kariya (S. R. K. Sir), Bhopal Phone : 0 903 903 7779, 0 98930 58881.
| AB | | AC | ( −1) + ( −2)2 + ( −6)2
2
12 + ( −3)2 + ( −5)2
−1 + 6 + 30 35 35 35
= = = A = cos–1
1 + 4 + 36 1 + 9 + 25 41 35 41 41
BA ( î + 2 ĵ + 6k̂ ) (2 î − ĵ + k̂ ) 6 2−2+6 6
cos B = = 2 2 2 2 2 2 =⇒ cos B = ⇒ B = cos–1
| BA | | BC | 1 + 2 + 6 2 + ( −1) + (1) 41 6 41 41
     
Solved Example: If a, b, c are three mutually perpendicular vectors of equal magnitude, prove that a + b + c
  
is equally inclined with vectors a, b and c .
     
Solution.: Let | a | = | b | = | c | = λ (say). Since a, b, c are mutually
     
perpendicular vectors, therefore a . b = b . c = c . a = 0 ..............(i)
   2
Now, a+b+c
           
= a . a + b . b + c . c + 2a . b + 2b . c + 2c . a
 2  
= | a | | + | b |2 + | c |2 [Using (i) ]
  
= 3λ2 [∵ | a | = | b | = | c | = λ]
  
∴ | a + b + c | = 3λ ..............(ii)
     
Suppose a + b + c makes angles θ 1, θ 2, θ 3 with a, b and c respectively. Then,
         
a . (a + b + c ) a.a+a.b+a.c
cosθ 1 =     =    
| a || a +b + c | |a|| a+b +c |
 
| a |2 |a| λ 1
=     =    = = [Using (ii)]
| a || a +b + c | |a+b+c| 3λ 3
 1 
∴ θ 1 cos–1  

 3
 1   1 
Similarly, θ 2 = cos–1   and θ = cos–1




 ∴ θ 1 = θ2 = θ 3.
 3  3 3

     
Hence, a + b + c is equally inclineded with a, b and c
Solved Example: Prove using vectors : If two medians of a triangle are equal, then it is isosceles.
Solution. : Let ABC be a triangle and let BE and CF be two equal medians. Taking A as the origin,
 
let the position vectors of B and C be b and c respectively. Then,
1  1  1  
P.V. of E = c and, P.V. of F = b ∴ BE = (c − 2b)
2 2 2
1  
CF = 2 (b − 2c )
Now, BE = CF ⇒ | BE | = | CF |
2
1  
1   2
⇒ | BE | = | CF | ⇒
2 2 (c − 2b)
= (b − 2c )
2 2
1   1      
⇒ | c − 2b |2 = | b − 2c |2 ⇒ | c − 2b |2 = | b − 2c |2
4 4
       
⇒ (c − 2b) . (c − 2b) = (b − 2c ) . (b − 2c )
           
⇒ c . c – 4b . c + 4b . b = b . b – 4b . c + 4c . c
       
⇒ | c |2 – 4b . c + 4 | b |2 = | b |2 – 4b . c + 4 | c |2
FREE Download Study Package from website: www.TekoClasses.com & www.MathsBySuhag.com

 
⇒ 3 | b |2 = 3 | c |2 ⇒ | b |2 = | c |2
⇒ AB = AC Hence, triangle ABC is an isosceles triangle.
Solved Example: Using vectors : Prove that cos (A + B) = cos A cos B – sin A sin B
Solution. Let OX and OY be the coordinate axes and let î and ĵ be unit vectors along OX and OY

page 40 of 77
respectively. Let ∠XOP = A and ∠XOQ = B. Drawn PL ⊥ OX and QM ⊥ OX.
Clearly angle between OP and OQ is A + B
In ∆OLP, OL = OP cos A and LP = OP sin A. Therefore OL = (OP cos A) î and LP = (OP sin A) − ĵ ( )
Now. OL + LP = OP
⇒ OP = OP [(cos A ) î – (sin A) ĵ ]

Teko Classes, Maths : Suhag R. Kariya (S. R. K. Sir), Bhopal Phone : 0 903 903 7779, 0 98930 58881.
In ∆OMQ, OM = OQ cos B and MQ = OQ sin B.
Therefore,
OM = (OQ cos B) î , MQ = (OQ sin B) ĵ
Now, OM + MQ = OQ
From (i) and (ii), we get
OP . OQ = OP [(cos A) î – (sin A) ĵ ] . OQ [(cos B) î + (sin B) ĵ ]
= OP . OQ [cos A cos B – sin A sin B]
But, OP . OQ = | OP | | OQ | cos (A + B) = OP . OQ cos (A + B)
∴ OP . OQ cos (A + B) = OP . OQ [cos A cos B – sin A sin B]
⇒ cos (A + B) = cos A cos B – sin A sin B
Solved Example: Prove that in any triangle ABC
(i) c2 = a2 + b2 – 2ab cos C (ii) c = bcosA + acosB.
Solution. (i) In ∆ABC, AB + BC + CA = 0
or, BC + CA = – AB ......(i)
Squaring both sides
( BC )2 + ( CA )2 + ( BC ). CA + ( AB )2
⇒ a2 + b2 + 2 ( BC . CA ) = c2 ⇒ c2 = a2 + b2 = 2 ab cos (π – C)
⇒ c2 = a2 + b2 – 2ab cosC
(ii) ( BC + CA ). AB = – AB . AB
BC . AB + CA . AB = – c2
– ac cosB – bc cos A = – c2
acosB + bcosA = c.
Solved Ex.: If D, E, F are the mid-points of the sides of a triangle ABC, prove by vector method that area of
1
∆DEF = (area of ∆ABC)
4
 
Solution. Taking A as the origin, let the position vectors of B and C be b and c respectively. Then, the
1   1  1 
position vectors of D, E and F are (b + c ) , c and b respectively..
2  2 2
1  1   −b
Now, DE = c – (b + c ) =
2 2 2
1  1   −c
and DF = b – ( (b + c ) =
2 2 2
 
1 1  − b × − c 
∴ Vector area of ∆DEF (DE × DF) =
2 2  2 2 
1   1  1 ( AB × AC)
= (b × c ) =  
8 4 2 
1 1
= (vector area of ∆ABC) Hence, area of ∆DEF = area of ∆ABC.
4 4
Solved Example: P, Q are the mid-points of the non-parallel sides BC and AD of a trapezium ABCD. Show
that ∆APD = ∆CQB.
 
Solution. Let AB = b and AD = d

Now DC is parallel to AB ⇒ there exists a acalar t sush that DC = t DB = t b
 
∴ AC = AD + DC = d + t b
1    1 
The position vectors of P and Q are (b + d + t b) and respectively..
2 2 d
FREE Download Study Package from website: www.TekoClasses.com & www.MathsBySuhag.com

Now 2∆ APD = AP × AD
1     1  
= (b + d + t b) × d = (1 + t) (b × d)
2 2
 1      

page 41 of 77
Also 2∆ CQB = CQ × CB =  d − (d + tb × [b − (d + t b)]
 2 
 1  1    
= − d − t b × [− d + (1 − t ) b] = − (1 − t ) d × b + t b × d
 2  2
1   1  
= (1 − t + 2t ) b × d = (1 + t ) b × d = 2∆ APD Hence the result.
2 2
         
Solved Example: Let u and v are unit vectors and w is a vector such that u × v + u = w and w × u = v then

Teko Classes, Maths : Suhag R. Kariya (S. R. K. Sir), Bhopal Phone : 0 903 903 7779, 0 98930 58881.
  
find the value of [u v w ] .
      
Solution. Given u × v + u = w and w × u = v
      
⇒ (u × v + u ) × u= w × u ⇒  (u × v ) × u + u × u = v   (as, w × u = v )
⇒ (u . u) v − (v . u) u + u × u = v (using u . u = 1 and u × u = 0, since unit vector)
     
⇒ v − ( v . u) u = v ⇒ (u . v ) u = 0
  
⇒ u.v = 0 (as; u ≠ 0) .............(i)
  
⇒ u . (v × w )
       
= u . ( v × (u × v + u)) (given w = u × v + u)
              
= u . ( v × (u × v ) + v × u) = u . ((v . v ) u − ( v . u) v + v × u)
     
= u . (| v |2 u − 0 + v × u) (as; u . v = 0 from (i))
          
= | v |2 (u . u) – u . ( v × u) = | v |2 | u |2 – 0 (as, [u v u] = 0)
 
=1 (as; | u | = | v | = 1) ∴ [u v w ] = 1
Sol. Ex.: In any triangle, show that the perpendicular bisectors of the sides are concurrent.
Solution. Let ABC be the triangle and D, E and F are respectively middle points of sides BC, CA and AB.
Let the perpendicular of D and E meet at O join OF. We are required to prove that OF is ⊥ to AB. Let the
  
position vectors of A, B, C with O as origin of reference be a , b and c respectively..
1   1   1  
∴ OD = ( b + c ), OE = ( c + a ) and OF = (a + b )
2 2 2
     
Also BC = c – b , CA = a – c and AB = b – a
1    
Since OD ⊥ BC, (b + c ) . (c – b ) = 0
2
⇒ b2 = c 2 ............(i)
1    
Similarly (c + a) . (a + c ) = 0
2
⇒ a2 = c 2 ............(ii)
from (i) and (ii) we have a2 – b2 = 0
    1    
⇒ (a + b ) . (b + a ) = 0 ⇒ (b + a ) . (b – a ) = 0
2
Solved Example: A, B, C, D are four points in space. using vector methods, prove that
2 2 2 2 2 2
AC + BD + AD + BC ≥ AB + CD what is the implication of the sign of equaility.
   
Solution.: Let the position vector of A, B, C, D be a , b , c and d respectively then
               
( ) ( ) (
AC2 + BD2 + AD2 + BC2 = (c − a ) . (c − a ) + d − b . d − b + d − a . d − a + c − b . c − b ) ( ) ( ) (  2
)
  
  2         2  
= | c |2 + | a | – 2 a . c + | d |2 + | b |2 – 2 d . b + | d |2 + | a | – 2 a . d + | c | + | b |2 – 2b . c
 2     2    2
  2 
= | a | + | b |2 – 2 a . b + | c | + | d |2 – 2 c . d + | a | + | b |2 + | c | + | d |2
           
+ 2 a . b + 2c . d – 2a . c – 2b . d – 2a . d – 2b . c
           
( ) ( ) ( ) ( ) (
= a − b . a − b + c − d . c − d + a − b − c − d ≥ AB2 + CD2 )
       
( ) (
= AB2 + CD2 + a + b − c − d . a + b − c − d ≤ AB2 + CD2 )
∴ AC2 + BD2 + AD2 + BC2 ≥ AB2 + CD2
   
for the sign of equality to hold, a + b − c − d = 0
   
a −c = d−b
⇒ AC and BD are collinear the four points A, B, C, D are collinear
SHORT REVISION
FREE Download Study Package from website: www.TekoClasses.com & www.MathsBySuhag.com

1. DEFINITIONS: A VECTOR may be described as a quantity having both magnitude & direction. A

vector is generally represented by a directed line segment, say AB . A is called the initial point & B is
→ →
called the terminal point. The magnitude of vector AB is expressed by  AB .

page 42 of 77
ZERO VECTOR a vector of zero magnitude i.e.which has the same initial & terminal point, is called a
ZERO VECTOR. It is denoted by O.
 
UNIT VECTOR a vector of unit magnitude
 in direction of a vector a is called unit vector along a and is
a
denoted by â symbolically â=  .
a

Teko Classes, Maths : Suhag R. Kariya (S. R. K. Sir), Bhopal Phone : 0 903 903 7779, 0 98930 58881.
EQUAL VECTORS two vectors are said to be equal if they have the same magnitude, direction & represent
the same physical quantity.
COLLINEAR VECTORS two vectors are said to be collinear if their directed line segments are parallel
disregards to their direction. Collinear vectors are also called PARALLEL VECTORS. If they have the same
direction they are named as like vectors otherwise unlike vectors.
   
Simbolically, two non zero vectors a and b are collinear if and only if, a=Kb ,
where K ∈ R
COPLANAR VECTORS a given number of vectors are called coplanar if their line segments are all parallel
to the same plane. Note that “TWO VECTORS ARE ALWAYS COPLANAR”.

POSITION VECTOR let O be a fixed origin, then the position vector of a point P is the vector OP . If

a &b
→  & 
position vectors of two point A and B, then ,
AB = b − a = pv of B − pv of A .
  → →
2. VECTOR ADDITION : If two vectors a & b are represented by OA&OB , then their
  →
sum a + b is a vector represented by OC , where OC is the diagonal of the parallelogram OACB.
         
 a + b = b + a (commutative)  (a + b) + c = a + ( b + c) (associativity)
         
 a + 0=a =0 + a  a + ( − a ) = 0 = ( − a ) +a
3. MULTIPLICATION OF VECTOR BY SCALARS :
  
If a is a vector & m is a scalar, then m a is a vector parallel to a whose modulus is m times that of
  
a . This multiplication is called SCALAR MULTIPLICATION. If a & b are vectors & m, n are scalars, then:
     
m(a ) = (a )m = ma m( na ) = n( ma ) = ( mn )a
      
( m + n )a = ma + na m(a + b) = ma + mb
4. SECTION FORMULA :
 
If a & b are the position vectors of two points A & B then the p.v. of a point which divides AB in the
   
 na + mb a+b
ratio m : n is given by : r = . Note p.v. of mid point of AB = .
m+n 2
5. DIRECTION COSINES :

Let a = a1î + a 2 ĵ + a 3k̂ the angles which this vector makes with the +ve directions OX,OY & OZ are
called DIRECTION ANGLES & their cosines are called the DIRECTION COSINES .
a a a
cos α= 1 , cos β= 2 , cos Γ = 3 . Note that, cos² α + cos² β + cos² Γ = 1
a a a
6. VECTOR EQUATION OF A LINE :
     
Parametric vector equation of a line passing through two point A(a ) & B(b) is given by, r = a + t (b − a )
 
where t is a parameter. If the line passes through the point A(a ) & is parallel to the vector b then its
  
equation is, r = a + t b
FREE Download Study Package from website: www.TekoClasses.com & www.MathsBySuhag.com

page 43 of 77
Teko Classes, Maths : Suhag R. Kariya (S. R. K. Sir), Bhopal Phone : 0 903 903 7779, 0 98930 58881.
     
Note that the equations of the bisectors of the angles between the lines r = a + λ b & r = a + µ c is :
 
( ) 
r = a + t b + c & r = a + p c − b .( )

  
7. TEST OF COLLINEARITY : Three points A,B,C with position vectors a , b, c respectively are
  
collinear, if & only if there exist scalars x , y, z not all zero simultaneously such that ; xa + yb + zc = 0 ,
where x + y + z = 0.
8. SCALAR PRODUCT OF TWO VECTORS :
  
 a.b = a b cos θ(0 ≤ θ ≤ π) ,
 
note that if θ is acute then a.b > 0 & if θ is obtuse then a.b < 0
  2          
 a.a = a =a 2 ,a.b=b.a (commutative)  a . (b + c) = a . b + a . c (distributive)
    
 a.b = 0 ⇔ a ⊥ b (a ≠ 0 b ≠ 0)

  a.b
 î . î = ˆj. ˆj = k̂ . k̂ = 1 ; î . ˆj = ˆj. k̂ = k̂ . î = 0  projection of a on b =  .
b
  a ⋅ b  
  
    a ⋅ b 
Note: That vector component of a along b =    b and perpendicular to b = a –    b .
 b2     b2 
  a .b
 the angle φ between a & b is given by cos φ =   0≤φ≤π
ab
  
 if a = a1î + a 2 ĵ + a 3k̂ & b = b1î + b 2 ĵ + b 3k̂ then a.b = a1b1 + a2b2 + a3b3
 
a = a12 + a 2 2 + a 32 , b = b12 + b 2 2 + b 32
   
Note : (i) Maximum value of a . b =  a   b 
     
(ii) Minimum values of a . b = a . b = −  a   b 
(iii)
   
( ) ( ) ( )

Any vector a can be written as , a = a . i i + a . j j + a . k k .
 
  a b
(iv) A vector in the direction of the bisector of the angle between the two vectors a & b is  +  . Hence
a b
 
( )
bisector of the angle between the two vectors a & b is λ a + b , where λ ∈ R+. Bisector of the
 
( )
exterior angle between a & b is λ a − b , λ ∈ R+ .
9. VECTOR PRODUCT OF TWO VECTORS :
     
(i)  If a & b are two vectors & θ is the angle between them then a × b = a b sin θn ,
     
where n is the unit vector perpendicular to both a & b such that a ,b&n forms a
right handed screw system .
   
    2  2  2   2 a .a a .b
(ii)  Lagranges Identity : for any two vectors a & b ;(a x b) = a b − (a . b) =    
a .b b.b
FREE Download Study Package from website: www.TekoClasses.com & www.MathsBySuhag.com

(iii)  Formulation of vector product in terms of scalar product:


  
The vector product a x b is the vector c , such that
      
(i) | c | = a 2 b 2 − (a ⋅ b) 2 (ii) c ⋅ a = 0; c ⋅ b =0 and

page 44 of 77
  
(iii) a , b, c form a right handed system
       
(iv)  a × b = 0 ⇔ a & b are parallel (collinear) (a ≠ 0, b ≠ 0) i.e. a = Kb , where K is a scalar..
 
 a × b ≠ b × a (not commutative)
     
 (ma )× b = a × (mb ) = m(a × b) where m is a scalar .
    
 a × ( b + c ) = ( a × b ) + (a × c )

Teko Classes, Maths : Suhag R. Kariya (S. R. K. Sir), Bhopal Phone : 0 903 903 7779, 0 98930 58881.
(distributive)
 î × î = j × j = k̂ × k̂ = 0
ˆ ˆ  î × ˆj = k̂ , ˆj × k̂ = î , k̂ × î = ˆj
î ĵ k̂
   
(v)  If a = a1î + a 2 ĵ + a 3k̂ & b = b1î + b 2 ĵ + b 3k̂ then a × b = a1 a 2 a3
b3b1 b 2
 
(vi)  Geometrically a × b = area of the parallelogram whose two adjacent sides are
 
represented by a & b .
 
  a×b
(vii)  Unit vector perpendicular to the plane of a & b is n̂ = ±  
a×b

 
 
r a×b
A vector of magnitude ‘r’ & perpendicular to the palne of a & b is ±  
( )
a×b
 
  a×b
 If θ is the angle between a & b then sin θ =  
a b
(viii) Vector area
 
 If a,b&c are the pv’s of 3 points A, B & C then the vector area of triangle ABC =
[
1  
]
     
a x b + b x c + cxa . The points A, B & C are collinear if a x b + b x c + cx a = 0
2
  1  
 Area of any quadrilateral whose diagonal vectors are d1 &d 2 is given by d1 x d 2
2
10. SHORTEST DISTANCE BETWEEN TWO LINES :
If two lines in space intersect at a point, then obviously the shortest distance between them is zero. Lines
which do not intersect & are also not parallel are called SKEW LINES. For Skew lines the direction of
the shortest distance would be perpendicular to both the lines. The magnitude of the shortest distance

vector would be equal to that of the projection of AB along the direction of the line of shortest distance,
→ → → → →  
 
LM is parallel to p x q i.e. LM = Pr ojection of AB on LM = Pr ojection of AB on px q
→      
AB . (p x q) ( b − a ) . ( p x q)
=   =  
pxq pxq
 
1. The two lines directed along p & q will intersect only if shortest distance = 0 i.e.
( ) [( ) ]
          
(b − a ).( px q) = 0 i.e. b − a lies in the plane containing p & q . ⇒ b − a p q = 0 .
  
      b x(a 2 − a 1 )
2. If two lines are given by r1 = a 1 + Kb & r2 = a 2 + Kb i.e. they are parallel then , d = 
b
11. SCALAR TRIPLE PRODUCT / BOX PRODUCT / MIXED PRODUCT :
  
 The scalar triple product of three vectors a ,b&c is defined as :
      
a xb.c = a b c sin θ cos φ where θ is the angle between a & b & φ is the angle between a × b & c .

FREE Download Study Package from website: www.TekoClasses.com & www.MathsBySuhag.com


It is also defined as [a bc] , spelled as box product .
 Scalar triple product geometrically represents the volume of the parallelopiped whose three couterminous
  
edges are represented by a,b&c i.e.V=[a bc]

page 45 of 77
 In a scalar triple product the position of dot & cross can be interchanged i.e.
         
a.( bxc )=(a x b).c OR[a b c]=[bca]=[ca b]
       
 a . (b x c) = − a .( cx b) i. e. [ a b c ] = − [ a c b ]
a a a
1 2 3
   
 If a = a1î + a 2 ĵ + a 3k̂ ; b = b1î + b 2 ĵ + b 3k̂ & c = c1î + c 2 ĵ + c3k̂ then [a b c] = b1 b 2 b 3 .

Teko Classes, Maths : Suhag R. Kariya (S. R. K. Sir), Bhopal Phone : 0 903 903 7779, 0 98930 58881.
c1 c2 c3
           
In general , if a = a 1 l + a 2 m + a 3 n ; b = b1 l + b 2 m + b 3 n & c = c1 l + c2 m + c3 n
a1 a 2 a 3
[ ]

[ ]
   
then a b c = b1 b 2 b 3 l m n ; where  , m & n are non coplanar vectors .
c1 c2 c3
   
 If a , b , c are coplanar ⇔ [ a b c ] = 0 .

 Scalar product of three vectors, two of which are equal or parallel is 0 i.e. [ a b c ] = 0 ,
   
Note : If a , b , c are non − coplanar then [ a b c ] > 0 for right handed system &

[ a b c ] < 0 for left handed system .
      
 [i j k] = 1  [ K a b c ] = K[ a b c ]  [(a + b) c d ] = [ a c d ] + [ b c d ]
  
 The volume of the tetrahedron OABC with O as origin & the pv’s of A, B and C being a , b & c
1 
respectively is given by V = [a b c]
6
   
 The positon vector of the centroid of a tetrahedron if the pv’s of its angular vertices are a , b , c & d are
1    
given by [a + b + c + d ] .
4
Note that this is also the point of concurrency of the lines joining the vertices to the centroids of the
opposite faces and is also called the centre of the tetrahedron. In case the tetrahedron is regular it is
equidistant from the vertices and the four faces of the tetrahedron .
[
     
Remember that : a − b b − c c − a = 0 & ] [
     
a + b b+ c c+a = 2 a b c . ]

[ ]
  
*12. VECTOR TRIPLE PRODUCT : Let a ,b,c be any three vectors, then the expression
  
a × ( b × c) is a vector & is called a vector triple product .
  
GEOMETRICAL INTERPRETATION OF a × ( b × c)
  
Consider the expression a × ( b × c) which itself is a vector, since it is a cross product of two vectors
          
a & ( b x c ) . Now a x ( b x c ) is a vector perpendicular to the plane containing a & ( b x c ) but b x c
    
is a vector perpendicular to the plane b&c , therefore a x ( b x c ) is a vector lies in the plane of
       
b&c and perpendicular to a . Hence we can express a x ( b x c ) in terms of b & c
    
i.e. a x ( b x c ) = xb + yc where x & y are scalars .
                 
 a x ( b x c ) = (a . c) b − (a . b) c  (a x b) x c = (a . c) b − (b . c) a
     
 (a x b) x c ≠ a x (b x c)
13. LINEAR COMBINATIONS / Linearly Independence and Dependence of Vectors :
      
Given a finite set of vectors a , b , c ,...... then the vector r =xa + yb+ zc+........ is called a linear
  
combination of a , b , c ,...... for any x, y, z ...... ∈ R. We have the following results :
 
(a) FUNDAMENTALTHEOREM IN PLANE : Let a,b be non zero , non collinear vectors . Then any vector r
FREE Download Study Package from website: www.TekoClasses.com & www.MathsBySuhag.com

 
coplanar wit h a,b can be expressed uniquely as a linear combinatio n of a,b
  
i.e. There exist some unique x,y ∈ R such that xa + yb=r .


page 46 of 77
(b) FUNDAMENTAL THEOREM IN SPACE : Let a,b,c be non−zero, non−coplanar vectors in space. Then
 
any vector r , can be uniquily expressed as a linear combination of a,b,c i.e. There exist some unique
   
x,y ∈ R such that xa + yb+ zc = r .
  
(c) If x1,x 2 ,......x n are n non zero vectors, & k1, k2, .....kn are n scalars & if the linear combination
     
k1x1 + k 2 x 2 + ........k n x n = 0 ⇒ k1 = 0,k 2 = 0.....k n = 0 then we say that vectors x1,x 2 ,......x n

Teko Classes, Maths : Suhag R. Kariya (S. R. K. Sir), Bhopal Phone : 0 903 903 7779, 0 98930 58881.
are LINEARLY INDEPENDENT VECTORS .
  
(d) If x1,x 2 ,......x n are not LINEARLY INDEPENDENT then they are said to be LINEARLY DEPENDENT
  
vectors . i.e. if k1x1 + k 2 x 2 +........+ k n x n = 0 & if there exists at least one kr ≠ 0 then
  
x1,x 2 ,......x n are said to be LINEARLY DEPENDENT .
  
Note : If a = 3i + 2j + 5k then a is expressed as a LINEAR COMBINATION of vectors î , ˆj, k̂ . Also , a ,
î , ˆj, k̂ form a linearly dependent set of vectors. In general , every set of four vectors is a linearly
dependent system.
 î , ˆj , k̂ are LINEARLY INDEPENDENT set of vectors. For K1î + K 2 ĵ + K 3k̂ = 0 ⇒ K1 = 0 = K2 = K3.
     
 Two vectors a &b are linearly dependent ⇒ a is parallel to b i.e. a xb= 0 ⇒ linear dependence of
     
a &b . Conversely if a xb≠0 then a &b are linearly independent .
     
 If three vectors a ,b,c are linearly dependent, then they are coplanar i.e. [ a , b, c ] = 0 , conversely, if
  
[ a , b, c ] ≠ 0 , then the vectors are linearly independent.
14. COPLANARITY OF VECTORS :
   
Four points A, B, C, D with position vectors a ,b,c,d respectively are coplanar if and only if there exist
   
scalars x, y, z, w not all zero simultaneously such that xa+ yb+zc+wd =0 where, x + y + z + w = 0.
15. RECIPROCAL SYSTEM OF VECTORS :
           
If a ,b,c & a ' ,b ' ,c ' are two sets of non coplanar vectors such that a.a'=b .b '=c .c '=1 then the
two systems are called Reciprocal System of vectors.
     
bx c cx a axb
a'=    ; b' =    ; c' =
[ ] [ ] [ ]
Note : 
abc abc abc

16. EQUATION OF A PLANE :


    
(a) The equation ( r − r0 ).n =0 represents a plane containing the point with p.v. r0 wheren is a
 
vector normal to the plane . r . n = d is the general equation of a plane.
(b) Angle between the 2 planes is the angle between 2 normals drawn to the planes and the angle between
a line and a plane is the compliment of the angle between the line and the normal to the plane.
17. APPLICATION OF VECTORS :

(a) Work done against a constant force F over a
  
displacement s is defined as W=F.s

(b) The tangential velocity V of a body moving in a
   
circle is given by V = w × r where r is the pv of the
point P.
    
(c) The moment of F about ’O’ is defined as M = r × Fwherer

is the pv of P wrt ’O’. The direction of M is along the
 
normal to the plane OPN such that r ,F&M form a
right handed system.
    
(d) Moment of the couple = ( r1 − r2 ) × F where r1 & r2 are pv’s of the
 
point of the application of the forces F&− F.
FREE Download Study Package from website: www.TekoClasses.com & www.MathsBySuhag.com

3 -D COORDINATE GEOMETRY USEFUL RESULTS


A General :
(1) Distance (d) between two points (x1 , y1 , z1) and (x2 , y2 , z2)

page 47 of 77
d = ( x 2 − x1 ) 2 + ( y 2 − y1 ) 2 + (z 2 − z1 ) 2
(2) Section Fomula
m 2 x1 + m1 x 2 m 2 y1 + m1 y 2 m 2 z1 + m1 z 2
x= ; y= ; z= m1 + m 2
m1 + m 2 m1 + m 2
( For external division take –ve sign )
Direction Cosine and direction ratio's of a line

Teko Classes, Maths : Suhag R. Kariya (S. R. K. Sir), Bhopal Phone : 0 903 903 7779, 0 98930 58881.
(3) Direction cosine of a line has the same meaning as d.c's of a vector.
(a) Any three numbers a, b, c proportional to the direction cosines are called the direction ratios i.e.
l m n 1
= = =±
a b c a + b2 + c2
2
same sign either +ve or –ve should be taken through out.
note that d.r's of a line joining x1 , y1 , z1 and x2 , y2 , z2 are proportional to x2 – x1 , y2 – y1 and z2 – z1
(b) If θ is the angle between the two lines whose d.c's are l1 , m1 , n1 and l2 , m2 , n2
cosθ = l1 l2 + m1 m2 + n1 n2
hence if lines are perpendicular then l1 l2 + m1 m2 + n1 n2 = 0
l1 m1 n1
if lines are parallel then = =
l2 m 2 n 2
l1 m1 n1
note that if three lines are coplanar then l2 m 2 n 2 = 0
l3 m 3 n 3
(4) Projection of the join of two points on a line with d.c's l, m, n are
l (x2 – x1) + m(y2 – y1) + n(z2 – z1)
B PLANE: (i) General equation of degree one in x, y, z i.e. ax + by + cz + d = 0 represents a plane.
(ii) Equation of a plane passing through (x1 , y1 , z1) is
a (x – x1) + b (y – y1) + c (z – z1) = 0
where a, b, c are the direction ratios of the normal to the plane.
(iii) Equation of a plane if its intercepts on the co-ordinate axes are x1 , y1 , z1 is
x y z
+ + = 1.
x1 y1 z1
(iv) Equation of a plane if the length of the perpendicular from the origin on the plane is p and d.c's of the
perpendicular as l , m, , n is l x + m y + n z = p
(v) Parallel and perpendicular planes – Two planes
a1 x + b1 y + c1z + d1 = 0 and a2x + b2y + c2z + d2 = 0 are
perpendicular if a1 a2 + b1 b2 + c1 c2 = 0
a1 b1 c1
parallel if = =
a 2 b 2 c 2 and
a1 b1 c1 d1
coincident if = = =
a 2 b 2 c2 d 2
(vi) Angle between a plane and a line is the compliment of the angle between the normal to the plane and the
    
Line : r = a + λ b  b .n
line . If Plane : r . n = d  then cos(90 − θ) = sin θ =   .
 | b | .| n |
where θ is the angle between the line and normal to the plane.
(vii) Length of the perpendicular from a point (x1 , y1 , z1) to a plane ax + by + cz + d = 0 is
ax1 + by1 + cz1 + d
p=
a 2 + b 2 + c2
(viii) Distance between two parallel planes ax + by + cz + d1 = 0 and ax + by + cz + d2 = 0 is
d1 − d 2
FREE Download Study Package from website: www.TekoClasses.com & www.MathsBySuhag.com

a 2 + b2 + c2
(ix) Planes bisecting the angle between two planes
a1x + b1y + c1z + d1 = 0 and a2 + b2y + c2z + d2 = 0 is given by

page 48 of 77
a1x + b1y + c1z + d1 a 2 x + b 2 y + c2z + d 2
= ±
a12 + b12 + c12 a 22 + b 22 + c 22
Of these two bisecting planes , one bisects the acute and the other obtuse angle between the given
planes.
(x) Equation of a plane through the intersection of two planes P1 and P2 is given by P1 + λP2 = 0

Teko Classes, Maths : Suhag R. Kariya (S. R. K. Sir), Bhopal Phone : 0 903 903 7779, 0 98930 58881.
C STRAIGHT LINE IN SPACE
(i) Equation of a line through A (x1 , y1 , z1) and having direction cosines l ,m , n are
x − x1 y − y1 z − z1
= =
l m n
and the lines through (x1 , y1 ,z1) and (x2 , y2 ,z2)
x − x1 y − y1 z − z1
= =
x 2 − x1 y 2 − y1 z 2 − z1
(ii) Intersection of two planes a1x + b1y + c1z + d1 = 0 and a2x + b2y + c2z + d2 = 0
together represent the unsymmetrical form of the straight line.
x − x1 y − y1 z − z1
(iii) General equation of the plane containing the line = = is
l m n
A (x – x1) + B(y – y1) + c (z – z1) = 0 where Al + bm + cn = 0 .
LINE OF GREATEST SLOPE
AB is the line of intersection of G-plane and H is the horizontal
plane. Line of greatest slope on a given plane, drawn through a
given point on the plane, is the line through the point 'P'
perpendicular to the line of intersetion of the given plane with
any horizontal plane.


EXERCISE–1 
  
Q.1 If a&b are no n co llinear vecto rs such that , p = ( x + 4 y)a + (2 x + y + 1)b &
    
q = ( y − 2 x + 2)a + (2 x − 3y − 1)b ,find x & y such that 3p = 2q .
      
Q.2 (a) Show that the points a − 2b + 3c;2a + 3b − 4c & − 7 b + 10c are collinear .
(b) Prove that the points A = (1,2,3), B (3,4,7), C (−3,−2,−5) are collinear & find the ratio in which
B divides AC.
→ →
Q.3 Points X & Y are taken on the sides QR & RS , respectively of a parallelogram PQRS, so that QX = 4 XR

→ →  21  →
& RY = 4 YS . The line XY cuts the line PR at Z . Prove that PZ =   PR .
 25 
Q.4 Find out whether the following pairs of lines are parallel, non-parallel & intersecting, or non-parallel &
non-intersecting.
  
(
r1 = i + j + 2 k + λ 3 i − 2 j + 4 k )
  
(
r1 = i − j + 3 k + λ i − j + k )
( ) ( )
(i)  (ii) 
r2 = 2 i + j + 3 k + µ − 6 i + 4 j − 8 k r2 = 2 i + 4 j + 6 k + µ 2 i + j + 3 k
  
(
r1 = i + k + λ i + 3 j + 4 k )
(iii) 
(
r2 = 2 i + 3 j + µ 4 i − j + k )
Q.5 Let OACB be paralelogram with O at the origin & OC a diagonal. Let D be the mid point of OA.
Using vector method prove that BD & CO intersect in the same ratio. Determine this ratio.
Q.6 A line EF drawn parallel to the base BC of a ∆ ABC meets AB & AC in F & E respectively. BE & CF
meet in L. Use vectors to show that AL bisects BC.

Q.7 ‘O’is the origin of vectors and A is a fixed point on the circle of radius‘a’with centre O. The vector OA
 →  
is denoted by a . A variable point ‘P’ lies on the tangent at A & OP = r . Show that a.r = a 2 . Hence
if P ≡ (x,y) & A ≡ (x1,y1) deduce the equation of tangent at A to this circle.
FREE Download Study Package from website: www.TekoClasses.com & www.MathsBySuhag.com

Q.8 (a) By vector method prove that the quadrilateral whose diagonals bisect each other at right angles
is a rhombous.
(b) By vector method prove that the right bisectors of the sides of a triangle are concurrent.
    
The resultant of two vectors a&b is perpendicular to a . If b = 2 a show that the resultant of

page 49 of 77
Q.9
  
2a & b is perpendicular to b .
   
Q.10 a , b, c and d are the position vectors of the points A ≡ (x, y, z) ; B ≡ (y, – 2z, 3x) ; C ≡ (2z, 3x, – y)
 π

( )
^
( ) ( )
and D≡(1,–1, 2) respectively. If | a | = 2 3 ; a b = (a^c ) ; a ^d = and a ^ĵ is obtuse, then find x, y,, z.
  2  

Teko Classes, Maths : Suhag R. Kariya (S. R. K. Sir), Bhopal Phone : 0 903 903 7779, 0 98930 58881.
Q.11 If r and s are non zero constant vectors and the scalar b is chosen such that r + b s is minimum, then
   
show that the value of b s 2 + | r + b s |2 is equal to | r |2 .
Q.12 Use vectors to prove that the diagonals of a trapezium having equal non parallel sides are equal &
conversely.
Q.13(a) Find a unit vector â which makes an angle (π/4) with axis of z & is such that â + i + j is a unit vector.
  2   2
 a b   a−b 
(b) Prove that   −   =    
 2 b2   | a | | b | 
a        
Q.14 Given four non zero vectors a , b , c and d . The vectors a,b&c are coplanar but not collinear pair by
   ∧ ∧ π ∧ ∧
pair and vector d is not coplanar with vectors a,b&c and (a b) = (b c) = , (da) = α, (d b) = β then
∧ 3
prove that ( d c) = cos −1 (cos β − cos α ) .
Q.15 (a) Use vectors to find the acute angle between the diagonals of a cube .
(b) Prove cosine & projection rule in a triangle by using dot product .
Q.16 In the plane of a triangle ABC, squares ACXY, BCWZ are described , in the order given, externally to
→  →  → →
 
the triangle on AC & BC respectively. Given that CX = b , CA = a , CW = x , CB = y . Prove that
    → →
a . y + x . b = 0 . Deduce that AW . BX = 0 .
Q.17 A ∆ OAB is right angled at O ; squares OALM & OBPQ are constructed on the sides OA and OB
externally. Show that the lines AP & BL intersect on the altitude through 'O' .
  
Q.18 Given that u = î − 2ˆj + 3k̂ ; v = 2î + ˆj + 4k̂ ; w = î + 3ˆj + 3k̂ and
      
(u ·R − 10)î + ( v · R − 20)ˆj + ( w · R − 20)k̂ = 0. Find the unknown vector R .
           
Q.19 If O is origin of reference, point A( a ) ; B( b ) ; C( c ) ; D(a + b) ; E(b + c) ; F(c + a ); G (a + b + c) where
  
a = a1î + a 2 ĵ + a 3k̂ ; b = b1î + b 2 ĵ + b 3k̂ and c = c1î + c 2 ĵ + c3k̂ then prove that these points are
vertices of a cube having length of its edge equal to unity provided the matrix.
a1 a 2 a 3 
b b b 
 1 2 3  is orghogonal. Also find the length XY such that X is the point of intersection of CM and
 1
c c 2 c 3
GP ; Y is the point of intersection of OQ and DN where P, Q, M, N are respectively the midpoint of
sides CF, BD, GF and OB.
        
Q.20 (a) If a + b + c = 0 , show that a xb=bxc=cxa . Deduce the Sine rule for a ∆ ABC.
(b) Find the minimum area of the triangle whose vertices are A(–1, 1, 2); B(1, 2, 3) and C(t, 1, 1)
where t is a real number.
Q.21 (a) Determine vector of magnitude 9 which is perpendicular to both the vectors :
4î − ˆj + 3k̂ & − 2î + ˆj − 2k̂  π
(b) A triangle has vertices (1, 1, 1) ; (2, 2, 2), (1, 1, y) and has the area equal to csc   sq. units.
Find the value of y. 4
Q.22 The internal bisectors of the angles of a triangle ABC meet the opposite sides in D, E, F ; use vectors to
prove that the area of the triangle DEF is given by
(2abc) ∆
where ∆ is the area of the triangle.
(a + b ) ( b + c) (c + a )
 
FREE Download Study Package from website: www.TekoClasses.com & www.MathsBySuhag.com

 
Q.23 If a,b,c,d are position vectors of the vertices of a cyclic quadrilateral ABCD prove that :
   
    
 
a x b + bxd + dxa b x c + cxd + d x b
    +     =0
(b − a ) . (d − a ) ( b − c) . ( d − c)

page 50 of 77
Q.24 T h e l e n g t h o f t h e e d g e o − ABC is 'a' . Point E and F are taken on the edges
f t h e r e g u l a r t e t r a h e d r o n D

→ →
AD and BD respectively such that E divides DA and F divides BD in the ratio 2:1 each . Then find the
area of triangle CEF.
  1 3    
Q.25 Let a = 3 î − ˆj and b = î + ĵ and x = a + (q 2 − 3)b , y = −p a + qb . If x ⊥ y , then express p
2 2

Teko Classes, Maths : Suhag R. Kariya (S. R. K. Sir), Bhopal Phone : 0 903 903 7779, 0 98930 58881.
as a function of q, say p = f (q), (p ≠ 0 & q ≠ 0) and find the intervals of monotonicity of f (q).
EXERCISE–2
    1   
Q.1 A (a ) ; B(b) ; C(c) are the vertices of the triangle ABC such that a = (2î − r − 7k̂ ) ; b = 3 r + ˆj − 4k̂ ;
     2 
c = 22î − 11 j − 9 r
ˆ . A vector p = 2 j − k̂
ˆ is such that ( r + p ) is parallel to î and ( r − 2î ) is parallel to
  
p . Show that there exists a point D (d ) on the line AB with d = 2 t î + (1 − 2 t )ˆj + ( t − 4)k̂ . Also find the
shortest distance C from AB.
Q.2 The position vectors of the points A, B, C are respectively (1, 1, 1) ; (1, −1, 2) ; (0, 2, −1). Find a unit
vector parallel to the plane determined by ABC & perpendicular to the vector (1, 0, 1) .
(a 1 − a ) 2 (a 1 − b ) 2 (a 1 − c) 2
 
Let (b1 − a ) (b1 − b) (b1 − c) = 0 and if the vectors α = î + aˆj + a 2 k̂ ; β = î + bˆj + b 2 k̂ ;
2 2 2
Q.3
(c1 − a ) 2 (c1 − b) 2 (c1 − c) 2
  
γ = î + cˆj + c 2 k̂ are non coplanar, show that the vectors α1 = î + a1 ĵ + a1 k̂;β1 = î + b1ˆj + b1 k̂ and
2 2

γ1 = î + c1 ĵ + c12 k̂ are coplaner..
Q.4 Given non zero number x1, x2, x3 ; y1, y2, y3 and z1, z2 and z3 such that xi > 0 and yi < 0 for all i = 1, 2, 3.
(i) Can the given numbers satisfy
x1 x 2 x 3 x1x 2 + y1y 2 + z1z 2 = 0

y1 y 2 y 3 = 0 and x 2 x 3 + y 2 y 3 + z 2 z 3 = 0
z1 z 2 z 3 x 3 x1 + y3 y1 + z 3z1 = 0
(ii) If P = (x1, x2, x3) ; Q (y1, y2, y3) and O (0, 0, 0) can the triangle POQ be a right angled triangle?
Q.5 The pv's of the four angular points of a tetrahedron are : A j + 2 k ; B 3 i + k ; C 4 i + 3 j + 6 k ( ) ( ) ( )
(
& D 2 i + 3 j + 2 k . Find : )
(i) the perpendicular distance from A to the line BC. (ii) the volume of the tetrahedron ABCD.
(iii) the perpendicular distance from D to the plane ABC.
(iv) the shortest distance between the lines AB & CD.

Q.6 The length of an edge of a cube ABCDA1B1C1D1 is equal to unity. A point E taken on the edge AA 1 is
→ 1 → → 1
such that AE = . A point F is taken on the edge BC such that BF = . If O1 is the centre of
3 4
the cube, find the shortest distance of the vertex B1 from the plane of the ∆ O1EF.

Q.7 The vector OP = î + 2 ĵ + 2k̂ turns through a right angle, passing through the positive x-axis on the way..
Find the vector in its new position.
Q.8 Find the point R in which the line AB cuts the plane CDE where
    
a = î + 2ˆj + k̂ , b = 2î + ˆj + 2k̂ , c = − 4 ˆj + 4k̂ , d = 2î − 2ˆj + 2k̂ & e = 4î + ˆj + 2k̂ .
  
Q.9 If a = a1î + a 2 ĵ + a 3k̂ ; b = b1î + b 2 ĵ + b 3k̂ and c = c1î + c 2 ĵ + c3k̂ then show that the value of the
  
  a · î a · ĵ a · k̂
      
scalar triple product [ na + b nb + c nc + a ] is (n3 + 1) b · î b · ĵ b · k̂
  
c · î c · ĵ c · k̂
             
Q.10 F i n d t h e α & β if a x (b x c) + (a . b) b = (4 − 2β − sin α ) b + (β 2 − 1) c & ( c . c) a = c while b & c
s c a l a r s

are non zero non  collinear


 vectors.
FREE Download Study Package from website: www.TekoClasses.com & www.MathsBySuhag.com

Q.11 If the vectors b,c,d are not coplanar, then prove that the vector
            
(a × b) × (c × d ) + (a × c) × (d × b) + (a × d ) × (b × c) is parallel to a .
Q.12 a , b , c are non−coplanar unit vectors . The angle between b & c is α, between c & a is β and between

page 51 of 77
( )
a & b is γ . If A (a cos α ) , B b cos β , C ( c cos γ ) , then show that in ∆ ABC,

(
a x b x c ) =
b x ( c x a )
=
(
c x a x b )= ∏ (
a x b x c) where
sin A sin B sin C ∑ sin α cos β cos γ n 1
b x c c x a a x b
n 1 = , n 2 = & n 3 = .

Teko Classes, Maths : Suhag R. Kariya (S. R. K. Sir), Bhopal Phone : 0 903 903 7779, 0 98930 58881.
b x c c x a a x b
     
Q.13 Given that a,b,p,q are four vectors such that a +b=µp,b.q=0 &( b) 2 =1 , where µ is a scalar then
    
prove that (a.q )p −( p.q )a = p.q .
           
Q.14 Show that a = p x (q x r ) ; b = q x ( r x p) & c = r x ( p x q) represents the sides of a triangle. Further
prove that a unit vector perpendicular to the plane of this triangle is
n 1 tan ( p ^ q ) + n 2 tan (q ^ r ) + n 3 tan ( r ^ p)
     
    
± where a , b , c , p , q are non zero vectors and
n 1 tan ( p ^ q ) + n 2 tan (q ^ r ) + n 3 tan ( r ^ p)
     
     
   pxq qx r rxp

n =
no two of p , q , r are mutually perpendicular & 1   ; 2   & 3   
n = 
n =
pxq qx r rxp
Q.15 Given four points P1, P2, P3 and P4 on the coordinate plane with origin O which satisfy the condition
3
O P n −1 + O P n +1 = O P n , n = 2, 3
2
(i) If P1, P2 lie on the curve xy = 1, then prove that P3 does not lie on the curve.
(ii) If P1, P2, P3 lie on the circle x2 + y2 = 1, then prove that P4 lies on this circle.
  
Q.16 Let a = α i + 2 j − 3 k , b = i + 2 α j − 2 k and c = 2 i − α j + k . Find the value(s) of α, if any, such that
{(a × b) × ( b × c)} × (c × a ) = 0. Find the vector product when
 
α = 0.

    p.r p.s
Q.17 Prove the result (Lagrange’s identity) (p × q ) · ( r × s ) =     & use it to prove the following. Let
q.r q.s
(ab)denote the plane formed by the lines a,b. If (ab) is perpendicular to (cd) and (ac) is perpendicular to
(bd) prove that (ad) is perpendicular to (bc).      
     p 2 b + ( b . a ) a − p( b x a )
Q.18 (a) If px + ( x × a ) = b ; (p ≠ 0) prove that x = 2 .
2
p (p + a )
(b)

       
 
  
Solve the following equation for the vector p ; p xa + p . b c = b x c where a , b , c are non ( )
zero non coplanar vectors and a is neither perpendicular to b nor to c , hence show that
  a bc 
 
[ ]
 p × a +   c  is perpendicular to b − c .

 a ·c  


Q.19 Find a vector v which is coplanar with the vectors i + j − 2 k & i − 2 j + k and is orthogonal to the

vector − 2 i + j + k . It is given that the projection of v along the vector i − j + k is equal to 6 3 .
   
Q.20 Consider the non zero vectors a , b , c & d such that no three of which are coplanar then prove that
 
[ ] [ ] [ ] [ ]
     
   
a b c d + c a b d = b a c d + d a b c . Hence prove that a , b , c & d represent the position vectors of

[ ] [ ]
 
b cd + a b d
 =1 .
[ ] [ ]
the vertices of a plane quadrilateral if   
a cd + a b c
         
Q.21 The base vectors a1,a 2 ,a 3 are given in terms of base vectors b1,b 2 ,b3 as, a1 = 2b1 + 3b 2 − b3 ;
            
a 2 = b1 − 2b 2 + 2b 3 & a 3 = −2b1 + b 2 − 2b 3 . If F = 3b1 − b 2 + 2b 3 , then express F in terms of
  
a1, a 2 & a 3 .
()

If A (a ) ; B b & C ( c) are three non collinear points , then for any point P ( p) in the plane of the
  
Q.22
FREE Download Study Package from website: www.TekoClasses.com & www.MathsBySuhag.com

[  ] ( )
      
∆ ABC , prove that ; (i) a b c = p . a x b + bx c + cxa

(ii) The vector v perpendicular to the plane of the triangle ABC drawn from the origin 'O' is given by
[ ](
      
)

page 52 of 77
 a bc a ×b + b×c + c×a 
v =± where ∆ is the vector area of the triangle ABC.
4∆2
Q.23 Given the points P (1, 1, –1), Q (1, 2, 0) and R (–2, 2, 2). Find
(a) PQ× P R
(b) Equation of the plane in
(i) scalar dot product form (ii) parametric form (iii) cartesian form

Teko Classes, Maths : Suhag R. Kariya (S. R. K. Sir), Bhopal Phone : 0 903 903 7779, 0 98930 58881.
(iv) if the plane through PQR cuts the coordinate axes at A, B, C then the area of the ∆ABC
  
Q.24 Let a , b & c be non coplanar unit vectors, equally inclined to one another at an angle θ. If
      
a x b + b x c = p a + q b + r c . Find scalars p , q & r in terms of θ.
 
Q.25 Solve the simultaneous vector equations for the vectors x and y .
        
x + c × y = a and y + c × x = b where c is a non zero vector..
EXERCISE–3
Q.1 Find the angle between the two straight lines whose direction cosines l, m, n are given by
2l + 2m – n = 0 and mn + nl + lm = 0.
Q.2 If two straight line having direction cosines l, m, n satisfy al + bm + cn = 0 and f m n + g n l + h l m = 0
f g h
are perpendicular, then show that + + = 0.
a b c
Q.3 P is any point on the plane lx + my + nz = p. A point Q taken on the line OP (where O is the origin) such
that OP. OQ = p2. Show that the locus of Q is p( lx + my + nz ) = x2 + y2 + z2.
Q.4 Find the equation of the plane through the points (2, 2, 1), (1, –2, 3) and parallel to the x-axis.
Q.5 Through a point P (f, g, h), a plane is drawn at right angles to OP where 'O' is the origin, to meet the
r5
coordinate axes in A, B, C. Prove that the area of the triangle ABC is where OP = r..
2f g h
Q.6 The plane lx + my = 0 is rotated about its line of intersection with the plane z = 0 through an angle θ.
Prove that the equation to the plane in new position is lx + my + z l 2 + m 2 tan θ = 0
Q.7 Find the equations of the straight line passing through the point (1, 2, 3) to intersect the straight line
x + 1 = 2 (y – 2) = z + 4 and parallel to the plane x + 5y + 4z = 0.
x −3 y−3 z
Q.8 Find the equations of the two lines through the origin which intersect the line = = at an
π 2 1 1
angle of .
Q.9 A variable3plane is at a constant distance p from the origin and meets the coordinate axes in points A, B
and C respectively. Through these points, planes are drawn parallel to the coordinates planes. Find the
locus of their point of intersection.
x + 2 2 y + 3 3z + 4
Q.10 Find the distance of the point P (– 2, 3, – 4) from the line = = measured parallel to
3 4 5
the plane 4x + 12y – 3z + 1 = 0.
Q.11 Find the equation to the line passing through the point (1, –2, –3) and parallel to the line
2x + 3y – 3z + 2 = 0 = 3x – 4y + 2z – 4.
Q.12 Find the equation of the line passing through the point (4, –14, 4) and intersecting the line of intersection
of the planes : 3x + 2y – z = 5 and x – 2y – 2z = –1 at right angles.
Q.13 Let P = (1, 0, – 1) ; Q = (1, 1, 1) and R = (2, 1, 3) are three points.
(a) Find the area of the triangle having P, Q and R as its vertices.
(b) Give the equation of the plane through P, Q and R in the form ax + by + cz = 1.
(c) Where does the plane in part (b) intersect the y-axis.
(d) Give parametric equations for the line through R that is perpendicular to the plane in part (b).
Q.14 Find the point where the line of intersection of the planes x – 2y + z = l and x + 2y – 2z = 5, intersects
the plane 2x + 2y + z + 6 = 0.
Q.15 Feet of the perpendicular drawn from the point P (2, 3, –5) on the axes of coordinates are A, B and C.
Find the equation of the plane passing through their feet and the area of ∆ABC.
FREE Download Study Package from website: www.TekoClasses.com & www.MathsBySuhag.com

Q . 1 6 F i n d t h e e q u a t i o n s t o t h e l i n e w h i c h c a n b e d r a w n f r o m t h e p o i n t ( 2 , – 1 , 3 ) p e r p e n d i c u l a r t o t h e l i n e s

x −1 y − 2 z − 3 x−4 y z+3
= = and = = at right angles.
2 3 4 4 5 3
x −1 y + 2 z
= =

page 53 of 77
Q.17 Find the equation of the plane containing the straight line and perpendicular to the
2 −3 5
plane x – y + z + 2 = 0.
x +1 y − p z+2 x y−7 z+7
Q.18 Find the value of p so that the lines = = and = = are in the same
−3 2 1 1 −3 2
plane. For this value of p, find the coordinates of their point of intersection and the equation of the plane
containing them.

Teko Classes, Maths : Suhag R. Kariya (S. R. K. Sir), Bhopal Phone : 0 903 903 7779, 0 98930 58881.
Q.19 Find the equations to the line of greatest slope through the point (7, 2 , –1) in the plane
x – 2y + 3z = 0 assuming that the axes are so placed that the plane 2x + 3y – 4z = 0 is horizontal.
Q.20 Let ABCD be a tetrahedron such that the edges AB, AC and AD are mutually perpendicular. Let the
area of triangles ABC, ACD and ADB be denoted by x, y and z sq. units respectively. Find the area of
the triangle BCD.
Q.21 The position vectors of the four angular points of a tetrahedron OABC are (0, 0, 0); (0, 0, 2); (0, 4, 0)
and (6, 0, 0) respectively. A point P inside the tetrahedron is at the same distance 'r' from the four plane
faces of the tetrahedron. Find the value of 'r'.
x + 6 y + 10 z + 14
Q.22 The line = = is the hypotenuse of an isosceles right angled triangle whose opposite
5 3 8
vertex is (7, 2, 4). Find the equation of the remaining sides.
Q.23 Find the foot and hence the length of the perpendicular from the point (5, 7, 3) to the line
x − 15 y − 29 5 − z
= = . Also find the equation of the plane in which the perpendicular and the given
3 8 5
straight line lie.
x −1 y − 2 z + 3
Q.24 Find the equation of the line which is reflection of the line = = in the plane
3x – 3y + 10z = 26. 9 −1 −3
x −1 y z x −3 y z −2
Q.25 Find the equation of the plane containing the line = = and parallel to the line = = .
2 3 2 2 5 4
Find also the S.D. between the two lines.
EXERCISE–4
→ → → → → → →
Q.1(a) Let OA = a , OB = 10 a + 2 b and OC = b where O, A & C are non−collinear points . Let p denote the
area of the quadrilateral OABC, and let q denote the area of the parallelogram with OA and OC as
adjacent sides. If p = kq, then k = _______ .
    
(b) If A , B & C are vectors such that | B | = | C | , Prove that ;
[(A + B) x (A + C )] x(Bx C).(B + C) = 0
       
[ JEE ' 97, 2 + 5 ]
Q.2(a) Vectors x , y & z each of magnitude 2 , make angles of 60 0 with each other. If
                
x × ( y × z) = a , y × (z × x ) = b and x × y = c then find x, y and z in terms of a , b and c .
(b) The position vectors of the points P & Q are 5 i + 7 j − 2 k and − 3 i + 3 j + 6 k respectively. The
 
vector A = 3 i − j + k passes through the point P & the vector B = − 3 i + 2 j + 4 k passes through the
 
point Q. A third vector 2 i + 7 j − 5 k intersects vectors A & B . Find the position vectors of the points
of intersection. [ REE ' 97, 6 + 6 ]
Q.3(a) Select the correct alternative(s)
   
(i) If a = i + j + k , b = 4 i + 3j + 4 k and c = i + αj + βk are linearly dependent vectors & c = 3 , then:
(A) α = 1, β = −1 (B) α = 1, β = ±1 (C) α = −1, β = ±1 (D) α = ±1, β = 1
  
(ii) For three vectors u , v , w which of the following expressions is not equal to any of the remaining three?
           
(A) u . ( v x w ) (B) ( v x w ) . u (C) v . ( u x w ) (D) ( u x v ) . w
(iii) Which of the following expressions are meaningful ?
           
(A) u . ( v x w ) (B) ( u . v ) . w (C) ( u . v ) w (D) u x ( v . w )
(b) Prove, by vector methods or otherwise, that the point of intersection of the diagonals of a trapezeum lies
on the line passing through the mid−points of the parallel sides.
FREE Download Study Package from website: www.TekoClasses.com & www.MathsBySuhag.com

(You may assume that the trapezeum is not a parallelogram.)


 
(c) For any two vectors u & v , prove that [ JEE ' 98 , 2 + 2 + 2 + 8 + 8 ]
  2  2 2 2 2       
(i) (u. v) + | u × v| = | u| | v| & (ii) (1 + | u| )(1 +| v|2 ) = (1 − u. v)2 + | u + v + (u × v)|2

page 54 of 77
               
Q.4(a) If x × y = a , y × z = b, x. b = γ , x. y = 1 and y. z = 1 then find x, y & z in terms of a , b and γ .
→ →
(b) Vectors AB = 3 i − j + k & CD = − 3 i + 2 j + 4 k are not coplanar. The position vectors of points
A and C are 6 i + 7j + 4 k and − 9j + 2k respectively . Find the position vectors of a point P on the
→ → →
line AB & a point Q on the line CD such that PQ is perpendicular to AB and CD both.

Teko Classes, Maths : Suhag R. Kariya (S. R. K. Sir), Bhopal Phone : 0 903 903 7779, 0 98930 58881.
       
Q.5(a) Let a = 2î + ˆj − 2k̂ & b = î + ˆj . If c is a vector such that a · c =| c | , c − a = 2 2 and the angle
 
( ) 
between a × b and c is 30º, then a × b × c =
  
( )
(A) 2/3 (B) 3/2 (C) 2 (D) 3
     
(b) Let a =2î + j+k̂ , b = i + 2 j − k and a unit vector c be coplanar. If c is perpendicular to a , then c =
ˆ   

(A)
1
2
(− j + k ) (B)
1
3
(− i − j − k )
5
( i − 2 j
)(C)
1
(D)
3
( i − j − k)
1   

 
(c) Let a & b be two non-collinear unit vectors . If u = a − (a . b ) b & v = a x b , then v is :
        

(D) u + u . (a + b)
          
(A) u (B) u + u . a (C) u + u . b
   
(d) Let u & v be unit vectors . If w is a vector such that w + (w x u) = v , then prove that
  

( u x v) . w ≤ 2 and the equality holds if and only if u is perpendicular to v .


   1

Q.6(a) An arc AC of a circle subtends a right angle at the centre O. The point B divides the arc in the ratio 1 : 2.
→  →  →  
If OA = a & OB = b , then calculate OC in terms of a & b .
   
(b) If a , b , c are non-coplanar vectors and d is a unit vector, then find the value of,
( )(
   
) ( ) ( )( )
        
a . d b x c + b . d ( c x a ) + c . d a x b independent of d . [ REE '99, 6 + 6 ]
Q.7(a) Select the correct alternative :
  
(i) If the vectors a , b & c form the sides BC, CA & AB respectively of a triangle ABC, then
           
(A) a . b + b . c + c . a = 0 (B) a × b = b × c = c × a
           
(C) a . b = b . c = c . a (D) a × b + b × c + c × a = 0
( )( )
        
(ii) Let the vectors a , b , c & d be such that a × b × c × d = 0 . Let P1 & P2 be planes determined by
   
the pairs of vectors a , b & c , d respectively . Then the angle between P1 and P2 is :
(A) 0 (B) π/4 (C) π/3 (4) π/2
  
(iii) If a , b & c are unit coplanar vectors, then the scalar triple product
[ 2 a − b ]
    
2b−c 2c−a = [ JEE ,2000 (Screening) 1 + 1 + 1 out of 35 ]
(A) 0 (B) 1 (C) − 3 (D) 3
(b) Let ABC and PQR be any two triangles in the same plane . Assume that the perpendiculars from the
points A, B, C to the sides QR, RP, PQ respectively are concurrent . Using vector methods or otherwise,
prove that the perpendiculars from P, Q, R to BC, CA, AB respectively are also concurrent .
[ JEE '2000 (Mains) 10 out of 100 ]
  
Q.8. (i) If a = i + j − k , b = − i + 2 j + 2 k & c = − i + 2 j − k , find a unit vector normal to the vectors
   
a + b and b − c .
    
(ii) Given that vectors a & b are perpendicular to each other, find vector υ in terms of a & b satisfying
[ υ , a , b] = 1
    
the equations, υ . a = 0 , υ . b = 1 and
( ) ( b + c) . Find angle between vectors
      1 
(iii) a , b & c are three unit vectors such that a × b × c =
  2
FREE Download Study Package from website: www.TekoClasses.com & www.MathsBySuhag.com

 
a & b given that vectors b & c are non-parallel.
(iv) A particle is placed at a corner P of a cube of side 1 meter . Forces of magnitudes 2, 3 and 5 kg weight
act on the particle along the diagonals of the faces passing through the point P . Find the moment of these
forces about the corner opposite to P . [ REE '2000 (Mains) 3 + 3 + 3 + 3 out of 100 ]

page 55 of 77
Q.9(a) The diagonals of a parallelogram are given by vectors 2i + 3j − 6 k and 3i − 4 j − k . Determine its sides
and also the area.
(b) Find the value of λ such that a, b, c are all non-zero and
( )
−4 i + 5j a + (3i − 3j + k ) b + (i + j + 3k ) c = λ (ai + bj + ck )
 
[ REE '2001 (Mains) 3 + 3]

Q.10(a) Find the vector r which is perpendicular to a = i − 2j + 5k and b = 2i + 3j − k and
( )

Teko Classes, Maths : Suhag R. Kariya (S. R. K. Sir), Bhopal Phone : 0 903 903 7779, 0 98930 58881.

r ⋅ 2i + j + k + 8 = 0.
(b) Two vertices of a triangle are at − i + 3j and 2i + 5j and its orthocentre is at i + 2j . Find the position
vector of third vertex. [ REE '2001 (Mains) 3 + 3]
    2  2  2
Q.11 (a) If a , b and c are unit vectors, then a − b + b − c + c − a does NOT exceed
(A) 4  (B) 9 (C) 8 (D) 6
   
(b) Let a = î − k̂ , b = x î + ĵ + (1 − x )k̂ and c = yî + x ĵ + (1 + x − y)k̂ . Then [a , b, c] depends on
(A) only x (B) only y (C) NEITHER x NOR y (D) both x and y
[ JEE '2001 (Screening) 1 + 1 out of 35]
Q.12(a) Show by vector methods, that the angular bisectors of a triangle are concurrent and find an expression
for the position vector of the point of concurrency in terms of the position vectors of the vertices.
  
(b) Find 3–dimensional vectors v 1 , v 2 , v 3 satisfying
           
v 1 ⋅ v 1 = 4, v 1 ⋅ v 2 = –2, v 1 ⋅ v 3 = 6, v 2 ⋅ v 2 = 2, v 2 ⋅ v 3 = –5, v 3 ⋅ v 3 = 29.
 
(c) Let A(t ) = f1 (t )i + f2 (t )j and B( t ) = g1 ( t ) i + g 2 ( t ) j , t ∈ [0, 1], where f1, f2, g1, g2 are continuous
  
functions. If A(t ) and B( t ) are nonzero vectors for all t and A(0) = 2i + 3j ,
    
A(1) = 6 i + 2j , B(0) = 3i + 2j and B(1) = 2i + 6 j , then show that A(t ) and B(t ) are parallel for
some t. [ JEE '2001 (Mains) 5 + 5 + 5 out of 100 ]
     
Q.13(a) If a and b are two unit vectors such that a + 2 b and 5 a – 4 b are perpendicular to each other then
 
the angle between a and b is
 1  2
(A) 450 (B) 600 (C) cos–1   (D) cos–1  
  3  7
 
(b) Let V = 2î + j − k̂ and W = î + 3k̂ . If U is a unit vector, then the maximum value of the scalar triple
ˆ

[ ]
  
product U V W is [JEE 2002(Screening), 3 + 3]
(A) –1 (B) 10 + 6 (C) 59 (D) 60

Q.14 Let V be the volume of the parallelopiped formed by the vectors a = a1î + a 2 ĵ + a 3k̂ ,
 
b = b1î + b 2 ĵ + b3k̂ , c = c1î + c 2 ĵ + c3k̂ . If ar , br , cr , where r = 1, 2, 3, are non-negative real
3
numbers and ∑ (a r + b r + c r ) = 3L, show that V < L3. [JEE 2002(Mains), 5]
r =1
  
Q.15 If a = î + aˆj + k̂ , b = ˆj + ak̂ , c = aî + k̂ , then find the value of ‘a’ for which volume of
parallelopiped formed by three vectors as coterminous edges, is minimum, is
1 1 1
(A) (B) – (C) ± (D) none [JEE 2003(Scr.), 3]
3 3 3
Q.16(i) Find the equation of the plane passing through the points (2, 1, 0) , (5, 0, 1) and (4, 1, 1).
(ii) If P is the point (2, 1, 6) then find the point Q such that PQ is perpendicular to the plane in (i) and the mid
point of PQ lies on it. [ JEE 2003, 4 out of 60]
    
Q.17 If u , v , w are three non-coplanar unit vectors and α, β, γ are the angles between u and v ,
FREE Download Study Package from website: www.TekoClasses.com & www.MathsBySuhag.com

      
v and w , w and u respectively and x , y , z are unit vectors along the bisectors of the angles
2 α β γ
α, β, γ respectively. Prove that [x × y y × z z × x ] = [u v w ] sec
      1    2
sec 2 sec 2 .
16 2 2 2

page 56 of 77
[ JEE 2003, 4 out of 60 ]
x −1 y + 1 z −1 x −3 y−k z
Q.18(a) If the lines = = and = = intersect, then k =
2 3 4 1 2 1
2 9
(A) (B) (C) 0 (D) – 1
9 2
(b) A unit vector in the plane of the vectors 2î + ˆj + k̂ , î − ˆj + k̂ and orthogonal to 5î + 2ˆj + 6k̂

Teko Classes, Maths : Suhag R. Kariya (S. R. K. Sir), Bhopal Phone : 0 903 903 7779, 0 98930 58881.
6î − 5k̂ 3ˆj − k̂ 2î − 5k̂ 2î + ĵ − 2k̂
(A) (B) (C) (D)
61 10 29 3
      ˆ 
(c) If a = î + j + k̂ , a · b = 1 and a × b = j − k̂ , then b = [ JEE 2004 (screening)]
(A) î (B) î − ˆj + k̂ (C) 2ˆj − k̂ (D) 2î
           
Q.19(a) Let a , b, c, d are four distinct vectors satisfying a × b = c × d and a × c = b × d . Show that
       
a ·b + c ·d ≠ a ·c + b·d .
(b) T is a parallelopiped in which A, B, C and D are vertices of one face. And the face just above it has
corresponding vertices A', B', C', D'. T is now compressed to S with face ABCD remaining same and
A', B', C', D' shifted to A., B., C., D. in S. The volume of parallelopiped S is reduced to 90% of T. Prove
that locus of A. is a plane.
(c) Let P be the plane passing through (1, 1, 1) and parallel to the lines L1 and L2 having direction ratios
1, 0, –1 and –1, 1, 0 respectively. If A, B and C are the points at which P intersects the coordinate axes,
find the volume of the tetrahedron whose vertices are A, B, C and the origin.  
     b · a    b · a 
Q.20(a) If a , b, c are three non-zero, non-coplanar vectors and b1 = b −  a , b = b +  a ,
| a |2 2
| a |2
               
  c ·a  b· c    c · a  b1 · c    c·a  b·c    c·a  b·c 
c1 = c −  2 a +  2 b1,c2 = c −  2 a −  2 b1,c3 = c −  2 a +  2 b1,c 4 = c −  2 a −  2 b1
|a| |c| |a| | b1 | |c| |c| |c| |b|
then the set of orthogonal vectors is
(
  
(A) a , b1 , c3 ) (
  
(B) a , b1 , c 2 ) (
  
(C) a , b1 , c1 ) (  
(D) a , b 2 , c2 )
(b) A variable plane at a distance of 1 unit from the origin cuts the co-ordinate axes at A, B and C. If the
1 1 1
centroid D (x, y, z) of triangle ABC satisfies the relation 2 + 2 + 2 = k, then the value of k is
x y z
(A) 3 (B) 1 (C) 1/3 (D) 9 [JEE 2005 (Screening), 3]
(c) Find the equation of the plane containing the line 2x – y + z – 3 = 0, 3x + y + z = 5 and at a distance of
1
from the point (2, 1, – 1).
6
(d) Incident ray is along the unit vector v̂ and the reflected ray is along the
unit vector ŵ . The normal is along unit vector â outwards. Express
ŵ in terms of â and v̂ .
[ JEE 2005 (Mains), 2 + 4 out of 60 ]
Q.21(a) A plane passes through (1, –2, 1) and is perpendicular to two planes 2x – 2y + z = 0 and
x – y + 2z = 4. The distance of the plane from the point (1, 2, 2) is
(A) 0 (B) 1 (C) 2 (D) 2 2
    
(b) Let a = î + 2 j + k̂ , b = î − j + k̂ and c = î + j − k̂ . A vector in the plane of a and b whose projection
ˆ ˆ ˆ
 1
on c is , is [JEE 2006,3 marks each]
3
(A) 4î − ˆj + 4k̂ (B) 3î + ˆj − 3k̂ (C) 2î + ˆj − 2k̂ (D) 4î + ˆj − 4k̂

(c) Let A be vector parallel to line of intersection of planes P1 and P2 through origin. P1 is parallel to the
FREE Download Study Package from website: www.TekoClasses.com & www.MathsBySuhag.com

vectors 2 ˆj + 3 k̂ and 4 ˆj – 3 k̂ and P2 is parallel to ˆj – k̂ and 3 î + 3 ˆj , then the angle between



vector A and 2 î + ˆj – 2 k̂ is

page 57 of 77
π π π 3π
(A) (B) (C) (D) [JEE 2006, 5]
2 4 6 4
(d) Match the following
(i) Two rays in the first quadrant x + y = | a | and ax – y = 1 intersects each other in the interval
a ∈ (a0, ∞), the value of a0 is (A) 2
(ii) Point (α, β, γ) lies on the plane x + y + z = 2.
 
Let a = α î + β ˆj + γ k̂ , k̂ × (k̂ × a ) = 0, then γ =

Teko Classes, Maths : Suhag R. Kariya (S. R. K. Sir), Bhopal Phone : 0 903 903 7779, 0 98930 58881.
(B) 4/3
1 0 1 0
(iii) ∫ (1 − y ) dy + ∫ ( y − 1) dy ∫ 1 − x dx + ∫ 1 + x dx
2 2
(C)
0 1 0 −1
(iv) If sinA sinB sinC + cos A cosB = 1,
then the value of sin C = (D) 1 [JEE 2006, 6]
(e) Match the following

 1 
(i) ∑ tan −1  2  = t , then tan t = (A) 0
i =1  2i 
(ii) Sides a, b, c of a triangle ABC are in A.P.
a b c
and cos θ1 = , cos θ2 = , cos θ3 = ,
b+c a +c a+b
θ θ
then tan 2 1 + tan 2 3 = (B) 1
2 2
(iii) A line is perpendicular to x + 2y + 2z = 0 and
passes through (0, 1, 0). The perpendicular
5
distance of this line from the origin is (C)
3
(D) 2/3 [JEE 2006, 6]
ANSWER KEY
EXERCISE–1
FREE Download Study Package from website: www.TekoClasses.com & www.MathsBySuhag.com

Q.1 x = 2 , y = −1 Q.2 (b) externally in the ratio 1 : 3


Q.4 (i) parallel (ii) the lines intersect at the point p.v. − 2 i + 2 j (iii) lines are skew

page 58 of 77
Q.5 2:1 Q.7 xx1 + yy1 = a2 Q.10 x = 2, y = – 2, z = – 2
−1 1 1 1
Q.13 (a) i − j+ k Q.15 (a) arc cos Q.18 − î + 2ˆj + 5k̂
2 2 2 3

11 3 5a2
Q.19 Q.20 (b) Q.21 (a) ± 3( î − 2ˆj − 2k̂ ), (b) y = 3 or y = – 1 Q.24 sq. units

Teko Classes, Maths : Suhag R. Kariya (S. R. K. Sir), Bhopal Phone : 0 903 903 7779, 0 98930 58881.
3 2 12 3

q (q 3 − 3)
Q.25 p = ; decreasing in q ∈ (–1, 1), q ≠ 0
4
EXERCISE–2
1
Q.1 2 17 Q.2 ± (î + 5 ĵ − k̂ ) Q.4 NO, NO
3 3

6 3 11 4 1 1
Q.5 (i) 14 (ii) 6 (iii) 10 (iv) 6 Q.6 Q.7 î − ĵ − k̂
7 5 170 2 2 2
 (−1) n π
Q.8 p.v. of R = r = 3i + 3k Q.10 α =nπ + ,n ∈ I&β = 1
2

Q.16 α = 2/3 ; if α = 0 then vector product is − 60 2 i + k ( )


 [ ] (

abc   
) ( ) ( )
  
b.c b
(b) p =     a + c × b +   −   
  
b. b c 
(
9 − j + k )
( ) ( ) ( )
Q.18 Q.19
 (a . c ) a . b a .b a .b 
  
Q.21 F = 2a1 + 5a 2 + 3a 3

4 22
Q.23 (a) 2î − 3ˆj + 3k̂ , (b) (i) – 4, (ii) r̂ = î + ˆj − k̂ + λ (ˆj + k̂ ) + µ(−3î + ĵ + 3k̂ ) , (iii) – 4, (iv)
9
1 2 cos θ 1
Q.24 p=− ; q= ; r= −
1 + 2 cosθ 1 + 2 cos θ 1 + 2 cosθ

1 2cosθ 1
or p= ; q=− ; r=
1 + 2 cos θ 1 + 2cosθ 1 + 2 cos θ
         
 a + ( c. a ) c + b × c b + ( c. b) c + a × c
Q.25 x =  , y= 
1 + c2 1 + c2

EXERCISE–3
x −1 y − 2 z − 3
Q.1 θ = 900 Q.4 y + 2z = 4 Q.7 = =
2 2 −3
x y z x y z 1 1 1 1 17
Q.8 = = or = = Q.9 2
+ 2+ 2 = 2 Q.10
1 2 −1 −1 1 − 2 x y z p
FREE Download Study Package from website: www.TekoClasses.com & www.MathsBySuhag.com

2
x −1 y + 2 z + 3 x − 4 y + 14 z − 4
Q.11 = = Q.12 = =
6 13 17 3 10 4

page 59 of 77
3 2x 2y z  3 
Q.13 (a) ; (b) + − = 1; (c)  0, , 0  ; (d) x = 2t + 2 ; y = 2t + 1 and z = – t + 3
2 3 3 3  2 
x y z 19 x − 2 y +1 z−3
Q.14 (1, –2, – 4) Q.15 + + = 1 , Area = sq. units Q.16 = =
2 3 −5 2 11 − 10 2

Teko Classes, Maths : Suhag R. Kariya (S. R. K. Sir), Bhopal Phone : 0 903 903 7779, 0 98930 58881.
Q.17 2x + 3y + z + 4 = 0 Q.18 p = 3, (2, 1, –3) ; x + y + z = 0
x−7 y−2 z +1 2
Q.19 = = Q.20 (x 2 + y 2 + z 2 ) Q.21
22 5 −4 3
x −7 y−2 z−4 x −7 y−2 z−4
Q.22 = = ; = =
3 6 2 2 −3 6
x − 4 y +1 z − 7
Q.23 (9, 13, 15) ; 14 ; 9x – 4y – z = 14 Q.24 = =
9 −1 −3
Q.25 x – 2y + 2z – 1 = 0; 2 units
EXERCISE–4
Q.1 (a) 6
            
Q.2 (a) x = a × c ; y = b × c ; z = b + a × c or b × c − a (b) (2, 8, − 3) ; (0, 1, 2)
Q.3 (a) (i) D (ii) C (iii) A, C
       
a×b  a×b   a×b  a×b
γ −a × γ a×b γ +b× γ
Q.4 (a) x = ; y= ; z= (b) P ≡ (3, 8, 3) & Q ≡ (−3, −7, 6)
  2
a×b γ   2
 a × b
 γ   γ 
 
 

[a b c]
   
Q.5 (a) B (b) A (c) A, C Q.6 (a) c = − 3 a + 2b (b)
Q.7 (a) (i) B (ii) A (iii) A
  
b a xb 2π 
Q.8 (i) + i ; (ii)  2 +   2 ; (iii) ; (iv) | M| = 7
b (a × b ) 3

Q.9 (a)
1  
2
( 1
2
)
5i − j − 7k , (− i + 7j − 5k );
1
2
1274 sq. units (b) λ = 0, λ = –2 + 29

 5 17
Q.10 (a) r = −13i + 11j + 7k ; (b) î + ĵ
7 7
  
Q.11 (a) B (b) C Q.12 (b) v 1 = 2i , v 2 = − i ± j, v 3 = 3i ± 2j ± 4 k
Q.13 (a) B ; (b) C Q.15 D Q.16 (i) x + y – 2z = 3 ; (ii) (6, 5, –2)
Q.18 (a) B, (b) B, (c) A Q.19 (c) 9/2 cubic units
Q.20 (a) B, (b) D ; (c) 2x – y + z – 3 = 0 and 62x + 29y + 19z – 105 = 0, (d) ŵ = v̂ – 2( â · v̂ ) â
Q.21 (a) D; (b) A; (c) B, D; (d) (i) D, (ii) A, (iii) B, C, (iv) D; (e) (i) B, (ii) D, (iii) C
EXERCISE–5
Part : (A) Only one correct option
FREE Download Study Package from website: www.TekoClasses.com & www.MathsBySuhag.com

1. The locus of a point P which moves such that PA2 – PB2 = 2k2 where A and B are (3, 4, 5) and
(– 1, 3 – 7) respectively is
(A) 8x + 2y + 24z – 9 + 2k2 = 0 (B) 8x + 2y + 24z – 2k2 = 0
(C) 8x + 2y + 24z + 9 + 2k2 = 0 (D) none of these
The position vectors of three points A, B, C are i + 2 j + 3 k , 2 i + 3 j + k & 3 i + j + 2 k . A unit vector

page 60 of 77
2.
perpendicular to the plane of the triangle ABC is:

(A)  −

(
1    
 i+ j+k
3
) (B) 
 3
(
 1    
 i− j+k ) (C) 
 1    
 3
(
 i+ j−k ) (D) none
3. The square of the perpendicular distance of a point P (p, q, r) from a line through A(a, b, c) and whose
direction cosine are , m, n is
(A) Σ {(q – b) n – (r – c) m}2 (B) Σ {(q + b) n – (r + c) m}2

Teko Classes, Maths : Suhag R. Kariya (S. R. K. Sir), Bhopal Phone : 0 903 903 7779, 0 98930 58881.
(C) Σ {(q – b) n + (r – c) m}2 (D) none of these
4. A variable plane passes through a fixed point (1, 2, 3). The locus of the foot of the perpendicular drawn
from origin to this plane is:
(A) x 2 + y2 + z2 − x − 2y − 3z = 0 (B) x 2 + 2y2 + 3z2 − x − 2y − 3z = 0
(C) x 2 + 4y2 + 9z2 + x + 2y + 3 = 0 (D) x 2 + y2 + z2 + x + 2y + 3z = 0
5. The equation of the plane which bisects the angle between the planes 3x − 6y + 2z + 5 = 0 and
4x − 12y + 3z − 3 = 0 which contains the origin is
(A) 33x − 13y + 32z + 45 = 0 (B) x − 3y + z − 5 = 0 (C) 33x + 13y + 32z + 45 = 0 (D) None
6. The distance of the point of intersection of the line x – 3 = (1/2) (y–4) = (1/2) (z–5) and the plane
x + y + z = 17 from the point (3, 4, 5)
(A) 2 (B) 3 (C) 1/3 (D) 1/2
7. The lines x = ay + b, z = cy + d and x = a’y + b’, z = c’y + d’ will be mutually perpendicular provided
(A) (a + a’)(b + b’) (c + c’) (B) aa’ + cc’ + 1 = 0
(C) aa’ + bb’ + cc’ + 1 = 0 (D) (a + a’) (b + b’) (c + c’) + 1 = 0

  
8. A straight line r = a + λ b meets the plane r . n̂ = p in the point P whose position vector is
   
  a . n̂     p − a . n̂     a . n̂     p − a . n̂  
(A) a +    b (B) a +    b (C) a −  b . n̂  b (D) a −    b
 b . n̂   b . n̂     b . n̂ 
x −1 y − 2 z−3 x −1 y − 2 z−3
9. Equation of the angle bisector of the angle between the lines = = & = = is
1 1 1 1 1 −1
x −1 y−2 x −1 y−2 z−3
(A) = ;z –3=0 (B) = =
2 2 1 2 3
y−2 z−3
(C) x – 1 = 0 ; = (D) None of these
1 1
x − 2 y +1 z − 2
10. The distance of the point, (− 1, − 5, − 10) from the point of intersection of the line, = =
3 4 12
and the plane, x − y + z = 5, is:
(A) 10 (B) 11 (C) 12 (D) 13
11. If a plane cuts off intercepts OA = a, OB = b, OC = c from the coordinate axes, then the area of the
triangle ABC =
1 1 1 1
(A) b 2 c2 + c2a 2 + a 2 b2 (B) (bc + ca + ab) (C) abc (D) (b − c)2 + (c − a)2 + (a − b)2
2 2 2 2
12. The angle between the lines whose direction cosines satisfy the equations  + m + n = 0 and
2 = m 2 + n2 is
π π π π
(A) (B) (C) (D)
6 2 3 4
13. If a1, b1, c1 and a2 , b2, c2 are the direction ratios of two lines and θ is the angle between the lines then
tan θ is equal to
Σ(b1c 2 − b 2 c 1 )2 Σ(b1c 2 − b 2 c1 )2 Σ(b1c 2 + b 2 c 1 )2
(A) (B) (C) (D) none of these
a1b1 + a 2b 2 + c 1c 2 a1a 2 + b1b 2 + c 1c 2 a1a 2 + b1b 2 + c1c 2
14. A point moves so that the sum of the squares of its distances from the six faces of a cube given by
x = ± 1, y = ± 1, z = ± 1 is 10 units. The locus of the point is
(A) x 2 + y2 + z2 = 1 (B) x 2 + y2 + z2 = 2 (C) x + y + z = 1 (D) x + y + z = 2
15. In the adjacent figure ‘P’ is any arbitrary interior point of the triangle ABC such that the lines AA1, BB1 and
PA 1 PB1 PC1
CC1 are concurrent at P. Value of + + is always equal to .
AA 1 BB1 CC1

(A) 1 (B) 2 (C) 3 (D) None of these


16. The plane ax + by + cz = d, meets the coordinate axes at the points A, B and C respectively. Area of triangle
ABC is equal to
d2 a 2 + b 2 + c 2 d2 a 2 + b 2 + c 2 d2 a 2 + b 2 + c 2
(A) (B) (C) (D) None of these
| abc | 2 | abc | 4 | abc |
17. The length of projection, of the line segment joining the points (1, –1, 0) and (–1, 0, 1), to the plane
2x + y + 6z = 1, is equal to
255 237 137 155
FREE Download Study Package from website: www.TekoClasses.com & www.MathsBySuhag.com

(A) (B) (C) (D)


61 61 61 61
18. Two systems of rectangular axes have the same origin. If a plane cuts them at distances a, b, c and
a1, b1, c1 from the origin, then
1 1 1 1 1 1 1 1 1 1 1 1

page 61 of 77
(A) 2 + 2 + 2 = 2 + 2 + 2 (B) 2 − 2 + 2 = 2 − 2 + 2
a b c a1 b1 c1 a b c a1 b1 c 1
(C) a2 + b2 + c2 = a1 + b1 + c 1
2 2 2
(D) a2 – b2 + c2 = a12 − b12 + c 12
19. The angle between the plane 2x – y + z = 6 and a plane perpendicular to the planes x + y + 2z = 7 and
x – y = 3 is :
π π π π
(A) (B) (C) (D)
4 3 6 2

Teko Classes, Maths : Suhag R. Kariya (S. R. K. Sir), Bhopal Phone : 0 903 903 7779, 0 98930 58881.
20. The non zero value of ‘a’ for which the lines 2x – y + 3z + 4 = 0 = ax + y – z + 2 and
x – 3y + z = 0 = x + 2y + z + 1 are co-planar is :
(A) – 2 (B) 4 (C) 6 (D) 0
21. The equation of the plane through the point (–1, 2 , 0) and parallel to the lines
x y +1 z − 2 x − 1 2y + 1 z + 1
= = and = = is -
3 0 −1 1 2 −1
(A) x + 2y + 3z - 1 = 0 (B) x – 2y + 3z + 5 = 0
(C) x + y – 3z + 1 = 0 (D) x + y + 3z – 1 = 0
22. The equation of the plane bisecting the acute angle between the planes 2x + y + 2z = 9 and
3x – 4y + 12z + 13 = 0 is :
(A) 11x + 33y – 34z – 172 = 0 (B) 11x + 33y – 34z – 182 = 0
(C) 41x – 7y + 86z – 52 = 0 (D) 41x – 7y + 86z – 62 = 0
23. The base of the pyramid AOBC is an equilateral triangle OBA with each side equal to 4 2 , ' O ' is the

| |
origin of reference, AO is perpendicular to the plane of ∆ OBC and AO = 2 . Then the cosine of the
angle between the skew straight lines one passing through A and the mid point of OB and the other
passing through O and the mid point of BC is :
1 1 1
(A) − (B) 0 (C) (D)
2 6 2
24. The coplanar points A , B , C , D are (2 − x , 2 , 2) , (2 , 2 − y , 2) , (2 , 2 , 2 − z) and (1 , 1 , 1)
respectively . Then :
1 1 1 1 1 1
(A) + + =1 (B) x + y + z = 1 (C) + + = 1(D) none of these
x y z 1− x 1− y 1− z
→ →
25. Let the centre of the parallelopiped formed by PA = i + 2 j + 2 k ; PB = 4 i − 3 j + k ;

PC = 3 i + 5 j − k is given by the position vector (7, 6, 2). Then the position vector of the point P is:
(A) (3, 4, 1) (B) (6, 8, 2) (C) (1, 3, 4) (D) (2, 6, 8)
→ 1 → →
26. Taken on side A C of a triangle ABC, a point M such that A M = A C . A point N is taken on the
3
→ → → → →
side CB such that BN = CB then, for the point of intersection X of A B & M N which of the following
holds good?
→ 1 → → 1 → → 3 → → →
(A) XB = AB (B) A X = AB (C) X N = M N (D) X M = 3 XN
3 3 4
27. If the acute angle that the vector, α i + β j + γ k makes with the plane of the two vectors
  

2 i + 3 j − k & i − j + 2 k is cot −1 2 then:


(A) α (β + γ) = β γ (B) β (γ + α) = γ α (C) γ (α + β ) = α β (D) α β + β γ + γ α = 0
→ 1
28. Locus of the point P, for which OP represents a vector with direction cosine cos α =
2
( ' O ' is the origin) is:
(A) A circle parallel to y z plane with centre on the x − axis
(B) a cone concentric with positive x − axis having vertex at the origin and the slant
height equal to the magnitude of the vector
(C) a ray emanating from the origin and making an angle of 60º with x − axis

(D) | |
a disc parallel to y z plane with centre on x − axis & radius equal to O P sin 60º
x − x2 y − y2 z − z2
29. Equation of the plane passing through A(x 1, y1, z1) and containing the line = = is
d1 d2 d3
x − x1 y − y1 z − z1 x − x2 y − y2 z − z2
x 2 − x 1 y 2 − y 1 z 2 − z1 x1 − x 2 y1 − y 2 z1 − z 2
(A) =0 (B) =0
FREE Download Study Package from website: www.TekoClasses.com & www.MathsBySuhag.com

d1 d2 d3 d1 d2 d3
x − d1 y − d2 z − d3 x y z
x1 y1 z1 x1 − x 2 y1 − y 2 z1 − z 2
(C) =0 (D) =0

page 62 of 77
x2 y2 z2 d1 d2 d3
30. The equations of the line of shortest distance between the lines
x y z x−2 y −1 z−2
= = and = = are
2 −3 1 3 −5 2
x − ( 62 / 3 ) y + 31 z − (31/ 3)
(A) 3(x – 21) = 3y + 92 = 3z – 32 (B) = =
1/ 3 1/ 3 1/ 3

Teko Classes, Maths : Suhag R. Kariya (S. R. K. Sir), Bhopal Phone : 0 903 903 7779, 0 98930 58881.
x − 21 y + (92 / 3) z − (32 / 3) x−2 y+3 z −1
(C) = = (D) = =
1/ 3 1/ 3 1/ 3 1/ 3 1/ 3 1/ 3
31. A line passes through a point A with p.v. 3 i + j − k & is parallel to the vector 2 i − j + 2 k . If P is a point on
this line such that AP = 15 units, then the p.v. of the point P is:
(A) 13 i + 4 j − 9 k (B) 13 i − 4 j + 9 k (C) 7 i − 6 j + 11 k (D) − 7 i + 6 j − 11 k
32. The equations of the planes through the origin which are parallel to the line
x −1 y+3 z +1 5
= = and distant from it are
2 −1 −2 3
(A) 2x + 2y + z = 0 (B) x + 2y + 2z = 0 (C) 2x – 2y + z = 0 (D) x – 2y + 2z = 0
33. The value(s) of k for which the equation x 2 + 2y2 – 5z2 + 2kyz + 2zx + 4xy = 0 represents a pair of
planes passing through origin is/are
(A) 2 (B) – 2 (C) 6 (D) – 6
x y 2
34. The equation of lines AB is = = . Through a point P(1, 2, 5), line PN is drawn perpendicular
2 −3 6
to AB and line PQ is drawn parallel to the plane 3x + 4y + 5z = 0 to meet AB is Q. Then
 52 78 156   9 
(A) coordinate of N is  , − ,  (B) the coordinates of Q is  3, − , 9 
 49 49 49   2 
x −1 y−2 z−5 x −1 y−2 z−5
(C) the equation of PN is = = (D) the equation of PQ is = =
3 − 176 − 89 4 − 13 8
x − 15 y − 29 z−5
35. Let a perpendicular PQ be drawn from P (5, 7, 3) to the line = = when Q is the
3 8 −5
foot. Then
(A) Q is (9, 13, – 15) (B) PQ = 14
(C) the equation of plane containing PQ and the giv en line is 9x – 4y – z – 14 = 0 (D) none
EXERCISE–6
1. Find the equation of the plane which contains the origin and the line of intersection of the
   
planes r . a = p and r . b = q
x−a y −b z−c x − a′ y − b ′ z − c′
2. If the lines = = and = = intersect at a point then the coordinate of the
a ′ b ′ c ′ a b c
point of intersection.
3. The locus of a point which is a equidistant from the two giv en points with position v ectors
   1     
a and b is the plane  r − (a + b) . ( a – b ) = 0 bisecting the line joining the points normally..
 2 
4. The foot of the perpendicular from (a, b, c) on the line x = y = z is the point (r, r, r) where
3r = a + b + c.
5. Match the following :
Column A Column B
(a) Sum of the square of the direction (P) 0
cosines of line is
(b) All the points on the z-axis have (Q) 1
their x and y coordinate equal to
(c) Distance between the points (1, 3, 2) (R) 9
and (2, 3, 1) is
(d) Shortest distance between the lines (S) 2
x−6 y−2 z−2 x+4 y z +1
= = and = = is
1 −2 2 3 −2 −2
6. Show that the angle between the straight lines whose direction cosines are given by the equations
π 1 1 1
 + m + n = 0 and amn + bn + cm = 0 is if + + = 0.
3 a b c
7. Prove that the two lines whose direction cosines are given by the relations.p + qm + rn = 0 &
a2 + bm 2 + cn2 = 0 are perpendicular if, p2(b + c) + q2 (c + a) + r2 (a + b) = 0 and parallel if
p2 q 2 r 2
+ + =0.
FREE Download Study Package from website: www.TekoClasses.com & www.MathsBySuhag.com

a b c
8. Find the plane π passing through the points of intersection of the planes 2x + 3y− z +1= 0 and
x + y − 2z + 3 = 0 and is perpendicular to the plane 3x − y − 2z = 4. Find the image of point (1, 1, 1) in
plane π.

page 63 of 77
 
9. Given parallel planes r . (2 î − λ ĵ + k̂ ) = 3 and r . (4 î + ĵ − µ k̂ ) = 5 for what values of α, planes
 
r . (µ î − α ĵ + 3 k̂ ) = 0 & r . (α î − 3 ĵ + 2λ k̂ ) = 0 would be perpendicular..
10. The edges of a rectangular parallelepiped are a, b, c; show that the angles between the four diagonals
a 2 ± b2 ± c2
−1
are given by cos .
a 2 + b2 + c2
 

Teko Classes, Maths : Suhag R. Kariya (S. R. K. Sir), Bhopal Phone : 0 903 903 7779, 0 98930 58881.
11. Prove that the line of intersection of the planes r . ( î + 2 ĵ + 3 k̂ ) = 0 and r . (3 î + 2 ĵ + k̂ ) = 0 is

r = t( î − 2 ĵ + k̂ ). Show that the line is equally inclined to î and k̂ and makes an angle
(1/2) sec−1 3 with. ĵ .
x −1 y +1 x +1
12. Find the shortest distance between the lines = =z& = (y − 2); z = 2
2 3 3

13. Show that the line L whose equation is, r = (2 î − 2 ĵ + 3 k̂ ) + λ ( î − ĵ + 4 k̂ ) is parallel to the plane π

whose v ector r . ( î + 5 ĵ + k̂ ) = 5. Find the distance between them.
    2  
A sphere has an equation r − a + r −b
2
14. = 72 where a = î + 3 ĵ − 6 k̂ and b = 2 î + 4 ĵ + 2 k̂ . Find:
(i) the centre of the sphere (ii) the radius of the sphere
(iii)

perpendicular distance from the centre of the sphere to the plane r . 2 î + 2 ĵ − k̂ = − 3. ( )
15. Find the equation of the sphere which is tangential to the plane x − 2y − 2z = 7 at (3, −1, −1) and
passes through the point (1, 1, −3).
16. P1 and P2 are planes passing through origin. L1 and L2 also passes through origin. L1 lies on P1 not on P2 and
L2 lies on P2 but not on P1. Show that there exists points A, B, C and whose permutation A′ .B′.C′ can be
chosen such that [IIT - 2004]
(i)A is on L1, B on P1 but not on L1 and C not on P1.(ii)A′ in on L2, B′ on P2 but not on L2 and C′ not on P2.
17. A parallelopiped ‘S’ has base points A, B, C and D and upper face points A′, B′, C′ and D′. This parallelopiped
is compressed by upper face A′B′C′D′ to form a new parallelopiped ‘T’ having upper face points A′′, B′′, C′′
and D′′. Volume of parallelpiped T is 90 percent of the volume of parallelopiped S. Prove that the locus of ‘A′′’
is a plane. [IIT - 2004]

EXERCISE–5 EXERCISE–6
1. C 2. A 3. A   
4. A 5. D 6. B
1. r . (a q − pb) = 0
7. B 8. B 9. A 2. ( a + a′, b + b′, c + c ′ )
10. D 11. A 12. C 3. True 4. True
13. B 14. B 15. A 5. (a) → (Q), (b) → (P), (c) → (S), (d) → (R)
16. B 17. B 18. A
 12 − 78 57 
19. D 20. A 21. A 8. 7x + 13y + 4z – 9 = 0 ;  , , 
 117 117 117 
22. C 23. D 24. A
3 10
25. A 26. C 27. A 9. α = + 3 12. 13.
59 3 3
28. B 29. AB 30. ABC
31. AB 32. AD 33. BC 8
14. (i) (0, 5, 5) (ii) 9 (iii)
3
34. ABCD 35. BC
15. (x – 2)2 + (y – 1)2 + (z – 1)2 = 5
EXERCISE–7
Part : (A) Only one correct option
 
FREE Download Study Package from website: www.TekoClasses.com & www.MathsBySuhag.com

   
1. The lengths of the diagonals of a parallelogram constructed on the vectors p = 2 a + b & q = a − 2 b ,
 
where a & b are unit vectors forming an angle of 60º are:
(A) 3 & 4 (B) 7 & 13 (C) 5 & 11 (D) none

page 64 of 77
2
   
2.  a − b  =
 a 2 2
 b 

  2
 a a − b b 
2
 
2 2 a − b
(B)     (C)  

Teko Classes, Maths : Suhag R. Kariya (S. R. K. Sir), Bhopal Phone : 0 903 903 7779, 0 98930 58881.
(A) a − b   (D) none
a b  a b 
   
   
3. A, B, C & D are four points in a plane with pv's a , b , c & d respectively such that

( 
)( ) ( )
     
a − d . b − c = b − d . ( c − a ) = 0. Then for the triangle ABC, D is its:
(A) incentre (B) circumcentre (C) orthocentre (D) centroid
  2π
{( ) ( )}
      2
4. Vectors a & b make an angle θ = . If a = 1, b = 2 then a + 3 b x 3 a − b =
3
(A) 225 (B) 250 (C) 275 (D) 300
   
5. a , a , a , a
Consider a tetrahedron with faces f 1, f 2, f 3, f 4. Let 1 2 3 4 be the vectors whose magnitudes are
respectively equal to the areas of f 1, f 2, f 3, f 4 & whose directions are perpendicular to these faces in the
outward direction. Then,
           
(A) a 1 + a 2 + a 3 + a 4 = 0 (B) a 1 + a 3 = a 2 + a 4 (C) a 1 + a 2 = a 3 + a 4 (D) none
        
6. For non−zero vectors a , b , c , a x b . c = a b c holds if and only if;
                 
(A) a . b = 0, b . c = 0 (B) c . a = 0, a . b = 0 (C) a . c = 0, b . c = 0 (D) a . b = b . c = c . a = 0
     
a .a a .b a.c
      
 
7. If a = i + j + k , b = i − j + k , c = i + 2 j − k , then the value of b . a b . b b.c =
     
c.a c.b c.c
(A) 2 (B) 4 (C) 16 (D) 64
        
8. ( ) ( )
If a , b & c are any three vectors, then a × b × c = a × b × c is true if:
     
(A) b & c are collinear (B) a & c are collinear (C) a & b are collinear (D) none
  
9. ( r . i) (i × r) + ( r . j) ( j × r) + ( r . k) ( k × r) =
  
(A) 0 (B) r (C) 2 r (D) 3 r
: ,
10. A point taken on each median of a triangle divides the median in the ratio 1 3 reckoning from the vertex.
Then the ratio of the area of the triangle with vertices at these points to that of the original triangle is:
(A) 5: 13 (B) 25: 64 (C) 13: 32 (D) none
→ → → → →
11. Given a parallelogram ABCD. If AB = a, AD = b & AC = c, then DB . AB has the value:

3 a 2 + b 2 − c2 a 2 + 3 b 2 − c2 a 2 − b 2 + 3 c2
(A) (B) (C) (D) none
2 2 2
12. The points whose position vectors are p i + q j + r k ; q i + r j + p k & r i + p j + q k are collinear if:
(A) p + q + r = 0 (B) p2 + q2 + r2 − pq − qr − rp = 0
(C) p3 + q3 + r3 − 3 pqr = 0 (D) none of these
        
13. If p & s are not perpendicular to each other and r x p = q x p & r . s = 0, then r =
   
    q . s    q . p   
(A) p . s (B) q −     p (C) q +     p (D) q + µ p for all scalars µ
 p . s  p . s
14. If a, b, c are pth, qth, rth terms of an H.P. and
  
     i j k
u = (q − r) i + (r − p) j + (p − q) k , υ = + + , then:
a b c
  
(A) u , υ are parallel vectors (B) u , υ are orthogonal vectors
      
(C) u . υ = 1 (D) u × υ = i + j + k
If p , q are two noncollinear and nonzero vectors such that (b − c ) p × q + (c − a ) p + (a − b )q = 0 ,
     
15.
where a, b, c are the length of the sides of a triangle, then the triangle is
(A) right angled (B) obtuse angled (C) equilateral (D) isoceles
FREE Download Study Package from website: www.TekoClasses.com & www.MathsBySuhag.com

16. If cos α i + j + k , i + cos β j + k & i + j + cos γ k (α ≠ β ≠ γ ≠ 2 n π) are coplanar then the value of
 2α 2β 2 γ
cos ec 2 + cos ec 2 + cos ec 2  =
 

page 65 of 77
(A) 1 (B) 2 (C) 3 (D) none of these
           
17. If r x b = cx b & r . a = 0 where a = 2 i + 3 j − k b = 3 i − j + k & c = i + j + 3 k , then r is equal to:
   ,   

(
(A) 2 i − j + k ) ( )
(B) 2 i + j − k (C) 2 − i + j + k
  
(
(D) 2 i + j + k
  
) ( )
[ ] [ ] [ ] [ ]
       
18. The value of d b c a + d c a b + d a b c − d a b c is equal to:

[ ]
     
[ ]

Teko Classes, Maths : Suhag R. Kariya (S. R. K. Sir), Bhopal Phone : 0 903 903 7779, 0 98930 58881.
(A) 0 (B) 2 a b c d (C) – 2 a b c d (D) none of these
19. The three vectors i + j , j + k , k + i taken two at a time form three planes. The three unit vectors
drawn perpendicular to these three planes form a parallelopiped of volume:
1 3 3 4
(A) (B) 4 (C) (D)
3 4 3 3
→ → → → → →
20. For any four points P, Q, R, S, P Q × RS − Q R × P S + R P × Q S is equal to 4 times the area of the
triangle:
(A) PQR (B) QRS (C) PRS (D) PQS
     
21. If a , b , c are three non − coplanar & p , q , r are reciprocal vectors, then:
 → → →
  → → →

 a + m b + n c .   p + mq + n r is equal to:
   
(A) 2 + m 2 + n2 (B)  m + m n + n  (C) 0 (D) none of these

 → ∧ → 2π → →
22. In a quadrilateral ABCD, A C is the bisector of the  A B

A D which is
 3
, 15 A C = 3 A B | | | |
→  → ∧ →

| |
= 5 A D then cos  BA

C D is:

14 21 2 2 7
(A) − (B) − (C) (D)
7 2 7 3 7 14
→ →
23. | | | |
In the isosceles triangle ABC A B = BC = 8, a point E divides AB internally in the ratio 1: 3, then the
→ → →
cosine of the angle between C E & CA is (where CA = 12) | |
3 7 3 8 3 7 −3 8
(A) − (B) (C) (D)
8   17 8 17
         
24. If p = 3 a − 5 b ; q = 3 a + b ; r = a + 4 b ; s = − a + b are four vectors such that

( ) ( ) ( )
 ∧  
q = 1 and r ∧ s = 1 then cos a ∧ b is:
  
sin p

19 19
(A) − (B) 0 (C) (D) 1
  
5 43 5 43
25. If p, q, r be three mutually perpendicular vectors of the same magnitude. If a vector x satisfies the
equation p × ( (x − q) × p ) + q × ( (x − r ) × q ) + r ( (x − p ) × r ) = 0 , then x is given by [IIT - 1997]
            

1    1   
(p + q − 2r ) (p + q + r ) (p + q + r ) (2p + q − r )
1    1   
(A) (B) (C) (D)
2 2 3 3

( )
         
26. Let a & b be two non−collinear unit vectors. If u = a − a . b b & v = a x b , then v is [IIT - 1999]

( )
        
(A) u (B) u + u . a (C) u + u . b
(D) u + u . a + b
   
     b.a    b.a 
27. If a , b , c are three non-zero, non coplanar vectors and b1 = b –  2 a , b 2 = b +  2 a ,
|a| |a|
           
  c.a  b.c    c.a  b1.c    c . a  b .c 
c1 = c –  2 a +  2 b1 , c 2 = c –  2 a –  2 b1 , c 3 = c –  2 a +  2 b1 ,
|a| |c| |a| | b1 | |c| |c|
 
FREE Download Study Package from website: www.TekoClasses.com & www.MathsBySuhag.com

 
  c.a  b.c 
c 4 = c –  2 a –  b1 , then the set of orthogonal vectors is [IIT - 2005]
|c| | b |2
           
(A) (a, b1, c 3 ) (B) (a, b1, c 2 ) (C) (a, b1, c 1 ) (D) (a, b 2 , c 2 )

page 66 of 77

28. Let A be vector parallel to line of intersection of planes P1 and P2 through origin, P1 is parallel to the vectors
2 î + 3 k̂ and 4 ĵ – 3 k̂ and P2 is parallel to ĵ – k̂ and 3 î + 3 ĵ , then the angle between vector

A and 2 î + ĵ – 2 k̂ is [IIT - 2006]
π π π 3π
(A) (B) (C) (D)
2 4 6 4

Teko Classes, Maths : Suhag R. Kariya (S. R. K. Sir), Bhopal Phone : 0 903 903 7779, 0 98930 58881.
Part : (B) May have more than one options correct
       
29. If a , b , c & d are linearly independent set of vectors & K1 a + K 2 b + K 3 c + K 4 d = 0 then:
(A) K1 + K2 + K3 + K4 = 0 (B) K1 + K3 = K2 + K4 = 0 (C) K1 + K4 = K2 + K3 = 0 (D) none of these
  
30. Given three vectors a , b , c such that they are non − zero, non − coplanar vectors, then which of the
following are coplanar.
                       
(A) a + b , b + c , c + a (B) a − b , b + c , c + a (C) a + b , b − c , c + a (D) a + b , b − c , c − a
       
31. Let p = 2 i + 3 j + a k , q = b i + 5 j − k & r = i + j + 3 k . If p , q , r are coplanar and p . q = 20, a & b
have the values:
(A) 1, 3 (B) 9, 7 (C) 5, 5 (D) 13, 9
     
32. If z1 = a i + b j & z 2 = c i + d j are two v ectors in i & j system where z1 = z 2 = r & z1 . z 2 = 0
 
then w1 = a i + c j & w 2 = b i + d j satisfy:
   
(A) w1 = r (B) w 2 = r (C) w1 . w 2 = 0 (D) none of these
     
33. If a & b are two non colinear unit vectors & a , b , x a − y b form a triangle, then:
  ∧ 
   a b
(A) x = − 1; y = 1 &  a + b  = 2 cos  
 2 
 
∧  ∧   
  
( )
 
(B) x = − 1; y = 1 & cos  a b +  a + b  cos a , − a + b  = − 1
   
  ∧    ∧ 
   a b  a b
(C)  a + b  = − 2 cot   cos   & x = − 1, y = 1 (D) none
 2   2 
   
The value(s) of α ∈ [0, 2π] for which vector a = i + 3 j + (sin 2α ) k makes an obtuse angle with the

34.
 α  α
k and c = ( tan α ) i + ( tan α ) j − 3 cos ec k are

Z-axis and the vectors b = (tan α ) i − j + 2 sin
2 2
orthogonal, is/are:
(B) π − tan −1 2
(A) tan −1 3
 (C)π + tan 3 (D) 2 π − tan −1 2
−1

35. A parallelogram is constructed on the vectors p & q . A vector which coincides with the altitude of the
  
parallelogram & perpendicular to the side p expressed in terms of the vectors p & q is:
( p x q) x p p x ( p x q)
         
 q.p  q.p  
(A) q −  2 p (B)  (C)  2 p − q (D) 
( p) p2 p p2
36. Identify the statement(s) which is/are incorrect?

(A) [ ( )] ( ) (
   
a x a× a×b = a×b a
  
  2
)
      
(B) If a , b , c are non coplanar vectors and v . a = v . b = v . c = 0 then v must be a null vector
   
(C) If a and b l ie i n a pl ane norm al to t he pl ane c ontai ni ng t he v ec t ors c and d
( )(
   
then a × b x c × d = 0 )
           
(D) If a , b , c and a ′ , b ′ , c′ are reciprocal system of vectors then a . b ′ + b . c′ + c . a ′ = 3

( )
    
37. If a = i + 2 j + 4 k , b = 2 i − 3 j + k , c = i + 4 j − 4 k , then the vector a × b × c is orthogonal to:
     
(A) a (B) b (C) c (D) a + b + c
   
38. If a , b , c are non-zero, non-collinear vectors such that a vector p = a b

( )) ( ( ))
FREE Download Study Package from website: www.TekoClasses.com & www.MathsBySuhag.com

cos 2 π − a (
 ∧      
b c and a vector q = a c cos π − a ∧ c b then p + q is


  
(A) parallel to a (B) perpendicular to a (C) coplanar with b & c (D) none of these

page 67 of 77
39. Which of the following statement(s) is/are true?
[ ]
       
(A) If n . a = 0, n . b = 0 & n . c = 0 for some non zero vector n , then a b c = 0
(B) there exist a vector having direction angles α = 30º & β = 45º
(C) locus of point for which x = 3 & y = 4 is a line parallel to the z - axis whose distance from the
z axis is 5
(D) the vertices of a regular tetrahedron are OABC where ' O ' is the origin. The vector
→ → →
OA + O B + OC is perpendicular to the plane ABC.

Teko Classes, Maths : Suhag R. Kariya (S. R. K. Sir), Bhopal Phone : 0 903 903 7779, 0 98930 58881.
40. In a ∆ ABC, let M be the mid point of segment AB and let D be the foot of the bisector of ∠ C. Then the
Area ∆ CDM
ratio is:
Area ∆ ABC
1 a−b 1 a−b 1 A−B A+B 1 A−B A+B
(A) (B) (C) tan cot (D) cot tan
4 a+b 2 a+b 2 2 2 4 2 2
        ,
41. The vectors a , b , c are of the same length & pairwise form equal angles. If a = i + j & b = j + k the

pv's of c can be:
 4 1 4 1 4 1  1 4 1
(A) (1, 0, 1) (B)  − , , −  (C)  , − ,  (D)  − , , − 
 3 3 3 3 3 3  3 3 3
EXERCISE–8
1. Through the middle point M of the side AD of a parallelogram ABCD the straight line BM is drawn
cutting AC at R and CD produced at Q prove that QR = 2RB
  
2. Show that the perpendicular distance of the point c from the line joining a & b is,
     
b× c + c×a + a× b
  .
b−a
( )
        
3. If α = i + 2 j + 3 k ; β = 2 i − j + k ; γ = 3 i + 2 j + k and α × β × γ = p α + q β + r γ then find the values
of p, q, r
   
4. If a = 2 i − 3 j + k , b = i − j + 2 k , c = 2 i + j − k & d = 3 i − j − 2 k
 , then find the value of
    
( a x b ) x ( a x c ). d
((q × c) × (p × b)) = b × ((p × c) × (q × a)) + c × ((p × a ) × (q × b))
   
5. Show that a ×
 bx c  cx a  axb   
6. It is giv en that x =    ; y =    ; z =    where a , b , c are non − coplanar vectors. Show
[a b c] [ a b c] [a b c]
  
that x , y , z also forms a non − coplanar system. Find the value of
        
x .(a + b) + y .( b + c ) + z .( c + a ) .
7. The median AD of a triangle ABC is bisected at E and BE is produced to meet the side AC in F. Prove
that AF = (1/3) AC and EF = (1/4) BF.
8. Points X and Y are taken on the sides QR and RS, respectively of a parallelogram PQRS, so that
QX = 4XR and RY = 4YS. The line XY cuts the line PR at Z. Find the ratio PZ: ZR.
  
9. Forces P , Q act at O & have a resultant R . If any transversal cuts their line of action at A,B,C respectively,,
P Q R
then show that + = .
OA OB OC
10. In a tetrahedron, if two pairs of opposite edges are perpendicular, then show that the third pair of
opposite edges is also perpendicular & in this case the sum of the squares of two opposite edges is
the same for each pair. Also show that the segment joining the mid points of opposite edges bisect one
another.
11. Use vectors to prove that the diagonals of a trapezium having equal non parallel sides are equal &
conversely.
      
12. Given four non zero vectors a , b , c and d . The vectors a , b & c are coplanar but not collinear pair by
    ∧ ∧ π ∧ ∧
pair and vector d is not coplanar with vectors a , b & c and ( a b ) = ( b c ) = , (d a ) = α, (d b) =β ,
∧
3
c) = cos −1 (cos β − cos α) .
FREE Download Study Package from website: www.TekoClasses.com & www.MathsBySuhag.com

prove that ( d
  
13. If p , q & r are three non-coplanar vectors, prove that,
  

page 68 of 77
p q r
  1      
a×b =       p . a q . a r .a
[q × r , r × p , p × q] 
p.b

q.b
 
r .b
   
14. Consider the non zero vectors a , b , c & d such that no three of which are coplanar then prove

[ ] [ ] [ ] [ ]
       
   
that a b c d + c a b d = b a c d + d a b c . Hence prove that a , b , c & d represent the position vectors

Teko Classes, Maths : Suhag R. Kariya (S. R. K. Sir), Bhopal Phone : 0 903 903 7779, 0 98930 58881.
[ b cd] + [ a b d]
    

   = 1.
[ a cd] + [ a b c]
of the vertices of a plane quadrilateral if and only if   

15.
       
( )   
Solve the following equation for the vector p ; p x a + p . b c = b x c where a , b , c are non zero non coplanar

[  ]

   a b c 
 
vectors and a is neither perpendicular to b nor to c , hence show that p x a +   c is perpendicular

 a .c 
   
to b − c .
     
16. If a, b, c & a', b', c' are reciprocal system of vectors then prove that:
  
            a + b +c
(i) [a b c] [a' b' c' ] = 1 (ii) (a' x b' )+(b' x c' ) + (c' x a' ) =  .
[abc ]
         
17. Let A = 2i + k; B = i + j + k & C = 4i − 3j + 7k. Determine a vector R satisfying R x B = C x B & R . A = 0
 
18. For any two vectors u & v , prove that [IIT - 1998]
             
(a) ( u. v) 2 + | u × v|2 = | u|2 | v|2 & (b) (1 + | u|2 )(1 +| v|2 ) = (1 − u. v)2 + | u + v + ( u × v)|2
19. Let ABC and PQR be any two triangles in the same plane. Assume that the perpendiculars from the
points A, B, C to the sides QR, RP, PQ respectively are concurrent. Using vector methods or otherwise,
prove that the perpendiculars from P, Q, R to BC, CA, AB respectively are also concurrent. [IIT - 2000]
 
20. Find 3 – dimensional vectors v 1, v 2 , v 3 satisfying
           
v 1 . v 1 = 4, v 1 . v 2 = –2, v 1 . v 3 = 6, v 2 . v 2 = 2, v 2 . v 3 = –5, v 3 . v 3 = 29. [IIT - 2001]

21. If û , v̂ , ŵ be three non-coplanar unit vectors with angles between û & v̂ is α, between v̂ & ŵ is β
  
and between ŵ & û is γ . If a , b , c are the unit vectors along angle bisectors of α, β , γ respectively,,

then prov e that, [ a × b , b × c , c × a ] = 161 [ û v̂ ŵ ]2 sec  α2  sec  β2  sec  2γ  .
2 2 2
[IIT - 2003]

EXERCISE–7
1. B
7. C
2. B
8. B
3. C
9. A
4. D
10. B
5. A
11. A
6. D
12. B
EXERCISE–8
3. p = 0; q = 10; r = − 3
13. B 14. B 15. C 16. B 17. C 18. A
4. – 98 6. 3
19. D 20. B 21. A 22. C 23. C 24. C
  
25. B 26. A 27. B 28. D 29. ABC 20. v 1 = 2 î , v 2 = − î ± ĵ , v 3 = 3 î ± 2 ĵ ± 4k̂ are some
30. BCD 31. AD 32. ABC 33. AB 34. BD possible values
35. BD 36. ACD 37. AD 38. BC 39. ACD
40. BC 41. AD
DOWNLOAD FREE FROM www.tekoclasses.com , PH.: 0 903 903 7779, 0 98930 58881
MATHS H.O.D.: SUHAG R.KARIYA , BHOPAL, VECTORS & 3D PART 6 OF 6

Assertion- Reason
Some questions (Assertion–Reason type) are given below. Each question contains Statement – 1 (Assertion)
and Statement – 2 (Reason). Each question has 4 choices (A), (B), (C) and (D) out of which ONLY ONE is
correct. So select the correct choice :Choices are :
(A)Statement – 1 is True, Statement – 2 is True; Statement – 2 is a correct explanation for Statement – 1.
(B)Statement – 1 is True, Statmnt – 2 is True; Statement – 2 is NOT a correct explanation for Statement – 1.
(C) Statement – 1 is True, Statement – 2 is False.
(D) Statement – 1 is False, Statement – 2 is True.
452. Let a, b, c be three non-coplanar vectors then ( b − c).[(c − a) × (a − b)] = 0
Statement 1: b − c can be expressed as linear combination of c − a and a − b .
Statement 2: Given non-coplanar vectors one vector can be expressed as a linear combination of other two.
453. A vector has components p and 1 with respect to a rectangular cartesian system. If the axes are rotated through an
angle α about the origin in the anticlockwise sense.
Statement–1 : If the vector has component p + 2 and 1 with respect to the new system then p = –1
Statement–2 : Magnitude of vector original and new system remains same
 
Let | a | = 4, | b | = 2 and angle between a and
 
454. b is π/6
  2   2 2
Statement–1 : (a × b) = 4 Statement–2 : (a × b) = | a |


Statement–1 :  b × c c × a a × b  = 0
     
455.
  
Statement–2 : If a , b , r are linearly dependent vectors then they are coplanar.
    
Statement–1 : If a + b = a − b then a is parallel to b .

456.
    
Statement–2 : If a + b = a − b then a.b = 0.
     
Let r be a non-zero vector satisfying r.a = r.b = r.c = 0 for given non–zero vectors a, b and c .
 
457.
 
Statement–1 : a, b and c are coplanar vectors.

   
Statement–2 : r is perpendicular to the vectors a, b and c .
 
458. Let a and r be two non–collinear vectors.
Statement–1 : vector a × ( a × r ) is a vector in the plane of a and r , perpendicular to a .
     

( )
    
Statement–2 : a × a × b = 0 , for any vector b .
  
459. Statement–1 : If three points P, Q, R have position vectors a , b , c respectively and
  
2a + 3b − 5c = 0  , then the points P, Q, R must be collinear. Statement–2 :
  If for three
points A, B, C; AB = λ AC , then the points A, B, C must be collinear.
( )
       
Statement–1 : Let a and b be two non collinear unit vectors. If u = a − a.b b and v = a × b

460.
then | v |=| u | .
 

1 ˆ ˆ ˆ π
Statement–2 : The vector ( 3
)
2i − 2 j + k is makes an angle of
3
with the vector

( )
5iˆ − 4ˆj + 3kˆ .
  
461. Statement-1: If u & v are unit vectors inclined at an angle α and x is a unit vector bisecting the angle between
 u+v
 
them, then x =
α
2 cos
2
Statement-2: If ∆ABC is an isosceles triangle with AB = AC = 1, then vector representing bisector of angle A is
 
 AB + AC
given by AD =
2
69 of 77
DOWNLOAD FREE FROM www.tekoclasses.com , PH.: 0 903 903 7779, 0 98930 58881
MATHS H.O.D.: SUHAG R.KARIYA , BHOPAL, VECTORS & 3D PART 6 OF 6

462. Statement-1: The direction ratios of line joining origin and point (x, y, z) must be x, y, z.
x y z
Statement-2: If P is a point (x, y, z) in space and OP = r, then direction cosines of OP are , , .
r r r
463. Statement-1: If the vectors 2iˆ − ˆj + kˆ , ˆi + 2ˆj − 3kˆ and 3iˆ − λˆj + 5kˆ are coplanar, then |λ|2 is equal to 16.
    
Statement-2: The vectors a , b and c are coplanar iff a, (b × c ) = 0

464. Statement-1: A line L is perpendicular to the plane 3x – 4y + 5z = 10
3 4 1
Statement-2: Direction co-sines of L be < ,− , >
5 2 5 2 2
        
465. Statement-1 : The points with position vectors a − 2b + 3c, − 2a + 3b − c , 4a − 7b + 7c are collinear.
        
Statement-2: The position vectors a − 2b + 3c, − 2 a + 3b − c, 4a − 7b + 7c are linearly dependent vectors.
      1  
466. Statement-1: If a, b, c are three unit vectors such that a × (b × c) = b then the angle between a & b is π/2
2
  1  
Statement-2: If a × (b × c) = b, then a .b = 0.
 
2
467. Statement-1: If cosα, cosβ, cosγ are the direction cosine of any line segment, cos2α + cos2β + cos2γ = 1.
Statement-2: If cosα, cosβ, cosγ are the direction cosine of line segment,
cos2α + cos2β + cos2γ = −1.
468. Statement-1: The direction cosines of one of the angular bisector of two intersecting lines having direction
cosines as l1 , m1, n1, & l2, m2, n2 is proportional to l1 + l2, m1+ m2, n1 + n2.
Statement-2: The angle between the two intersecting lines having direction cosines as l1, m1, n1 & l2, m2, n2 is
given by cosθ = l1 l2 + m1m2 + n1n2.
      
Statement-1: If a.b = 0 ⇒ Statement-2: a.b = 0 ⇒ either a = 0 or b = 0 or a ⊥ b

469. a ⊥b
   
470. Statement-1: A × B = B × A
   
Statement-2: A × B =| A || B | sinθ n̂ , when θ is angle, when your fingers curls from A to B
471. Statement-1 : A vector ⊥r the plane of (1, -1, 0), (2, 1, -1) & (-1, 1, 2) is 6iˆ + 6kˆ
   
Statement-2 : A × B always gives a vector perpendicular to plane of A & B
Statement-1 : Angle between planes r.n1 = q1 & r.n 2 = q 2 .
   
472.
 
(acute angle) is given by cosθ = n1 .n 2
Statement-2 : Angle between the planes in same as acute angle formed by their normals.
  
473. Statement-1: In ∆ABC, AB + BC + CA = 0
      
Statement-2 : If OA = a, OB = b then AB = a + b
       
474. Statement-1: a = 3i + pj + 3k and b = 2i + 3 j + qk are parallel vectors it p = 9/2 and q = 2.
       a a a
Statement-2 : If a = a1 i + a 2 j + a 3 k and b = b1 i + b 2 j + b3 k are parallel 1 = 2 = 3

b1 b 2 b3
475. Statement-1: The direction ratios of line joining origin and point (x, y, z) must be x, y, z
x y z
, ,
Statement-2: If P is a point (x,y, z) in space and OP = r then directions cosines of OP are
r r r 
  | [a − c bd] |
Statement-1: The shortest distance between the skew lines r = a + αb and r = c + β d is
   
476. 
| b×d |
Statement-2: Two lines are skew lines if three axist no plane passing through them.
Statement-1: a = ˆi + pjˆ + 2kˆ , b = 2iˆ + 3jˆ + qkˆ are parallel vectors of p = 3/2 and q = 4.
 
477.
a1 a 2 a 3
Statement-2: a = a1ˆi + a 2 ˆj + a 3 kˆ and b = b1ˆi + b 2 ˆj + b3 kˆ are parallel if

= = .

b1 b 2 b3

70 of 77
DOWNLOAD FREE FROM www.tekoclasses.com , PH.: 0 903 903 7779, 0 98930 58881
MATHS H.O.D.: SUHAG R.KARIYA , BHOPAL, VECTORS & 3D PART 6 OF 6

Statement-1: If a = 2iˆ + k,
ˆ b = 3jˆ + 4kˆ and c = 8iˆ − 3jˆ are coplanar then c = 4a − b .
     
478.
   
Statement-2: A set of vectors a1 , a 2 ...a n is said to be linearly independent if every relation of the form l1 a 1 + l2
 
a 2 + … + ln a n = 0 implies that l1 = l2 = …. = ln = 0 (scalars).
   
  (a − c).(b × d)
Statement-1: The shortest distance between the skew lines r = a + αb and r = c + β d is
   
479.  
| b×d |
Statement-2: Two lines are skew lines if there exists no plane passing through them.
480. Statement-1: The curve which is tangent to a sphere at a given point is the equation of a plane.
Statement-2: Infinite number of lines touch the sphere at a given point.
   
481. Statement-1: In ∆ABC AB + BC + CA = O
      
Statement-2: If OA = a, OB = b, then AB = a + b (∆ law of addition).
3
Statement-1: a = ˆi + pjˆ + 2kˆ and b = 2iˆ + 3jˆ + qkˆ are parallel vectors if P = , q = 4
 
482.
2
a a a
Statement-2: If a = a1ˆi + a 2 ˆj + a 3 kˆ and b = b1ˆi + b 2 ˆj + b3 kˆ are parallel then 1 = 2 = 3 .
 
b1 b 2 b3
Statement-1: If a = 2iˆ + kˆ a , a , a .....a , b = 3jˆ + 4kˆ and c = 8iˆ − 3jˆ are coplanar then c = 4a − b
         
483. 1 3 3 n
Statement-2: A set of vectors is said to be linearly independent if every relation of the form
l1a + l2a 2 + ..... + ln a n = 0 ⇒ l1 = l2 = ….. = ln = 0.
  
 
r = a1 + αb1 and r = a 2 + βb 2 is
   
484. Statement-1: The shortest distance between the skew lines
   
[b1 b 2 (a 2 − a1 )]
  Statement-2: Two lines are skew lines if there exists no plane passing through them.
(b1 × b 2 )
485. Statement-1 : The value of expression ˆi(ˆj × k)
ˆ + ˆj(kˆ × ˆi) + k(i
ˆ ˆ × ˆj) = 3
Statement-2 : ˆi(ˆj × k)
ˆ = [i.j.k]
ˆ ˆ ˆ =1
 
     a×b
Statement-1: A relation between the vectors r, a and b is r × a = b ⇒ r =   Statement-2 : r.a = 0
  
486.
a.a
3-Dimension
x −1 y + 3 z − 2 x − 2 y −1 z + 3
487. The equation of two straight line are = = and = =
2 1 −3 1 −3 2
Statement–1 : The given lines are coplanar
Statement–2 : The equation 2x1 – y1 = 1, x1 + 3y1 = 4, 3x1 + 2y1 = 5 are consistent.
488. Statement–1 : The distance between the planes 4x – 5y + 3z = 5 and 4x – 5y + 3z + 2 = 0 is
3
. Statement–2 The distance between ax + by + cz + d1 = 0 and ax + by + cz + d2
5 2
d1 − d 2
= 0 is .
a 2 + b2 + c2
x −1 y + 1 z − 3
489. Given the line = = and the plane π : x - 2y - z = 0
3 2 −1
Statement-1: L lies in π Statement-2: L is parallel to π
x y −1 z − 2
490. The image of the point (1, b, 3) in the Statement-1: Line = = will be (1, 0, 7)
1 2 3

71 of 77
DOWNLOAD FREE FROM www.tekoclasses.com , PH.: 0 903 903 7779, 0 98930 58881
MATHS H.O.D.: SUHAG R.KARIYA , BHOPAL, VECTORS & 3D PART 6 OF 6

Statement-2: Length of the perpendicular from the point A( α ) on the line r = a + tb, is given by d =
 

| (a − α) × b |
|b|

Answer
452. C 453. A 454. D 455. D 456. D 457. A 458. C
459. A 460. C 461. A 462. A 463. A 464. A 465. A
466. A 467. B 468. B 469. D 470. D 471. A 472. A
473. C 474. A 475. A 476. B 477. A 478. B 479. B
480. A 481. C 482. A 483. B 484. B 485. A 486. A
487. A 488. D 489. C 490. B

Que from Compt. Exams


Co-ordinate Geometry of Three Dimensions
1. The direction cosines of a line segment AB are − 2 / 17 , 3 / 17 , − 2 / 17 . If AB = 17 and the co-ordinates of A are (3, –
6, 10), then the co-ordinates of B are
(a) (1, –2, 4) (b) (2, 5, 8) (c) (–1, 3, –8) (d) (1, – 3, 8)
2. The projection of any line on co-ordinate axes be respectively 3, 4, 5 then its length is [MP PET 1995; RPET 2001]
(a) 12 (b) 50 (c) 5 2 (d) None of these
3. If centroid of the tetrahedron OABC , where A, B, C are given by (a , 2, 3),(1, b , 2) and (2, 1, c) respectively be (1, 2, –1),
then distance of P(a, b, c) from origin is equal to
(a) 107 (b) 14 (c) 107 / 14 (d) None of these
4. If P ≡ (0, 1, 0 ), Q ≡ (0, 0, 1) , then projection of PQ on the plane x + y + z = 3 is [EAMCET 2002]

(a) 3 (b) 3 (c) 2 (d) 2


5. The points A(4 , 5, 1), B(0,−1,−1), C(3, 9, 4 ) and D(−4 , 4 , 4 ) are
[Kurukshetra CEE 2002]
(a) Collinear (b) Coplanar (c) Non- coplanar
(d) Non- Collinear and non-coplanar
6. The angle between two diagonals of a cube will be [MP PET 1996, 2000; RPET 2000, 02; UPSEAT 2004]
(a) sin −1 1 / 3 (b) cos −1 1 / 3 (c) Variable (d) None of these
x − 8 y + 19 z − 10
7. The equations of the line passing through the point (1,2,–4) and perpendicular to the two lines = = and
3 − 16 7
x − 15 y − 29 z −5
= = , will be [AI CBSE 1983]
3 8 −5
x −1 y − 2 z + 4 x −1 y − 2 z + 4 x −1 y − 2 z + 4
(a) = = (b) = = (c) = = (d) None of these
2 3 6 −2 3 8 3 2 8
8. If three mutually perpendicular lines have direction cosines (l1 , m 1 , n 1 ), (l 2 , m 2 , n 2 ) and (l 3 , m 3 , n 3 ) , then the line having direction
cosines l1 + l 2 + l 3 , m 1 + m 2 + m 3 and n1 + n 2 + n 3 make an angle of ..... with each other
(a) 0 ° (b) 30 ° (c) 60 ° (d) 90 °
9. The straight lines whose direction cosines are given by al + bm + cn = 0, fmn + gnl + hlm = 0 are perpendicular, if
f g h a b c a b c
(a) + + =0 (b) + + = 0 (c) af = bg = ch (d) = =
a b c f g h f g h
10. If the straight lines x = 1 + s, y = −3 − λs, z = 1 + λs and x = t / 2, y = 1 + t, z = 2 − t , with parameters s and t respectively,
are co-planar, then λ equals [AIEEE 2004]
(a) 0 (b) –1 (c) –1/2 (d) –2
11. The co-ordinates of the foot of perpendicular drawn from point P(1, 0, 3) to the join of points A(4 , 7, 1) and B(3, 5, 3) is [RPET 01]
 5 7 17  2 5 7 5 2 7
(a) (5, 7, 1) (b)  , ,  (c)  , ,  (d)  , , 
3 3 3  3 3 3 3 3 3
x −1 y +1 z −1 x −3 y −k z
12. If the lines = = and = = intersect, then k = [IIT Screening 2004]
2 3 4 1 1 1

72 of 77
DOWNLOAD FREE FROM www.tekoclasses.com , PH.: 0 903 903 7779, 0 98930 58881
MATHS H.O.D.: SUHAG R.KARIYA , BHOPAL, VECTORS & 3D PART 6 OF 6

2 9
(a) (b) (c) 0 (d) None of these
9 2
13. A square ABCD of diagonal 2a is folded along the diagonal AC so that the planes DAC and BAC are at right angle. The
shortest distance between DC and AB is [Kurukshetra CEE 1998]
(a) 2a (b) 2a / 3 (c) 2 a / 5 (d) ( 3 / 2)a
14. A line with direction cosines proportional to 2,1, 2 meets each of the lines x = y + a = z and x + a = 2 y = 2 z . The co-ordinates
of each of the points of intersection are given by [AIEEE 2004]
(a) (2 a, a, 3 a), (2 a, a, a) (b) (3 a, 2 a, 3 a), (a, a, a) (c) (3 a, 2 a, 3 a), (a, a, 2a) (d) (3 a, 3 a, 3 a), (a, a, a)
15. The equation of the planes passing through the line of intersection of the planes 3 x − y − 4 z = 0 and x + 3 y + 6 = 0 whose
distance from the origin is 1, are
(a) x − 2 y − 2 z − 3 = 0 , 2 x + y − 2 z + 3 = 0 (b) x − 2y + 2z − 3 = 0 , 2 x + y + 2z + 3 = 0
(c) x + 2 y − 2 z − 3 = 0 , 2 x − y − 2 z + 3 = 0 (d) None of these
16. The co-ordinates of the points A and B are (2, 3, 4) (–2, 5,– 4) respectively. If a point P moves so that PA 2 − PB 2 = k where k is a
constant, then the locus of P is
(a) A line (b) A plane (c) A sphere (d) None of these
17. The equation of the plane passing through the points (1,−3,−2) and perpendicular to planes x + 2 y + 2 z = 5 and
3 x + 3 y + 2 z = 8 , is [AISSE 1987]
(a) 2 x − 4 y + 3 z − 8 = 0 (b) 2 x − 4 y − 3 z + 8 = 0 (c) 2x + 4 y + 3z + 8 = 0 (d) None of these
18. A variable plane at a constant distance p from origin meets the co-ordinates axes in A, B, C . Through these points planes are
drawn parallel to co-ordinate planes. Then locus of the point of intersection is
1 1 1 1 1 1 1
(a) + + = 2 (b) x 2 + y 2 + z 2 = p 2 (c) x + y + z = p (d) + + =p
x 2 y2 z2 p x y z
19. P is a fixed point (a, a, a) on a line through the origin equally inclined to the axes, then any plane through P perpendicular to OP,
makes intercepts on the axes, the sum of whose reciprocals is equal to
3 3a
(a) a (b) (c) (d) None of these
2a 2
20. The equation of the plane through the intersection of the planes x + 2 y + 3 z − 4 = 0 , 4 x + 3 y + 2 z + 1 = 0 and passing through
the origin will be [MP PET 1998]
(a) x + y + z = 0 (b) 17 x + 14 y + 11 z = 0 (c) 7 x + 4 y + z = 0 (d) 17 x + 14 y + z = 0
π
21. The d.r’s of normal to the plane through (1, 0, 0), (0, 1, 0 ) which makes an angle with plane x + y = 3 , are [AIEEE 2002]
4
(a) 1, 2 ,1 (b) 1,1, 2 (c) 1, 1, 2 (d) 2 , 1, 1
22. Two systems of rectangular axes have the same origin. If a plane cuts them at distance a, b, c and a', b', c' from the origin, then
1 1 1 1 1 1 1 1 1 1 1 1
(a) + + + + + =0 (b) + − + + − =0
a 2 b 2 c 2 a' 2 b ' 2 c ' 2 a 2 b 2 c 2 a' 2 b ' 2 c ' 2
1 1 1 1 1 1 1 1 1 1 1 1
(c) − − + − − =0 (d) + + − − − = 0 [AIEEE 2003]
a 2 b 2 c 2 a' 2 b ' 2 c ' 2 a 2 b 2 c 2 a' 2 b ' 2 c ' 2
x −1 y +1 z
23. If 4 x + 4 y − kz = 0 is the equation of the plane through the origin that contains the line = = , then k =
2 3 4
(a) 1 (b) 3 (c) 5 (d) 7 [MP PET 1992]
x y z
24. The distance of the point (1, –2, 3) from the plane x − y + z = 5 measured parallel to the line = = , is
2 3 −6
(a) 1 (b) 6/7 (c) 7/6 (d) None of these [AI CBSE 1984]
x −3 y−4 z −5
25. The distance of the point of intersection of the line = = and the plane x + y + z = 17 from the point (3, 4, 5)
1 2 2
is given by
(a) 3 (b) 3/2 (c) 3 (d) None of these
x −a+d y −a z −a−d x −b +c y −b z −b −c
26. The lines = = and = = are coplanar and then equation to the plane in
α −δ α α +δ β −γ β β +γ
which they lie, is
(a) x + y + z = 0 (b) x − y + z = 0 (c) x − 2y + z = 0 (d) x + y − 2z = 0
x −3 y−4 z −5
27. The line = = lies in the plane 4 x + 4 y − kz − d = 0 . The values of k and d are
2 3 4
(a) 4, 8 (b) –5, – 3 (c) 5, 3 (d) – 4, – 8
x −4 y −2 z −k
28. The value of k such that = = lies in the plane 2 x − 4 y + z = 7 , is [IIT Screening 2003]
1 1 2

73 of 77
DOWNLOAD FREE FROM www.tekoclasses.com , PH.: 0 903 903 7779, 0 98930 58881
MATHS H.O.D.: SUHAG R.KARIYA , BHOPAL, VECTORS & 3D PART 6 OF 6

(a) 7 (b) – 7 (c) No real value (d) 4


The shortest distance from the plane 12 x + 4 y + 3 z = 327 to the sphere x 2 + y 2 + z 2 + 4 x − 2 y − 6 z = 155 is [AIEEE 2003]
4
(a) 26 (b) 11 (c) 13 (d) 39
13
29. The radius of the circle in which the sphere x 2 + y 2 + z 2 + 2 x − 2 y − 4 z − 19 = 0 is cut by the plane x + 2 y + 2 z + 7 = 0 is
(a) 1 (b) 2 (c) 3 (d) 4 [AIEEE 2003]
30. The equation of motion of a rocket are: x = 2 t, y = −4 t, z = 4 t where the time 't' is given in seconds, and the co-ordinates of a
moving point in kilometers. What is the path of the rocket? At what distance will be the rocket be from the starting point 0(0, 0, 0)
in 10 seconds
(a) Straight line, 60 km (b) Straight line, 30 km (c) Parabola, 60 km (d) Ellipse, 60 km
31. The plane lx + my = 0 is rotated an angle α about its line of intersection with the plane z = 0 , then the equation to the plane in
its new position is
(a) lx + my ± z (l 2 + m 2 ) tan α = 0

(b) lx − my ± z (l 2 + m 2 ) tan α = 0

(c) lx + my ± z (l 2 + m 2 ) cos α = 0

(d) lx − my ± z (l 2 + m 2 ) cos α = 0
32. The distance between two points P and Q is d and the length of their projections of PQ on the co-ordinate planes are d1 , d 2 , d 3 .
Then d12 + d 22 + d 32 = kd 2 where ‘k’ is
(a) 1 (b) 5
(c) 3 (d) 2
33. If P1 and P2 are the lengths of the perpendiculars from the points (2,3,4) and (1,1,4) respectively from the plane
3 x − 6 y + 2 z + 11 = 0 , then P1 and P2 are the roots of the equation
(a) P 2 − 23 P + 7 = 0 (b) 7 P 2 − 23 P + 16 = 0
2
(c) P − 17 P + 16 = 0 (d) P 2 − 16 P + 7 = 0
34. The edge of a cube is of length ‘a’ then the shortest distance between the diagonal of a cube and an edge skew to it is
(a) a 2 (b) a
(c) 2 /a (d) a/ 2

Que from Compt. Exams


Co-ordinate Geometry of Three Dimensions
1 d 2 c 3 a 4 c 5 b
6 b 7 a 8 a 9 a 10 d
11 b 12 b 13 b 14 b 15 a
16 b 17 a 18 a 19 d 20 b
21 b 22 d 23 c 24 a 25 a
26 c 27 c 28 a 29 c 30 c
31 a 32 a 33 d 34 b 35 d

Que from Compt. Exams


Vector Algebra
1. Three forces of magnitudes 1, 2, 3 dynes meet in a point and act along diagonals of three adjacent faces of a cube. The
resultant force is [MNR 1987]
(a) 114 dyne (b) 6 dyne (c) 5 dyne (d) None of these
2. The vectors b and c are in the direction of north-east and north-west respectively and |b|=|c|= 4. The magnitude and
direction of the vector d = c – b, are [Roorkee 2000]
(a) 4 2 , towards north (b) 4 2 , towards west (c) 4, towards east (d) 4, towards south
2 2 2
3. If a, b and c are unit vectors, then | a − b | + | b − c | + | c − a | does not exceed[IIT Screening 2001]
(a) 4 (b) 9 (c) 8 (d) 6
4. The vectors AB = 3 i + 5 j + 4 k and AC = 5 i − 5 j + 2 k are the sides of a triangle ABC. The length of the median through
A is [UPSEAT 2004]
(a) 13 unit (b) 2 5 unit (c) 5 unit (d) 10 unit
5. Let the value of p = ( x + 4 y ) a + (2 x + y + 1) b and q = (y − 2 x + 2) a + (2 x − 3 y − 1) b , where a and b are non-collinear
vectors. If 3 p = 2 q , then the value of x and y will be [RPET 1984; MNR 1984]

74 of 77
DOWNLOAD FREE FROM www.tekoclasses.com , PH.: 0 903 903 7779, 0 98930 58881
MATHS H.O.D.: SUHAG R.KARIYA , BHOPAL, VECTORS & 3D PART 6 OF 6

(a) – 1, 2 (b) 2, – 1 (c) 1, 2 (d) 2, 1\


6. The points D, E, F divide BC, CA and AB of the triangle ABC in the ratio 1 : 4, 3 : 2 and 3 : 7 respectively and the point
K divides AB in the ratio 1 : 3 , then ( AD + BE + CF ) : CK is equal to [MNR 1987]
(a) 1 : 1 (b) 2 : 5 (c) 5:2 (d) None of these
7. If two vertices of a triangle are i − j and j + k , then the third vertex can be [Roorkee 1995]
(a) i + k (b) i − 2 j − k (c) i−k (d) 2i − j
(e) All the above
15 k
8. If a of magnitude 50 is collinear with the vector b = 6 i − 8 j − , and makes an acute angle with the positive
2
direction of z-axis, then the vector a is equal to
[Pb. CET 2004]
(a) 24 i − 32 j + 30 k (b) −24 i + 32 j + 30 k (c) 16 i − 16 j − 15 k (d) −12 i + 16 j − 30 k
9. If three non-zero vectors are a = a1 i + a 2 j + a 3 k, b = b 1 i + b 2 j + b 3 k and c = c 1 i + c 2 j + c 3 k. If c is the unit vector
2
a1 a2 a3
π
perpendicular to the vectors a and b and the angle between a and b is , then b1 b2 b3 is equal to
6
c1 c2 c3
3 (Σa12 ) (Σb12 ) (Σc12 ) (Σa12 ) (Σb12 )
(a) 0 (b) (c) 1 (d)
4 4
10. Let the unit vectors a and b be perpendicular and the unit vector c be inclined at an angle θ to both a and b. If
c = α a + β b + γ (a × b ), then [Orissa JEE 2003]

(a) α = β = cos θ , γ 2 = cos 2θ (b) α = β = cos θ , γ 2 = − cos 2θ


(c) α = cos θ , β = sin θ , γ 2 = cos 2θ (d) None of these
11. The vector a + b bisects the angle between the vectors a and b, if
(a) | a | =| b | (b)| a | =| b | or angle between a and b is zero
(c) | a | = m | b | (d) None of these
12. The points O, A, B, C, D are such that OA = a , OB = b , OC = 2 a + 3 b and OD = a − 2b . If | a | = 3| b |, then the
angle between BD and AC is
π π π
(a) (b) (c) (d) None of these
3 4 6
13. If A = i + 2 j + 3 k , B = −i + 2 j + k and C = 3 i + j, then the value of t such that A + t B is at right angle to vector C, is
[RPET 2002]
(a) 2 (b) 4 (c) 5 (d) 6
14. Let b = 4 i + 3 j and c be two vectors perpendicular to each other in the xy-plane. All vectors in the same plane having
projections 1 and 2 along b and c respectively, are given by [IIT 1987]
2 11 2 11 2 11 2 11
(a) 2 i − j, i + j (b) 2 i + j, − i + j (c) 2i + j, − i − j (d) 2i − j, − i + j
5 5 5 5 5 5 5 5
15. Let a = 2 i − j + k, b = i + 2 j − k and c = i + j − 2 k be three vectors. A vector in the plane of b and c whose projection
on a is of magnitude 2 / 3 is
[IIT 1993; Pb. CET 2004]
(a) 2 i + 3 j − 3 k (b) 2i + 3 j + 3 k (c) − 2i − j + 5 k (d) 2i + j + 5 k
16. A vector a has components 2p and 1 with respect to a rectangular cartesian system. The system is rotated through a
certain angle about the origin in the anti-clockwise sense. If a has components p+1 and 1 with respect to the new
system, then [IIT 1984]
1 1
(a) p = 0 (b) p = 1 or − (c) p = −1 or (d) p = 1 or −1
3 3
17. If u = 2 i + 2 j − k and v = 6 i − 3 j + 2 k, then a unit vector perpendicular to both u and v is [MP PET 1987]
1 1 18  1
(a) i − 10 j − 18 k (b)  i − 2j − k  (c) (7 i − 10 j − 18 k ) (d) None of these
17  5 5  473
18. If a = 2i + k, b = i + j + k and c = 4 i − 3 j + 7 k. If d × b = c × b and d . a = 0, then d will be [IIT 1990]
(a) i + 8 j + 2 k (b) i − 8 j + 2 k (c) −i + 8 j − k (d) −i − 8 j + 2 k
19. If a × r = b + λa and a . r = 3, where a = 2 i + j − k and b = −i − 2 j + k , then r and λ are equal to
7 2 6 7 2 5 6 2 6
(a) r = i + j, λ = (b) r = i + j, λ = (c) r= i + j, λ = (d) None of these
6 3 5 6 3 6 7 3 5

75 of 77
DOWNLOAD FREE FROM www.tekoclasses.com , PH.: 0 903 903 7779, 0 98930 58881
MATHS H.O.D.: SUHAG R.KARIYA , BHOPAL, VECTORS & 3D PART 6 OF 6

20. Let the vectors a, b, c and d be such that (a × b ) × (c × d ) = 0 . Let P1 and P2 be planes determined by pair of vectors a,
b and c, d respectively. Then the angle between P1 and P2 is [IIT Screening 2000; MP PET 2004]
π π π
(a) 0 o (b) (c) (d)
4 3 2
21. If a = i + j + k , a . b = 1 and a × b = j − k, then b = [IIT Screening 2004]
(a) i (b) i − j + k (c) 2 j − k (d) 2i
22. The position vectors of the vertices of a quadrilateral ABCD are a, b, c and d respectively. Area of the quadrilateral
formed by joining the middle points of its sides is [Roorkee 2000]
1 1
(a) | a ×b + b ×d +d ×a| (b) b ×c + c ×d + a ×d + b ×a
4 4
1 1
(c) a ×b + b ×c + c ×d + d×a (d) b ×c + c ×d +d×b
4 4
23. The moment about the point M (−2, 4 , − 6 ) of the force represented in magnitude and position by AB where the points
A and B have the co-ordinates (1, 2, − 3) and (3, − 4 , 2) respectively, is [MP PET 2000]
(a) 8i – 9j – 14k (b) 2i – 6j + 5k (c) – 3i + 2j – 3k (d)
– 5i + 8j – 8k
1 1 1
24 . If the vectors ai + j + k , i + b j + k and i + j + ck (a ≠ b ≠ c ≠ 1) are coplanar, then the value of + + =
1−a 1−b 1−c
[BIT Ranchi 1988; RPET 1987;IIIT 1987; DCE 2001; MP PET 2004; OR ISSA JE E 2005]
1 1
(a) – 1 (b) − (c) (d) 1
2 2
25. If α (a × b ) + β (b × c ) + γ (c × a ) = 0 and at least one of the numbers α , β and γ is non-zero, then the vectors a, b and
c are
(a) Perpendicular (b) Parallel (c) Coplanar (d) None of these
26. The volume of the tetrahedron, whose vertices are given by the vectors −i + j + k, i − j + k and i + j − k with reference
to the fourth vertex as origin, is
5 2 3
(a) cubic unit (b) cubic unit (c) cubic unit (d) None of these
3 3 5
27. Let a = i − j, b = j − k , c = k − i. If dˆ is a unit vector such that a . dˆ = 0 = [b c dˆ ], then dˆ is equal to [IIT 1995]
i + j−k i + j+k i + j − 2k
(a) ± (b) ± (c) ± (d) ± k
3 3 6
28. The value of 'a' so that the volume of parallelopiped formed by i + aj + k , j + a k and a i + k becomes minimum is
[IIT Screening 2003]
1
(a) – 3 (b) 3 (c) (d) 3
3
a . (b × c )
29. If b and c are any two non-collinear unit vectors and a is any vector, then (a . b ) b + (a . c ) c + (b × c ) =
| b×c|
[IIT 1996]
(a) a (b) b (c) c (d) 0
b +c
30. If a, b, c are non-coplanar unit vectors such that a × (b × c ) = , then the angle between a and b is [IIT 1995]
2
π π 3π
(a) (b) (c) (d) π
4 2 4
31. [(a × b ) × (b × c ) (b × c ) × (c × a ) (c × a ) × (a × b )] =
(a) [a b c ] 2 (b) [a b c ] 3 (c) [a b c ]4 (d) None of these
1 1 1
32. Unit vectors a, b and c are coplanar. A unit vector d is perpendicular to them. If (a × b ) × (c × d ) = i − j + k and
6 3 3
the angle between a and b is 30 o , then c is [Roorkee Qualifying 1998]
(i − 2 j + 2 k ) (2 i + j − k ) (−i + 2 j − 2 k ) (−i + 2 j + k )
(a) (b) (c) (d)
3 3 3 3
33. The radius of the circular section of the sphere | r | = 5 by the plane r . (i + j + k ) = 3 3 is [DCE 1999]
(a) 1 (b) 2 (c) 3 (d) 4
34. If x is parallel to y and z where x = 2i + j + αk , y = αi + k and z = 5 i − j , then α is equal to [J & K 2005]

(a) ± 5 (b) ± 6 (c) ± 7 (d) None of these

76 of 77
DOWNLOAD FREE FROM www.tekoclasses.com , PH.: 0 903 903 7779, 0 98930 58881
MATHS H.O.D.: SUHAG R.KARIYA , BHOPAL, VECTORS & 3D PART 6 OF 6

35. The vector c directed along the internal bisector of the angle between the vectors a = 7 i − 4 j − 4 k and b = −2i − j + 2 k
with | c | = 5 6 , is
5 5 5 5
(a) (i − 7 j + 2 k ) (b) (5 i + 5 j + 2 k ) (c) (i + 7 j + 2 k ) (d) (−5 i + 5 j + 2 k )
3 3 3 3
36. The distance of the point B (i + 2 j + 3 k ) from the line which is passing through A (4 i + 2 j + 2 k ) and which is parallel to
the vector C = 2i + 3 j + 6 k is [Roorkee 1993]

(a) 10 (b) 10 (c) 100 (d) None of these


37. Let a, b, c are three non-coplanar vectors such that
r1 = a − b + c , r2 = b + c − a , r3 = c + a + b ,
r = 2 a − 3 b + 4 c . If r = λ1 r1 + λ 2 r2 + λ 3 r3 , then
(a) λ1 = 7 (b) λ1 + λ 3 = 3 (c) λ1 + λ 2 + λ 3 = 4 (d) λ3 + λ2 = 2
38. Let a = 2 i + j + k , b = i + 2 j − k and a unit vector c be coplanar. If c is perpendicular to a, then c =
[IIT 1999; Pb. CET 2003; DCE 2005]
1 1 1 1
(a) (− j + k ) (b) (−i − j − k ) (c) (i − 2 j) (d) (i − j − k )
2 3 5 3
39. Let p, q, r be three mutually perpendicular vectors of the same magnitude. If a vector x satisfies equation
p × {(x − q ) × p} + q × {(x − r) × q} + r × {(x − p ) × r} = 0, then x is given by [IIT 1997 Cancelled]
1 1 1 1
(a) (p + q − 2 r) (b) (p + q + r) (c) (p + q + r) (d) (2 p + q − r)
2 2 3 3
40. The point of intersection of r × a = b × a and r × b = a × b , where a = i + j and b = 2i − k is [Orissa JEE 2004]
(a) 3 i + j − k (b) 3 i − k (c) 3i + 2 j + k (d) None of these

Que from Compt. Exams


Vector Algebra

1 c 2 b 3 b 4 c 5 b
6 b 7 e 8 b 9 d 10 b
11 b 12 d 13 c 14 d 15 a,c
16 b 17 b 18 d 19 b 20 a
21 a 22 c 23 a 24 d 25 c
26 b 27 c 28 c 29 a 30 c
31 c 32 a,c 33 d 34 c 35 a
36 b 37 b,c 38 a 39 b 40 a

for 39 Yrs. Que. of IIT-JEE


&
15 Yrs. Que. of AIEEE
we have distributed already a book

77 of 77

You might also like